2019 NDEB_SOLVED

Ace your homework & exams now with Quizwiz!

When smokers are compared to nonsmokers, probing depths after surgical periodontal treatment are A. greater. B. smaller. C. the same.

A. greater.

A salivary calculus is a A. sialolith. B. rhinolith. C. phlebolith. D. thrombolith.

A. sialolith.

Which of the following is NOT a true cyst? A. Dentigerous cyst. B. Simple bone cyst/traumatic bone cyst. C. Radicular cyst. D. Nasopalatine cyst.

B. Simple bone cyst/traumatic bone cyst.

A mandibular fracture during normal mastication is most likely to occur in a patient with A. osteoporosis. B. a large intraosseous lesion. C. an impacted tooth at the inferior border. D. advanced alveolar atrophy.

B. a large intraosseous lesion.

Pulp polyp is A. an acute pathological lesion. B. a proliferative reaction of the pulp. C. frequently found in elderly patients. D. accompanied by severe pain.

B. a proliferative reaction of the pulp.

In a 5 year old, a small mechanical exposure in a vital primary molar would be treated by A. extraction of the tooth. B. a pulp capping with calcium hydroxide. C. a routine amalgam restoration without any specific treatment for the exposed pulp. D. the use of a cavity liner.

B. a pulp capping with calcium hydroxide.

Ceramics used in dentistry exhibit A. greater strength in tension than compression. B. a tendency for tensile fracture. C. chemical instability. D. high thermal coefficients of expansion.

B. a tendency for tensile fracture.

Oral nitrate is used to treat the symptoms of A. hypertension. B. angina. C. arrhythmia. D. tachycardia.

B. angina.

Pulpotomy is the treatment of choice in carious pulp exposures of A. asymptomatic vital teeth with completely formed apices. B. asymptomatic vital teeth with incompletely formed apices. C. asymptomatic necrotic teeth with completely formed apices. D. asymptomatic necrotic teeth with incompletely formed apices.

B. asymptomatic vital teeth with incompletely formed apices.

The physiopathology of sleep apnea is most likely related to A. excessive oropharyngeal muscular tonus during sleep. B. central nervous system respiratory depression. C. obstruction in lower airways. D. a severe Angle Class III malocclusion.

B. central nervous system respiratory depression.

In recording centric relation registration, perforation of the recording material must be avoided because A. the material will undergo dimensional changes. B. contact of teeth could deflect the mandible. C. the recording material will be too weak and may fracture or tear.

B. contact of teeth could deflect the mandible.

An incisional biopsy of a clinically suspicious malignant lesion fails to support the clinical diagnosis. The most appropriate management is to A. reassure the patient that there is no malignancy. B. contact the pathologist to discuss the case. C. recall the patient in six months. D. perform a cytological smear.

B. contact the pathologist to discuss the case.

Increasing the kVp results in decreased A. density of the image. B. contrast of the image. C. energy of the x-ray beam.

B. contrast of the image.

Veillonella species in supragingival plaque A. act symbiotically with S. mutans to decrease pH and promote caries activity. B. convert lactate to acetic and propionic acid. C. enhance the progression of caries by metabolizing sucrose. D. act as pioneer microorganisms in the development of plaque

B. convert lactate to acetic and propionic acid.

The custom tray used in making a final complete denture impression must A. extend to the bottom of the vestibule. B. create adequate space for the impression material. C. have a horizontal handle. D. be stored in water until ready for use.

B. create adequate space for the impression material.

The lamina dura is A. spongy bone. B. cribriform plate. C. hypercalcified bone. D. compact bone.

B. cribriform plate.

The purpose of using intensifying screens in extraoral radiography is to A. improve resolution. B. decrease exposure time. C. decrease scatter radiation. D. increase sharpness

B. decrease exposure time.

Angular cheilitis in older patients who wear complete dentures is most likely associated with A. antibiotic therapy. B. decreased vertical dimension. C. decreased salivary flow. D. vitamin B deficiency.

B. decreased vertical dimension.

In radiography, too high a temperature of the developer will cause increased A. contrast. B. density. C. grey levels. D. resolution.

B. density.

Procaine is a local anaesthetic which is chemically classified as an A. amide. B. ester. C. aldehyde. D. ethamine. E. aminide.

B. ester.

Chlorpromazine produces all of the following EXCEPT A. photosensitivity. B. excessive salivation. C. anticholinergic effects. D. antiadrenergic effects.

B. excessive salivation.

The epithelial attachment does not migrate apically in A. juvenile periodontitis. B. hyperplastic gingivitis. C. chronic periodontitis. D. rapidly progressive periodontitis.

B. hyperplastic gingivitis.

The respiration of a patient with chronic obstructive pulmonary disease (COPD) is primarily controlled by A. hypercapnia. B. hypoxia. C. alkalosis. D. baroreceptors.

B. hypoxia.

In primary molars, radiographic bony changes from an infection are initially seen A. at the apices. B. in the furcation area. C. at the alveolar crest. D. at the base of the developing tooth.

B. in the furcation area.

The hybrid layer is composed of resin tags and A. decalcified peritubular dentin. B. intertubular dentin. C. resin plugs in the dentin surface. D. bonding agent at the interface.

B. intertubular dentin.

In Turner's syndrome, the patient is most likely to exhibit A. delayed tooth eruption. B. micrognathia. C. a wide maxilla. D. oligodontia.

B. micrognathia.

A benign neoplasm of bone is called a/an A. fibrous dysplasia. B. osteoma. C. torus. D. sarcoma. E. osteosarcoma.

B. osteoma.

The best way to increase the working time of a polyvinylsiloxane is to A. change the catalyst/base ratio. B. refrigerate the material. C. add oleic acid. D. reduce mixing time.

B. refrigerate the material.

After an inferior alveolar nerve block injection, a patient would develop seventh nerve paralysis if the injection was made into the A. internal maxillary artery. B. retroparotid space. C. internal pterygoid muscle. D. retromandibular vein. E. pterygoid plexus of veins

B. retroparotid space.

Repair of periodontal tissues is the replacement of lost tissue with one that is similar in A. function. B. structure. C. structure and function.

B. structure.

The gingivectomy is a useful technique to eliminate A. infrabony pockets. B. suprabony pockets. C. thick osseous ledges. D. mucogingival problems

B. suprabony pockets.

What is the most important factor to consider when deciding whether or not to use pulp protection? A. depth of the pulpal floor. B. thickness of the remaining dentin. C. amount of carious material removed. D. location of the carious lesion.

B. thickness of the remaining dentin.

The predominant type of movement produced by a finger spring on a removable appliance is A. torque. B. tipping. C. rotation. D. translation.

B. tipping.

The normal position of the alveolar crest in healthy periodontium is A. 1 to 2mm coronal to the CEJ. B. at the CEJ. C. 1 to 2mm apical to the CEJ. D. 3 to 4mm apical to the CEJ.

C. 1 to 2mm apical to the CEJ.

What is the threshold count of S. mutans in mixed saliva at which a patient is deemed "high risk" for caries? A. 100/ml. B. 10,000/ml. C. 1,000,000/ml. D. 100,000,000/ml.

C. 1,000,000/ml.

A 74 year old patient being treated for hypertension requires deep scaling using local anesthesia. It is appropriate to defer treatment if the patient's blood pressure is equal to or above A. 120/80. B. 140/90. C. 160/110. D. 180/110.

C. 160/110.

The volumetric polymerization shrinkage of a hybrid composite resin is in the order of A. 0%. B. 0.1 - 1.0%. C. 2 - 8%. D. 10 - 15%.

C. 2 - 8%.

What is the most appropriate film size for making an occlusal radiograph in a 3 year old patient? A. 0. B. 1. C. 2. D. 4.

C. 2.

For an optimum esthetic result, what is the MINIMUM time to wait after an anterior tooth extraction before making a final impression for a fixed partial denture? A. 2 weeks. B. 1 month. C. 3 months. D. 6 months. E. 1 year.

C. 3 months.

For the extraction of maxillary teeth, the most appropriate position of the maxillary occlusal plane is A. parallel to the floor. B. 20º to the floor. C. 60º to the floor. D. 90º to the floor.

C. 60º to the floor.

In inhalation analgesia, what is the safe maximal nitrous oxide concentration that can be delivered? A. 50%. B. 60%. C. 70%. D. 80%.

C. 70%.

Which of the following exposure factors will result in the most penetrating x-rays? A. 10kVp - 65mA. B. 85kVp - 5mA. C. 90kVp - 10mA. D. 65kVp - 15mA. E. 75kVp - 40mA.

C. 90kVp - 10mA.

Which of the following, if left untreated, is most likely to result in a periapical lesion? A. Internal resorption. B. Reversible pulpitis. C. Acute suppurative pulpitis. D. Chronic hyperplastic pulpitis. E. Diffuse calcification of the pulp.

C. Acute suppurative pulpitis.

Which of the following impression materials will still result in an accurate cast when poured two weeks after making the impression? A. Polyether. B. Polysulfide. C. Addition silicone. D. Condensation silicone.

C. Addition silicone.

The principal microorganism in localized aggressive periodontitis is A. Porphyromonas gingivalis. B. Fusobacterium nucleatum. C. Aggregatobacter (Actinobacillus) actinomycetemcomitans. D. Prevotella intermedia.

C. Aggregatobacter (Actinobacillus) actinomycetemcomitans.

Which of the following radiographs is best to diagnose caries and early alveolar bone loss? A. Periapical. B. Occlusal. C. Bitewing. D. Lateral jaw. E. Panoramic.

C. Bitewing.

Which of the following is increased after stimulating α1-adrenergic receptors? A. Contractility of the heart. B. SA node activity. C. Blood vessel constriction. D. Bronchial dilation.

C. Blood vessel constriction.

An infant has asymptomatic small whitish outgrowths at the junction of the soft palate and hard palate. What is the most likely diagnosis? A. Dental lamina cysts. B. Epstein pearls. C. Bohn nodules. D. Fordyce granules.

C. Bohn nodules.

Which of the following are the most appropriate for use as overdenture abutments? A. Central incisors. B. Second premolars. C. Canines. D. First premolars.

C. Canines.

The predominant immunoglobulin isolated from saliva is A. IgG. B. IgM. C. IgA. D. IgD.

C. IgA.

Which statement is true regarding amalgam? A. A thin layer can be left to seal the cavosurface margins. B. It bonds on its own to tooth structure. C. It wears at a rate similar to that of tooth structure. D. It is time-consuming to place compared to composite resin.

C. It wears at a rate similar to that of tooth structure.

What is the best imaging modality to assess the TMJ disc? A. Arthrography. B. Computed tomography. C. Magnetic resonance imaging. D. Corrected conventional tomography.

C. Magnetic resonance imaging.

Which of the following pontic designs will make it most difficult to maintain optimal oral hygiene? A. Modified ridge lap. B. Hygienic. C. Saddle. D. Conical. E. Ovoid.

C. Saddle.

Which drug is most adversely affected by ingestion of antacids? A. Cephalexin. B. Erythromycin. C. Tetracycline. D. Penicillin V.

C. Tetracycline.

located A. within basal bone. B. coronal to alveolar bone crest. C. apical to alveolar bone crest.

C. apical to alveolar bone crest.

Sclerosing osteitis/condensing osteitis in the periapical region is indicative of a/an A. acute inflammation of the pulp. B. pulpal abscess. C. chronic inflammation of the pulp. D. early apical abscess formation.

C. chronic inflammation of the pulp.

Before performing periodontal surgery, it is important to A. prescribe a mouthwash. B. prescribe systemic antibiotics. C. control plaque.

C. control plaque.

The inherited defect of teeth that may be associated with osteogenesis imperfecta is A. amelogenesis imperfecta. B. dentin dysplasia. C. dentinogenesis imperfecta. D. taurodontism. E. regional odontodysplasia.

C. dentinogenesis imperfecta.

Subgingival plaque in deep periodontal pockets consists primarily of A. gram-positive microorganisms. B. aerobic microorganisms. C. gram-negative microorganisms. D. viruses.

C. gram-negative microorganisms.

Vitamin D is activated in the A. skin upon ultraviolet radiation from the sun. B. liver upon hydroxylation. C. kidney upon hydroxylation. D. intestinal mucosa upon absorption.

C. kidney upon hydroxylation.

A possible manifestation of an uncomplicated mandibular fracture is A. diplopia. B. dyspnea. C. malocclusion. D. facial paralysis. E. orbital swelling.

C. malocclusion.

Damage to the lingual nerve following a mandibular third molar extraction is A. preventable in most cases. B. not likely to recover(?). C. more frequent than damage to the inferior alveolar nerve. D. too rare to inform the patient during consent.

C. more frequent than damage to the inferior alveolar nerve.

Increasing the kilovoltage setting on the dental x-ray machine results in A. more gamma radiation. B. greater collimation. C. more penetration. D. greater secondary radiation at the level of the skin.

C. more penetration.

When compared to permanent teeth, primary teeth have A. a greater thickness of enamel. B. a greater thickness of dentin. C. more prominent cervical constriction. D. pulps which are smaller in relation to crown size.

C. more prominent cervical constriction.

The muscle of the floor of the mouth in the molar region which requires special attention in the final mandibular denture impression is the A. genioglossus. B. geniohyoid. C. mylohyoid. D. hyoglossus.

C. mylohyoid.

Metronidazole can be used to treat A. denture stomatitis. B. recurrent aphthous ulcers. C. necrotizing ulcerative gingivitis (NUG). D. primary herpetic gingivostomatitis.

C. necrotizing ulcerative gingivitis (NUG).

The roots of the first permanent molar should be completely formed by the age of A. six years. B. seven years. C. nine years. D. eleven years. E. thirteen years.

C. nine years.

Desquamation of the gingiva usually occurs as a result of A. inflammation. B. benign neoplasia. C. normal cell turnover. D. a developmental abnormality.

C. normal cell turnover.

Local anesthetics block nerve conduction by interfering with ionic movement of A. calcium. B. potassium. C. sodium. D. chloride.

C. sodium.

The maximum recommended number of 1.8ml cartridges of 2% lidocaine with epinephrine 1:100,000 that may be safely administered to a 17kg child is approximately A. 0.5. B. 1. C. 1.5. D. 2. E. 2.5.

D. 2.

Daily fluid loss from a healthy individual at rest in a temperate climate is approximately A. 250ml. B. 500ml. C. 1.0L. D. 2.5L. E. 5.0L.

D. 2.5L.

For acute dental pain, the daily maximum cumulative dose of acetaminophen is A. 2400 mg. B. 3200 mg. C. 3600 mg. D. 4000 mg.

D. 4000 mg.

What is the maximum number of cartridges (1.8ml) of a 2 local anesthetic solution that can be administered without exceeding a total dose of 300mg? A. 2. B. 4. C. 6. D. 8. E. 10.

D. 8.

A patient complains of lip and tongue hypersensitivity (allodynea) following intake of hot, spicy food. The sensory nerve fibers that are associated with this form of pain are A. A-alpha. B. A-beta. C. A-gamma. D. A-delta and C.

D. A-delta and C.

Which of the following is the most predictable indicator of periodontal stability? A. Patient compliance with maintenance visit. B. Gingival inflammation. C. Plaque index. D. Absence of bleeding on probing.

D. Absence of bleeding on probing.

Assuming the daily maximum is not exceeded, which of the following is/are appropriate for pain management for an adult with a history of severe asthma and nasal polyps following an emergency pulpectomy? A. Naproxen 250 mg, every 6 to 8 hours. B. Acetylsalicylic acid 650 mg every 4 hours. C. Ketorolac 10 mg every 4 hours. D. Acetaminophen 1000 mg every 6 hours.

D. Acetaminophen 1000 mg every 6 hours.

Which of the following bacterial types is implicated in the initiation of gingivitis? A. Streptococcus salivarius. B. Streptococcus mutans. C. Leptothrix buccalis. D. Actinomyces viscosus.

D. Actinomyces viscosus.

Collagen A. is most common in hard tissues. B. forms insoluble high tensile strength fibres. C. has a triple helical structure. D. All of the above.

D. All of the above.

Endotoxin is A. a cell wall component of gram-negative bacteria. B. a potent inflammatory agent. C. present in diseased root cementum. D. All of the above.

D. All of the above.

A person who has sickle cell anemia may show certain radiographic changes in the bones of the skull. These changes may be A. ‟punched-out" radiolucent lesions. B. a moth-eaten appearance of the bone. C. gross irregularities with exostosis formation. D. a "hair on end" sign.

D. a "hair on end" sign.

A patient has a history of shortness of breath and ankle edema. You would suspect A. asthma. B. emphysema. C. rhinophyma. D. cardiac insufficiency.

D. cardiac insufficiency.

The principal component of the fibres of the periodontal ligament is A. elastin. B. reticulin. C. fibronectin. D. collagen.

D. collagen.

Hutchinson's incisors and mulberry molars are associated with A. congenital porphyria. B. fluorosis. C. rickets. D. congenital syphilis. E. cleidocranial dysplasia.

D. congenital syphilis.

Continued smoking will impair wound healing following a surgical procedure because of A. stain development. B. increased rate of plaque formation. C. increased rate of calculus formation. D. contraction of peripheral blood vessels. E. superficial irritation to tissues by smoke.

D. contraction of peripheral blood vessels.

Treatment of primary herpectic gingivostomatitis should include A. topical steroids. B. application of dilute hydrogen peroxide. C. cauterization. D. control of secondary infection.

D. control of secondary infection.

The setting of a zinc-phosphate cement can best be retarded by A. decreasing the particle size. B. increasing the concentration of water in the liquid. C. increasing the rate of addition of the powder to the liquid. D. cooling the glass mixing slab.

D. cooling the glass mixing slab.

With age, dentinal tubules become occluded because odontoblast cellular projections A. produce organic molecules. B. swell and expand. C. absorb dentinal fluid. D. deposit more mineral.

D. deposit more mineral.

Accessory root canals develop because root odontoblasts fail to A. produce matrix. B. survive. C. divide. D. differentiate.

D. differentiate.

A fixed partial denture with a single pontic is deflected a certain amount, a span of two similar pontics will deflect A. the same amount. B. twice as much. C. four times as much. D. eight times as much.

D. eight times as much.

Vestibuloplasty is a preprosthetic surgical procedure used to A. facilitate reliable impression making. B. provide adequate posterior inter-arch space. C. allow placement of teeth over the residual ridge. D. increase the supporting surface area.

D. increase the supporting surface area.

In neurons, glutamate is an amino acid that binds only to A. nuclear receptors. B. ionotropic receptors. C. metabotropic receptors. D. ionotropic and metabotropic receptors.

D. ionotropic and metabotropic receptors.

Alginate impression material A. is a reversible hydrocolloid. B. sets by condensation polymerization. C. is a rigid material. D. is an irreversible material. E. is a thermoplastic material.

D. is an irreversible material.

An increase of immunoglobulins is consistent with increased numbers of A. fibroblasts. B. neutrophils. C. lymphocytes. D. plasma cells.

D. plasma cells.

After the age of 6 years, growth of the mandible is greatest A. at the symphysis. B. between canines. C. along the lower border. D. posterior to first molars.

D. posterior to first molars.

A post is used in an endodontically treated tooth to A. obturate the canal. B. reinforce the root. C. reinforce the remaining coronal tooth structure. D. retain the core build up.

D. retain the core build up.

A post is used in an endodontically treated tooth to A. obturate the canal. B. strengthen the root. C. reinforce the remaining crown. D. retain the restoration.

D. retain the restoration.

Fordyce spots or granules is/are A. sweat glands. B. implanted epithelium. C. cystic formations. D. sebaceous glands. E. hyperkeratosis.

D. sebaceous glands.

The chemical that is used to retard the setting reaction in alginate impression materials is A. calcium sulfate. B. sodium sulfate. C. calcium phosphate. D. sodium phosphate.

D. sodium phosphate.

Compared with zinc-phosphate cement, polycarboxylate cement has A. longer working time. B. lower film thickness. C. increased compressive strength. D. superior biologic compatibility.

D. superior biologic compatibility.

Inclusion bodies in the nucleus or cytoplasm of cells are diagnostic of A. rickettsia. B. parasitic infestations. C. bacterial diseases. D. viral diseases.

D. viral diseases.

A 14 year old boy presents with bilateral white thickening of the buccal mucosa which has been present since birth. His brother has similar lesions. The most likely diagnosis is A. leukoplakia. B. lichen planus. C. mucous patches. D. white sponge nevus.

D. white sponge nevus.

Dental plaque is composed of A. desquamated epithelial cells. B. components from oral secretions. C. bacteria and their products. D. cuticle or pellicle. E. All of the above.

E. All of the above.

The management of syncope following local anesthetic administration does NOT include A. elevating the legs. B. placing in a supine position. C. administering oxygen. D. ensuring the airway is open. E. administering epinephrine.

E. administering epinephrine.

A hinge axis facebow records A. Bennett angle. B. centric relation. C. lateral condylar inclination. D. horizontal condylar inclination. E. opening and closing axis of the mandible.

E. opening and closing axis of the mandible.

When performing endodontic treatment on a vital tooth, the most appropriate termination point of apical root canal preparation is A. 0.5 to 1mm short of the radiographic apex. B. 3mm short of the radiographic apex. C. slightly through the apical foramen. D. to the point where the patient feels sensation

A. 0.5 to 1mm short of the radiographic apex.

What percentage of natal teeth are supernumerary? A. 10. B. 50. C. 90.

A. 10.

A patient with amyloidosis is taking 20 mg prednisone once a day. What is the minimum number of weeks taking the drug that triggers the need for corticosteroid prophylaxis before proceeding with major dentoalveolar surgery under general anesthesia? A. 2. B. 4. C. 6. D. 8.

A. 2.

Ideally, within how many hours should one receive medical attention for percutaneous exposure to blood borne pathogens? A. 2. B. 4. C. 6. D. 8.

A. 2.

When placing an implant in the posterior mandible, the minimum thickness of bone between the implant and the inferior alveolar nerve should be A. 2mm. B. 4mm. C. 6mm. D. 8mm.

A. 2mm.

The most appropriate recall interval for an 8 year old patient with high caries risk is A. 3 months. B. 6 months. C. 9 months. D. 12 months.

A. 3 months.

A protrusive relation record should be made by instructing the patient to protrude the mandible A. 3-6mm. B. 7-10mm. C. 11-13mm.

A. 3-6mm.

To initiate caries, bacterial plaque pH must reach A. 5.4. B. 5.9. C. 6.4. D. 6.9.

A. 5.4.

The microorganism most commonly associated with root surface caries is A. Actinomyces viscosus. B. Streptococcus mutans. C. Streptococcus salivarius. D. Lactobacillus acidophilus.

A. Actinomyces viscosus.

What is the most likely diagnosis for a patient who exhibits fatigue, nausea and yellow sclera? A. Acute viral hepatitis. B. Appendicitis. C. Peptic ulcer. D. Ulcerative colitis.

A. Acute viral hepatitis.

Which of the following is NOT correlated to early implant failure? A. Age of the patient. B. Type III/IV bone around the implant. C. Poorly controlled diabetes. D. Regular smoking habit.

A. Age of the patient.

In comparing polysulfide, polyether and addition cured silicone impression materials, which of the following statements is true? A. All three of the materials contract slightly during curing. B. All three of the materials expand slightly upon cooling from mouth temperature (37°C) to room temperature (20°C). C. After one week, addition cured silicones will undergo more distortion than polysulfides. D. Lead oxide is used as an activator in silicones.

A. All three of the materials contract slightly during curing.

What is the most likely diagnosis of a white lesion on the retromolar pad opposing a non-functional molar? A. Alveolar ridge keratosis. B. Candidiasis. C. Lichen planus. D. Squamous cell carcinoma

A. Alveolar ridge keratosis

A 24 year old patient complains of abdominal pain, frequent diarrhea and weight loss. The oral clinical examination shows linear mucosal ulcers with hyperplastic margins in the buccal vestibule. What is the most likely diagnosis? A. Crohn's disease. B. Leukemia. C. AIDS. D. Diabetes mellitus.

A. Crohn's disease.

A bitewing radiograph of an early mixed dentition should include the following proximal surfaces. A. Distal of the primary canine to distal of the permanent first molar. B. Distal of the primary canine to mesial of the permanent first molar. C. Mesial of the primary first molar to mesial of the permanent first molar. D. Mesial of the primary first molar to distal of the permanent first molar.

A. Distal of the primary canine to distal of the permanent first molar.

The smear layer present on the root canal wall after cleaning and shaping is best removed by using A. EDTA. B. hydrogen peroxide. C. chlorhexidine. D. isopropyl alcohol.

A. EDTA.

Which radiographic finding supports the diagnosis of lingual displacement of the root of an intruded primary incisor? A. Elongation. B. Foreshortening. C. Increased opacity. D. Widened periodontal ligament space.

A. Elongation.

Which of the following should NOT be administered to a patient with chest pain consistent with a myocardial infarction? A. Epinephrine. B. Nitroglycerin. C. Oxygen. D. Morphine. E. Acetylsalicylic acid.

A. Epinephrine.

Of the following structures, which would be projected closest to the occlusal plane when taking a mandibular posterior periapical radiograph? A. External oblique ridge. B. Mandibular canal. C. Submandibular salivary gland fossa. D. Mental foramen.

A. External oblique ridge.

In which of the following conditions is there a risk of malignant change after radiotherapy? A. Fibrous dysplasia. B. Mucocele. C. Lymphangioma. D. Torus palatinus.

A. Fibrous dysplasia.

Which type of headgear is most appropriate for the management of a 10 year old patient with an increased lower third of the face, incompetent lips, an Angle Class II malocclusion and vertical maxillary excess? A. High-pull. B. Cervical. C. Combination. D. Protraction

A. High-pull.

Which of the following characteristics is common to all types of aphthous ulcers? A. History of pain. B. Involvement of keratinized mucosa. C. Presence of vesicles. D. Presence of fever. E. Scar formation after healing.

A. History of pain.

What type of insurance must a dentist carry in order to practice dentistry in Canada? A. Malpractice. B. Office overhead. C. General liability. D. Long term disability. E. Employment.

A. Malpractice.

Zinc phosphate cement, when used as a luting agent, has which of the following properties? A. Mechanical retention. B. Insolubility. C. Anticariogenicity. D. Chemical adhesion.

A. Mechanical retention.

What is the alloy of choice for the framework of a resin-bonded fixed bridge? A. Nickel-chromium. B. Gold-palladium. C. Gold-silver. D. Gold-platinum-palladium.

A. Nickel-chromium.

What is the most appropriate space management for a 4 year old patient who has lost all their primary maxillary incisors due to trauma? A. No treatment. B. A removable Hawley appliance. C. A fixed lingual holding arch. D. A fixed Nance button appliance

A. No treatment.

What is the most appropriate management for nocturnal bruxism in the presence of malocclusion? A. Occlusal appliance, control of parafunctional habits and stress reduction techniques. B. Muscle relaxants, calcium channel blockers and benzodiazepines. C. Prosthetic reconstruction. D. Physiotherapy and removal of occlusal interferences.

A. Occlusal appliance, control of parafunctional habits and stress reduction techniques.

The genial tubercles are best visualized on which type of radiograph? A. Occlusal. B. Periapical. C. Bitewing. D. Panoramic.

A. Occlusal.

Hypercementosis may be associated with A. Paget's disease. B. ameloblastoma. C. hypophosphatasia. D. multiple myeloma.

A. Paget's disease.

Which disorder is associated with hypercementosis of teeth? A. Paget's disease. B. Fibrous dysplasia. C. Cherubism. D. Hyperparathyroidism.

A. Paget's disease.

Which of the following is NOT a symptom of hypercalcemia? A. Paresthesia. B. Constipation. C. Anoxia. D. Lethargy.

A. Paresthesia.

Which of the following lesions has a tendency to bleed easily? A. Pyogenic granuloma. B. Osteoma. C. Fibroma. D. Papilloma. E. Lipoma.

A. Pyogenic granuloma.

After the elimination of occlusal trauma, even in the presence of inflammation, which of the following is most likely to result? A. Reduction in tooth mobility. B. Regeneration of the periodontal ligament. C. Restoration of lost alveolar bone. D. Gain of clinical attachment.

A. Reduction in tooth mobility.

Chewing "automatism" is dependent on which mechanism? A. Reflexes between jaw closing and opening muscles. B. Swallowing and respiration neuronal activity. C. Periodontal receptor stimulation. D. Reticular formation neuronal activity.

A. Reflexes between jaw closing and opening muscles.

The most appropriate time to select a shade for a composite resin restoration is A. before administering local anesthetic. B. when the dental operatory light is illuminating the entire arch. C. after rubber dam placement. D. immediately before placement of the composite resin

A. before administering local anesthetic.

Patients who have undergone kidney transplantation are at an increased risk of developing A. brown tumours. B. plasma cell gingivitis. C. erosive lichen planus. D. squamous cell carcinoma.

A. brown tumours.

When using forceps to extract a maxillary first molar, the forceps movement should be principally in the buccal direction because the A. buccal bone is thinner than the palatal bone. B. buccal roots are shorter than palatal root. C. risk for sinus perforation is minimized. D. furcation is more accessible from the buccal.

A. buccal bone is thinner than the palatal bone.

Lichen planus occurs most frequently on the A. buccal mucosa. B. tongue. C. floor of the mouth. D. gingiva.

A. buccal mucosa.

If the lining cement is left on the gingival cavosurface margin of a Class II amalgam restoration, A. cement dissolution will lead to leakage. B. the preparation will lack retention form. C. the preparation will lack resistance form to bulk fracture. D. the preparation will lack appropriate outline form.

A. cement dissolution will lead to leakage.

Following successful root canal therapy, the most desirable apical tissue response would be A. cementum deposition into the apical foramen. B. formation of a connective tissue capsule over the foramen. C. proliferation of epithelium from the apical periodontal ligament. D. a chronic low grade inflammatory response surrounded by normal bone.

A. cementum deposition into the apical foramen.

Gingival connective tissue fibres are primarily composed of A. collagen. B. reticulin. C. elastin. D. oxytalin.

A. collagen.

After partial pulpotomy of a permanent central incisor in an 8 year old child, the most important clinical criterion of success is A. completion of root formation. B. retained natural colour of the tooth. C. pulp regeneration. D. formation of pulp stones.

A. completion of root formation.

The term "subdivision" in malocclusion classification refers to molar relationships that are A. different on the right and left side. B. the same on the right and left side. C. mildly abnormal. D. severely abnormal.

A. different on the right and left side.

When performing a frenectomy, a minimal amount of anesthetic solution is used to prevent A. distortion of the tissues. B. sloughing. C. secondary bleeding. D. irritation.

A. distortion of the tissues.

A 6 year old patient has an intrusive injury to tooth 5.2. All of the following are possible sequelae to the permanent successor EXCEPT A. enamel hypoplasia. B. root dilaceration. C. delayed eruption. D. ectopic eruption.

A. enamel hypoplasia.

A patient has significant pain and a fluctuant swelling of the left cheek. Clinical and radiographic examinations confirm a severe odontogenic infection associated with a carious nonrestorable tooth 2.3. The most appropriate initial management is to A. establish drainage. B. recommend a hot compress. C. prescribe an antibiotic. D. prescribe analgesics.

A. establish drainage.

A lateral force applied to the crown of a tooth will result in the crown moving A. in one direction and the root apex in the opposite direction. B. in the same direction as the root apex. C. along the line of force while the root apex remains stationary.

A. in one direction and the root apex in the opposite direction.

Acute anaphylactic reactions to penicillin are LEAST likely to occur A. in patients with a negative skin test to penicillin. B. within minutes after drug administration. C. in patients who have already experienced an allergic reaction to the drug. D. when the drug is administered parenterally.

A. in patients with a negative skin test to penicillin.

The success of replantation of an avulsed tooth is dependent upon A. length of time between avulsion and replantation. B. completion of endodontic therapy before replantation. C. immersing the tooth in fluoride solution before replantation. D. using calcium hydroxide as a treatment root canal filling.

A. length of time between avulsion and replantation.

Deposition of plaque on teeth occurs in A. less than 24 hours. B. 24 to 48 hours. C. 2 to 4 days. D. 5 to 7 days.

A. less than 24 hours.

The highest incidence of congenitally missing lateral incisors is most likely seen in a patient with A. unilateral cleft lip and palate. B. congenital heart disease. C. Down's syndrome. D. hyperthyroidism.

A. unilateral cleft lip and palate.

Excessive flare of the distobuccal cavosurface margin of a Class II amalgam cavity preparation will result in A. unsupported enamel at the margin. B. weak amalgam at the margin. C. poor retention. D. poor esthetics

A. unsupported enamel at the margin.

Hydroxyapatite used in dental surgery is A. used to fill osseous defects. B. derived from tooth enamel. C. resistant to fracture. D. osteogenic.

A. used to fill osseous defects.

On bite-wing radiographs of adults under the age of 30, the normal alveolar crest is A. at the cementoenamel junction. B. 1-2mm apical to the cementoenamel junction. C. 3-4mm apical to the cementoenamel junction. D. not clearly distinguishable

B. 1-2mm apical to the cementoenamel junction.

On bite-wing radiographs of adults under the age of 30, the normal alveolar crest is A. at the cementoenamel junction. B. 1-2mm apical to the cementoenamel junction. C. 3-4mm apical to the cementoenamel junction. D. not clearly distinguishable.

B. 1-2mm apical to the cementoenamel junction.

Roots of the permanent maxillary central incisors are completed by what age? A. 8 years. B. 10 years. C. 12 years. D. Later than 12 years.

B. 10 years.

Exclusive of third molars, the earliest that permanent dentition is usually completely erupted by the age of A. 9 to 11 years. B. 12 to 14 years. C. 15 to 17 years. D. 18 to 21 years.

B. 12 to 14 years.

Orthodontic correction of a maxillary midline diastema is most appropriate in a/an A. 8 year old patient without a thumb sucking habit. B. 14 year old patient without a thumb sucking habit. C. 8 year old patient with a thumb sucking habit. D. 14 year old patient with a thumb sucking habit

B. 14 year old patient without a thumb sucking habit.

When will infiltrating new blood vessels be histologically detectable following a free gingival graft? A. 2 to 3 hours. B. 2 to 3 days. C. 2 to 3 weeks. D. 2 to 3 months.

B. 2 to 3 days.

During CPR, if a patient is not breathing but has a pulse, how often should a breath be given? Every A. 3-4 seconds. B. 5-6 seconds. C. 7-8 seconds. D. 9-10 seconds.

B. 5-6 seconds.

Which of the following modifications to the standard procedure for mixing gypsum products will increase the compressive strength of the set material? A. Adding a small amount of salt to the water before mixing. B. Decreasing the water/powder ratio by a small amount. C. Using warmer water. D. Decreasing the mixing time. The

B. Decreasing the water/powder ratio by a small amount.

Which of the following is most likely to displace the adjacent teeth? A. Lateral periodontal cyst. B. Dentigerous cyst. C. Periapical osseous dysplasia (periapical cemento-osseous dysplasia). D. Apical abscess (periradicular abscess). E. Radicular cyst.

B. Dentigerous cyst.

Which of the following lesions is always associated with an impacted tooth? A. Ameloblastoma. B. Dentigerous cyst. C. Cementoblastoma. D. Keratocystic odontogenic tumour (odontogenic keratocyst). E. Adenomatoid odontogenic tumour.

B. Dentigerous cyst.

An ameloblastoma can develop from the epithelial lining of which of the following cysts? A. Periradicular. B. Dentigerous. C. Residual. D. Lateral periodontal.

B. Dentigerous.

In a sagittal split osteotomy of the mandible, the neurovascular bundle should remain in which segment of the mandible? A. Proximal. B. Distal. C. Lateral. D. Condylar.

B. Distal.

Which radiographic finding supports the diagnosis of labial displacement of the root of an intruded primary incisor? A. Elongation. B. Foreshortening. C. Decreased opacity. D. Widened periodontal ligament space.

B. Foreshortening.

Which of the following is the greatest risk factor for rampant caries in children? A. Frequent ingestion of polysaccharides. B. Frequent ingestion of high sucrose-containing foods. C. Severe enamel hypoplasia. D. Deficiency of vitamin D.

B. Frequent ingestion of high sucrose-containing foods.

Which of the following is NOT a sign of occlusal trauma? A. Fremitus. B. Gingival recession. C. Widened periodontal ligament. D. Tooth migration.

B. Gingival recession.

A 50 year old obese patient was diagnosed with type 2 diabetes last year and has recently started taking an oral hypoglycemic. He frequently skips meals in order to reduce his weight. During his 8:30 a.m. appointment, his speech becomes slurred and he is less alert than usual. Which of the following is the most appropriate management? A. Have him drink 175ml of diet cola. B. Give him 15g of glucose as tablets or in a solution. C. Have him eat a chocolate bar. D. Dismiss the patient and advise him to eat.

B. Give him 15g of glucose as tablets or in a solution.

Which of the following presents with high serum calcium levels, thinning of cortical bone and giant cell osteoclasts in the jaw and drifting teeth? A. Hyperthyroidism. B. Hyperparathyroidism. C. Hypothyroidism. D. Hypoparathyroidism.

B. Hyperparathyroidism.

Which of the following diseases predisposes a patient to exaggerated cardiovascular effects of epinephrine? A. Hyperparathyroidism. B. Hyperthyroidism. C. Hypogonadism. D. Acromegaly.

B. Hyperthyroidism.

Which of the following is the most potent analgesic? A. Acetylsalicylic acid 650mg. B. Ibuprofen 600mg. C. Acetaminophen 300mg with codeine 30mg. D. Codeine 60mg.

B. Ibuprofen 600mg.

Which of the following compounds released by inflammatory cells induces bone resorption? A. Nitric oxide. B. Interleukin-1. C. Bradykinin. D. Alkaline phosphatase.

B. Interleukin-1.

Which of the following muscles is a depressor of the mandible? A. Temporalis. B. Lateral pterygoid. C. Masseter. D. Medial pterygoid.

B. Lateral pterygoid.

Which of the following structures lies inferior to the mylohyoid muscle at the level of the mandibular second molar? A. Lingual artery. B. Lingual vein. C. Lingual nerve. D. Submandibular duct.

B. Lingual vein.

The condyle of the mandible is unique because A. it develops from Meckel's cartilage. B. both interstitial and appositional bone formation are present. C. both primary and secondary cartilage growth centres are present.

B. both interstitial and appositional bone formation are present.

The pulpal floor of an occlusal amalgam preparation on a mandibular first premolar should slope apically from A. mesial to distal. B. buccal to lingual. C. distal to mesial. D. lingual to buccal.

B. buccal to lingual.

Fluorides are effective in the prevention of dental caries by A. increasing the resistance of dentin to bacterial penetration. B. causing tooth enamel to be more resistant to demineralization. C. providing a more favorable pulpal blood supply. D. All of the above.

B. causing tooth enamel to be more resistant to demineralization.

Following periodontal surgery, the curetted root surface is repopulated by cells derived from all of the following tissues EXCEPT A. periodontal ligament. B. cementum. C. alveolar bone. D. epithelium. E. gingival connective tissue

B. cementum.

Overadjustment of a wrought wire denture clasp can lead to fracture because of a/an A. increase in modulus of elasticity. B. decrease in ductility due to strain hardening. C. decrease in the yield strength. D. increase in fracture toughness.

B. decrease in ductility due to strain hardening.

Permanent incisor eruption A. is unaffected by the timing of primary incisor loss. B. is initiated apical and lingual to the primary incisors. C. is unaffected by the gender of the patient. D. results in a reduction of arch length.

B. is initiated apical and lingual to the primary incisors.

Using less water for mixing plaster of Paris will result in set plaster that A. contracts. B. is stronger. C. is more porous. D. is less brittle.

B. is stronger.

A major difference between stable angina pectoris and myocardial infarction is that stable angina pectoris does NOT involve A. crushing substernal pain. B. ischemic myocardial necrosis. C. occlusive coronary artery disease. D. atherosclerosis. E. females under the age of 80 years.

B. ischemic myocardial necrosis.

When probing a healthy peridontium using light forces, the probe tip will most likely extend to the A. attached gingiva. B. junctional epithelium. C. transverse fibres of the periodontal ligament.

B. junctional epithelium.

Isolated gingival recession is most frequently seen on teeth that are A. heavily restored. B. labially prominent. C. mobile. D. nonvital.

B. labially prominent.

The most common problem associated with two adjacent implants is A. peri-implantitis. B. lack of interdental papilla. C. granulation tissue.

B. lack of interdental papilla.

In the pterygomandibular space, the inferior alveolar nerve passes A. anterior to the deep tendon of the temporal muscle. B. lateral to the sphenomandibular ligament. C. medial to the medial pterygoid muscle. D. medial to the pterygomandibular raphe.

B. lateral to the sphenomandibular ligament.

The presence of flaws or cracks in a material A. influences the strength of metals more than ceramics. B. leads to the development of stress concentrations. C. impacts compressive strength more than tensile strength. D. decreases its elastic modulus.

B. leads to the development of stress concentrations.

During extraction of a maxillary third molar, the tuberosity is fractured. The tooth with the tuberosity remains attached to the surrounding soft tissue. You should A. remove both and suture. B. leave both and stabilize, if possible. C. remove both, fill the defect with Gelfoam and suture. D. reflect the mucoperiosteum, remove the tooth, leaving the tuberosity in place and suture.

B. leave both and stabilize, if possible.

The most effective way of minimizing a patient's radiation dose is A. a lead apron and thyroid collar. B. prescription radiography. C. fast emulsion film. D. intensifying screens.

B. prescription radiography.

Treatment of a S. aureus infection with penicillin is often complicated by the A. inability of penicillin to penetrate the membrane of S. aureus. B. production of penicillinase by S. aureus. C. secretion of penicillin acetylase by S. aureus. D. allergic reaction caused by staphylococcal protein.

B. production of penicillinase by S. aureus.

Which of the following characteristics is NOT seen in all patients with aggressive periodontitis? A. Rapid attachment loss and bone destruction. B. Amount of microbial deposits inconsistent with disease severity. C. Diseased sites infected with Aggregatibacter (Actinobacillus) actinomycetemcomitans. D. Familial aggregation of diseased individuals.

C. Diseased sites infected with Aggregatibacter (Actinobacillus) actinomycetemcomitans.

In a dental office, what is the most common cause of respiratory distress? A. Anaphylaxis. B. Bronchospasm. C. Hyperventilation. D. Myocardial infarction.

C. Hyperventilation.

Which of the following constituents of a local anesthetic cartridge is most likely to be allergenic? A. Lidocaine. B. Epinephrine. C. Metabisulfite. D. Hydrochloric acid.

C. Metabisulfite.

Which of the following conditions would NOT require antibiotic premedication before endodontic therapy? A. Valvular heart disease. B. Cardiac prosthesis. C. Persistent odontogenic fistula. D. Immunosuppressive therapy. E. Organ transplant.

C. Persistent odontogenic fistula.

Which element found in radiograph processing solutions is of most concern environmentally? A. Mercury. B. Lead. C. Silver. D. Copper.

C. Silver.

Saliva is most effective in minimizing an acid challenge by its A. lubrication function. B. antimicrobial effect. C. buffering action. D. fluoride concentration.

C. buffering action.

Opioids would be CONTRAINDICATED for the management of A. pain. B. severe cough. C. diarrhea. D. depression.

C. diarrhea.

A clenching habit may be a factor in A. suprabony periodontal pocket formation. B. marginal gingivitis. C. increased tooth mobility. D. generalized recession.

C. increased tooth mobility.

The depth of penetration of X rays depends on the A. milliamperage. B. temperature of the filament. C. exposure time. D. kilovoltage.

D. kilovoltage.

In primary molars, the cusp with the largest pulp horn is the A. distolingual. B. distobuccal. C. mesiolingual. D. mesiobuccal.

D. mesiobuccal.

Latex gloves should A. be washed with plain soap before initial use. B. be washed with plain soap when used between patients. C. be washed with a disinfectant solution only. D. not be washed.

D. not be washed.

Patients with primary herpetic gingivostomatitis should NOT receive A. acetaminophen. B. acyclovir. C. benzocaine. D. prednisone. E. chlorhexidine.

D. prednisone.

The permanent teeth most frequently ankylosed are the A. canines. B. incisors. C. molars. D. premolars.

D. premolars.

Local anesthetics interfere with the transport of which of the following ions? A. Sodium. B. Calcium. C. Chloride. D. Potassium. E. Magnesium.

A. Sodium.

When tumour cells revert to a more primitive, embryonic or undifferentiated form with an increased capacity for reproduction and a decreased function, this is called A. anaplasia. B. metaplasia. C. hypoplasia. D. hyperplasia.

A. anaplasia.

The primary etiological factor associated with periodontal disease is A. bacterial plaque. B. calculus. C. diabetes mellitus. D. tooth mobility. E. smoking.

A. bacterial plaque.

Dental plaque A. contains insoluble glucans. B. contains food particles. C. contains predominantly Gram-negative organisms. D. becomes less anaerobic as it matures.

A. contains insoluble glucans.

The small bubble normally seen in a local anesthetic cartridge is A. nitrogen. B. air. C. oxygen. D. a breakdown product.

A. nitrogen.

Soft tissue pockets CANNOT be reduced by A. occlusal adjustment. B. scaling and root planing (debridement). C. open flap curettage. D. guided tissue regeneration

A. occlusal adjustment.

The electric pulp tester is the most suitable test in cases of teeth with A. open apices. B. obliterated canals. C. full cuspal coverage. D. concussion injury.

A. open apices.

The management of a single tooth posterior crossbite requires A. opening the bite to disclude the teeth. B. creating space in the opposing arch. C. correcting the functional shift of the mandible. D. rapid palatal expansion.

A. opening the bite to disclude the teeth.

The prolonged use of antibacterial lozenges or mouthwashes contributes to the development of A. oral candidiasis. B. geographic tongue. C. cancrum oris. D. Koplik's spots. E. aphthous ulcers.

A. oral candidiasis.

The periodontal probe should be inserted into the gingival sulcus A. parallel to the long axis of the tooth. B. parallel to the root surface. C. with a firm pushing motion. D. with a firm lateral motion

A. parallel to the long axis of the tooth.

When designing a removable partial denture, changing the tilt of the cast on the surveyor alters the A. path of insertion of the planned removable partial denture. B. the position of the survey line on the cast. C. the undercut and non-undercut areas. D. the direction of forces applied to the partial denture.

A. path of insertion of the planned removable partial denture.

In an infection caused by non-penicillinase producing staphylococcus, the drug of choice is A. penicillin V. B. cephalexin. C. tetracycline. D. vancomycin.

A. penicillin V.

In complete denture contruction, the physiologic rest position A. provides a quide to establish the vertical dimension of occlusion. B. determines the level of the occlusal plane. C. provides a guide for the selection of cusp inclination. D. determines the shape of the compensating curve. E. is useful to the determination of condylar inclination.

A. provides a quide to establish the vertical dimension of occlusion.

Anginal pain is A. provoked by exercise or cold weather. B. not relieved by rest. C. relieved by digoxin. D. aggravated by deep inspiration.

A. provoked by exercise or cold weather.

Many months after trauma to a primary incisor, the development of a greyish discolouration in the crown usually indicates A. pulp necrosis. B. pulp canal calcification. C. external root resorption. D. internal root resorption.

A. pulp necrosis.

The radiographic appearance of internal resorption is A. radiolucent enlargement of the pulp cavity. B. radiolucency around the apex of the root. C. radiolucency on the surfaces of the root. D. localized radiopacities in the pulp cavity. E. radiopacity around the apex of the root.

A. radiolucent enlargement of the pulp cavity.

In the Vita® Classical Shade Guide, the hue of the D shade series is A. red. B. grey. C. yellow. D. brown.

A. red.

An antipyretic drug A. reduces fever. B. provides analgesia. C. causes loss of consciousness. D. creates heat sensitivity. E. counters the tendency for epileptic seizures

A. reduces fever.

The most appropriate management of an avulsed maxillary central incisor in a 10 year old is the placement of a A. removable space maintainer. B. bonded resin bridge (Maryland bridge). C. conventional fixed partial denture. D. single tooth implant.

A. removable space maintainer.

alveolar nerve is a possible complication of A. removal of an impacted mandibular third molar tooth. B. removal of a torus mandibularis. C. a forceps removal of a mandibular second molar. D. distal wedge periodontal surgery.

A. removal of an impacted mandibular third molar tooth.

The inorganic ion that is implicated in primary hypertension is A. sodium. B. fluoride. C. potassium. D. magnesium.

A. sodium.

Metastasis is most likely to occur in A. squamous cell carcinoma. B. basal cell carcinoma. C. ameloblastoma. D. complex odontoma. E. odontogenic fibroma.

A. squamous cell carcinoma.

For sterilization to occur in an autoclave, the packaged instruments are subjected to pressurized A. steam. B. chemical vapour. C. boiling water. D. heated air.

A. steam.

Which of the following analyses is most appropriate for use in an adult patient? A. Moyers. B. Bolton. C. Tanaka-Johnston.

B. Bolton.

What is the most common site for intraoral squamous cell carcinoma? A. Gingiva. B. Floor of mouth. C. Buccal mucosa. D. Dorsum of tongue.

B. Floor of mouth.

Which virus is the most likely to cause an infection in a healthcare worker following exposure to blood from an individual infected with the virus? A. Hepatitis A. B. Hepatitis B. C. Hepatitis C. D. Human immunodeficiency virus

B. Hepatitis B.

Which drug is indicated for the management of an acute asthmatic attack A. Fluticasone. B. Salbutamol. C. Triamcinolone. D. Budesonide.

B. Salbutamol.

Polyvinylsiloxane impression materials have high A. polymerization shrinkage. B. dimensional stability. C. by-product formation. D. linear expansion.

B. dimensional stability.

The most common complication of a venipuncture is A. syncope. B. hematoma. C. thrombophlebitis. D. embolus.

B. hematoma.

A lateral cephalometric radiograph for a patient with a 3mm anterior functional shift should be taken with the patient in A. maximum intercuspation. B. initial contact. C. normal rest position. D. maximum opening. E. protrusive position.

B. initial contact.

The term applied to a low white blood cell count is A. leukocytosis. B. leukopenia. C. thrombocythemia. D. thrombocytopenia.

B. leukopenia.

A measure of the stiffness of a dental gold alloy is expressed as its A. proportional limit. B. modulus of elasticity. C. ultimate tensile strength. D. flow.

B. modulus of elasticity.

The dentino-enamel junction is the most sensitive portion of a tooth because A. free nerve endings terminate on odontoblasts at this region. B. odontoblastic processes branch considerably at this region. C. ameloblasts make synaptic connections with odontoblasts at this junction. D. odontoblastic tubules help convey hydrostatic forces to the pulp cells.

B. odontoblastic processes branch considerably at this region.

In teeth with pulp necrosis, the periapical area is involved if there is pain A. to thermal stimuli. B. on percussion. C. to electric pulp testing. D. when the patient is lying down.

B. on percussion.

After initial setting, a chemically cured glass ionomer cement restoration should have a coating agent applied to A. hasten the final set. B. protect the cement from moisture. C. retard the final set. D. protect the cement from ultraviolet light. E. create a smooth finish.

B. protect the cement from moisture.

The primary reason for placing a surgical dressing after a gingivectomy is to A. prevent hemorrhage. B. protect the wound. C. stabilize the teeth. D. protect the sutures.

B. protect the wound.

The major disadvantage of zinc phosphate used to cement crowns is the A. development of heat during setting. B. pulp irritation. C. lack of edge strength. D. low crushing strength.

B. pulp irritation.

A 4 year old has a primary central incisor that is yellow but asymptomatic. The most probable diagnosis is A. pulpal necrosis. B. pulpal calcification. C. internal resorption. D. external resorption.

B. pulpal calcification.

The radiographic images most suggestive of multiple myeloma is A. multiple radiopaque lesions. B. punched out radiolucent lesions. C. ground glass appearance. D. generalized hypercementosis.

B. punched out radiolucent lesions.

For a patient with dementia, aphasia is characterized by a diminished ability to A. execute personal oral hygiene tasks. B. recognize people. C. make decisions. D. appreciate the consequences of behaviour. E. express ideas or thoughts.

B. recognize people.

A pleomorphic adenoma is characterized by A. metastases. B. recurrence. C. pain. D. ulceration.

B. recurrence.

Upon examination of an edentulous patient, it is observed that the tuberosities contact the retromolar pads at the correct occlusal vertical dimension. The treatment of choice is to A. reduce the retromolar pads surgically to provide the necessary clearance. B. reduce the tuberosities surgically to provide the necessary clearance. C. construct new dentures at an increased occlusal vertical dimension to gain the necessary clearance. D. proceed with construction of the denture and reduce the posterior extension of the mandibular denture to eliminate interferences.

B. reduce the tuberosities surgically to provide the necessary clearance.

The placement of a reverse curve in a Class II amalgam preparation aids in A. retention form. B. resistance form. C. convenience form. D. outline form.

B. resistance form.

The hydrophilicity of an impression material is defined by its A. high water absorption. B. small contact angle. C. osmotic property. D. water content.

B. small contact angle.

A 22 year old presents with a fracture of the incisal third of tooth 2.1 exposing a small amount of dentin. The fracture occurred one hour previously. There is no mobility of the tooth but the patient complains that it is rough and sensitive to cold. The most appropriate emergency treatment is to A. open the pulp chamber, clean the canal and temporarily close with zinc oxide and eugenol. B. smooth the surrounding enamel and apply glass ionomer cement. C. smooth the surrounding enamel and apply a calcium hydroxide cement. D. place a provisional (temporary) crown.

B. smooth the surrounding enamel and apply glass ionomer cement.

World epidemiological data indicates that periodontal disease is the most likely cause of tooth loss in the following age group: A. 10 - 20 years. B. 20 - 30 years. C. 30 - 50 years. D. 65 - 75 years. E. over 75 years.

C. 30 - 50 years.

What is the earliest age to confirm the diagnosis of a congenitally missing mandibular second premolar? A. 1 year. B. 3 years. C. 5 years. D. 7 years.

C. 5 years.

In comparing D and F speed intraoral radiographic films, F speed film emulsion is approximately A. 20% more sensitive than D speed film. B. 40% more sensitive than D speed film. C. 60% more sensitive than D speed film. D. 80% more sensitive than D speed film.

C. 60% more sensitive than D speed film.

In the treatment of necrotizing ulcerative gingivitis with associated lymphadenopathy, which of the following medications is the most appropriate? A. An anti-inflammatory. B. A topical antibiotic. C. A systemic antibiotic. D. An analgesic.

C. A systemic antibiotic.

Which of the following drugs is used in the treatment of mild allergic reactions? A. Isoproterenol. B. Meperidine hydrochloride. C. Diphenhydramine hydrochloride. D. Propoxyphene.

C. Diphenhydramine hydrochloride.

Which anatomical structures form the inverted Y (Y line) in maxillary periapical radiographs? A. Nasopalatine/incisive canal and floor of the nasal fossa. B. Anterior nasal spine and nasopalatine/incisive canal. C. Floor of the nasal fossa and maxillary sinus border. D. Zygomatic process of the maxilla and maxillary sinus border

C. Floor of the nasal fossa and maxillary sinus border.

Which of the following has the largest effect on caries risk? A. Type of desserts eaten with meals. B. Total amount of sugar in the diet. C. Frequency of fermentable carbohydrate intake. D. Quality and range of nutrients in meals and snacks.

C. Frequency of fermentable carbohydrate intake.

Which of the following cements can chemically bond to enamel? A. Reinforced zinc oxide eugenol cement. B. Zinc phosphate cement. C. Glass ionomer cement.

C. Glass ionomer cement.

In long-standing gingivitis, the subgingival microflora shifts toward A. aerobic bacteria. B. Gram-positive bacteria. C. Gram-negative anaerobic bacteria. D. None of the above.

C. Gram-negative anaerobic bacteria.

Which of the following is a CONTRAINDICATION for placement of a fissure sealant on a permanent molar? A. Tooth erupted more than one year. B. Deep, narrow fissures. C. Inadequate moisture control. D. Community water supply fluoridated at 1.0ppm.

C. Inadequate moisture control.

Temporary anchorage devices (TADS) have the greatest effect on which orthodontic movement? A. Extrusion. B. Torqueing. C. Intrusion. D. Tipping.

C. Intrusion.

The surgical removal of tooth 3.6 requires anesthesia of the inferior alveolar nerve as well as which of the following nerves? A. Lingual, cervical plexus. B. Cervical, long buccal. C. Lingual, long buccal. D. Mental, long buccal.

C. Lingual, long buccal.

Which of the following will increase resistance to dislodging forces on a removable partial denture? A. Locating direct and indirect retainers as close as possible to the distal extension base(s). B. Locating direct and indirect retainers as far as possible from the distal extension base(s). C. Locating direct retainers as close as possible to the distal extension base(s) and the indirect retainers as far as possible from the distal extension base(s). D. Locating direct retainers as far as possible from the distal extension base(s) and the indirect retainers as close as possible to the distal extension base(s).

C. Locating direct retainers as close as possible to the distal extension base(s) and the indirect retainers as far as possible from the distal extension base(s).

Which of the following is the most appropriate management for a dry socket? A. Hydrogen peroxide irrigation of the socket. B. Vigorous curettage of the socket. C. Placement of a dressing in the socket. D. A prescription for antibiotics.

C. Placement of a dressing in the socket.

A patient with pain, fever and unilateral parotid swelling following a general anesthetic most likely has A. Mumps. B. sialolithiasis. C. acute bacterial sialadenitis. D. Sjögren's syndrome. E. sarcoidosis.

C. acute bacterial sialadenitis.

Flurbiprofen is an A. antibiotic. B. muscular relaxant. C. anti-inflammatory. D. antidepressant.

C. anti-inflammatory.

Accessory canals in permanent teeth are most commonly found in the A. cervical third of the root. B. middle third of the root. C. apical third of the root. D. bifurcation area. E. trifurcation area.

C. apical third of the root.

Enlargement of the gingiva, described as idiopathic fibromatosis, is best described as A. degeneration. B. inflammation. C. hyperplasia. D. neoplasia.

C. hyperplasia.

Loss of attachment in periodontal disease occurs A. at a consistent rate. B. in cycles lasting for about 3 months. C. in random cycles.

C. in random cycles.

An ankylosed tooth is usually A. nonvital. B. associated with a root fracture. C. infraerupted. D. found in the permanent dentition.

C. infraerupted.

The mesial furcation of maxillary permanent first molars is best probed from the A. buccal. B. buccal or lingual. C. lingual.

C. lingual.

The maxillary central incisors of a 2 year old child have been traumatically intruded 4mm. The most appropriate immediate management is to A. carefully remove both incisors. B. reposition the intruded teeth. C. make the patient comfortable without disturbing the teeth. D. reposition and splint the intruded teeth. E. order an occlusal radiograph.

C. make the patient comfortable without disturbing the teeth.

Mean x-ray beam energy is a function of A. exposure time. B. tube current. C. tube voltage. D. collimation.

C. tube voltage.

Gingival inflammation may result from A. trauma. B. chemical irritation. C. plaque. D. All of the above.

D. All of the above.

In the surgical removal of an impacted mandibular third molar, which of the following would be considered to be the most difficult? A. Mesio-angular. B. Horizontal. C. Vertical. D. Disto-angular.

D. Disto-angular.

Which of the following epithelial changes is most likely to be precancerous? A. Acanthosis. B. Hyperkeratosis. C. Parakeratosis. D. Dysplasia.

D. Dysplasia.

Following the IV administration of a benzodiazepine, a patient becomes apneic and unresponsive. Which of the following drugs should be administered immediately? A. Naloxone. B. Propranolol. C. Amphetamine. D. Flumazenil. E. Atropine.

D. Flumazenil.

Which of the following would differentiate clinically between an acute apical abscess (acute periradicular abscess) and an acute periodontal abscess on a single rooted tooth? A. Pain upon palpation. B. Tooth mobility. C. Pain upon percussion. D. Pulp vitality testing.

D. Pulp vitality testing.

The stratified squamous epithelial layer of oral mucosa does NOT contain A. melanocytes. B. Langerhans cells. C. Merkel cells. D. Schwann cells.

D. Schwann cells.

Which of the following is the most effective method of cleaning endodontic instruments prior to sterilization? A. Manual brushing. B. High pressure water jet. C. Washing with antimicrobial soap. D. Washing in an ultrasonic bath for 5 minutes.

D. Washing in an ultrasonic bath for 5 minutes.

All of the following drugs are useful in the treatment of cardiac arrhythmias EXCEPT A. digitalis. B. lidocaine. C. procainamide. D. aminophylline.

D. aminophylline.

Unconsciousness in syncope results from A. electrolyte imbalance. B. neurogenic shock. C. cerebral hyperemia. D. cerebral hypoxia.

D. cerebral hypoxia.

DMF-S is an index for expressing A. dental needs. B. tooth mortality. C. extent of dental neglect. D. dental caries.

D. dental caries.

The most radiosensitive cell type is a/an A. neuron. B. chondrocyte. C. myocyte. D. epithelial basal cell.

D. epithelial basal cell.

The primary function of root canal sealer is to A. prevent discolouration of the tooth. B. stimulate healing in the apical region. C. medicate the canal to eliminate remaining bacteria. D. fill space between gutta-percha and pulp canal wall.

D. fill space between gutta-percha and pulp canal wall.

The redness of inflamed gingiva is due to A. the degree of keratinization. B. subgingival deposits. C. increased collagen fibre density. D. increased vasodilation

D. increased vasodilation

A 15 year old presents with hypoplastic enamel on tooth 1.5. All other teeth are normal. This was most probably caused by a/an A. vitamin D deficiency. B. generalized calcium deficiency. C. high fever encountered by the patient when he had measles at age 3. D. infection of tooth 5.5 during the development of tooth 1.5. E. hereditary factor.

D. infection of tooth 5.5 during the development of tooth 1.5.

The use of conscious sedation for a restorative procedure in an office environment A. requires the services of an anesthesiologist. B. is contraindicated in children. C. precludes the use of local anesthesia. D. requires effective local anesthesia. E. may be used in conjunction with narcotics.

D. requires effective local anesthesia.

The collimator of an X ray tube A. produces a more homogeneous X ray beam. B. prevents secondary radiation. C. focuses the X ray beam. D. restricts the diameter of the X ray beam.

D. restricts the diameter of the X ray beam.

Administration of which of the following is most likely to predispose a patient to seizures? A. Acetaminophen. B. Codeine. C. Ibuprofen. D. Ketorolac. E. Meperidine

E. Meperidine

Which of the following drugs is a natural alkaloid obtained from opium? A. Hydromorphone. B. Meperidine. C. Methadone. D. Codeine. E. Morphine

E. Morphine

What is the most appropriate radiographic survey for a 3 year old patient with generalized open contacts and occlusal caries present? A. Two bite-wing radiographs. B. Two bite-wing radiographs and a panoramic radiograph. C. Four posterior periapical radiographs. D. Two anterior occlusal radiographs and 2 bitewing radiographs. E. No radiographs indicated.

E. No radiographs indicated.

Which of the following landmarks is used to determine the height of the occlusal plane of the posterior teeth in a mandibular complete denture? A. Maxillary central incisor. B. Residual ridge crest. C. Tongue. D. Mandibular canine. E. Retromolar pad.

E. Retromolar pad.

A rubber dam should be used in A. pulp capping procedures. B. amalgam placement. C. composite placement. D. removing carious dentin from deep lesions. E. all of the above.

E. all of the above.

Which of the following is/are NOT usually affected by hereditary ectodermal dysplasia? A. Salivary glands. B. Teeth. C. Sweat glands. D. Hair. E. Fingernails.

A. Salivary glands.

In normal gingiva, the predominant microflora of gingival plaque are A. gram-positive cocci. B. gram-negative cocci. C. gram-negative facultative and anaerobic rods. D. spirochetes.

A. gram-positive cocci.

In a standard dental cartridge (carpule) containing 1.8ml 2% lidocaine with epinephrine 1/100,000, the amount of vasoconstrictor is A. 18.0 mg. B. 0.018 mg. C. 1.8 mg. D. 0.18 mg. E. 180.0 mg.

B. 0.018 mg.

In clinically normal periodontium, the distance between the bottom of the gingival sulcus and the alveolar crest is A. 0.5-1mm. B. 1.5-2mm. C. 3-5mm.

B. 1.5-2mm.

Which of the following teeth is LEAST likely to develop a furcation involvement? A. 1.4. B. 1.5. C. 1.6. D. 1.7.

B. 1.5.

An overjet of 8mm is most often associated with which molar realationship? A. Class I. B. Class II. C. Class III.

B. Class II.

Which of the following drugs may cause generalized enlargement of interdental papillae? A. Digitalis. B. Nifedipine. C. Captopril. D. Propranolol.

B. Nifedipine.

Which of the following is a clinical CONTRAINDICATION for an all-ceramic maxillary anterior crown? A. Excessive overjet and overbite. B. Normal overjet and excessive overbite. C. An endodontically treated tooth with a cast post and core. D. A low caries index.

B. Normal overjet and excessive overbite.

Which of the following statements about the nutritional management of diabetes is correct? A. A diet planned according to Canada's Food Guide to Healthy Eating must be modified for a person with diabetes. B. The Glycemic Index of foods should be used when planning the diet. C. The fat content of the diet should be 30-35% of energy intake. D. Sucrose up to 10% of total daily energy intake (e.g. 50% of 2000 kcal/day) is acceptable.

B. The Glycemic Index of foods should be used when planning the diet.

The physiologic wear of hard dental tissue resulting from mastication is known as A. decalcification. B. attrition. C. abrasion. D. erosion.

B. attrition.

Marginal leakage of a composite resin restoration will A. not be detectable. B. be minimized by use of an adhesive resin. C. decrease with longevity.

B. be minimized by use of an adhesive resin.

A single tooth anterior crossbite found in a 9 year old should A. self-correct. B. be treated with a removable appliance. C. have 2 arch orthodontic treatment. D. be treated in the complete permanent dentition. E. be observed and treated when the cuspids have erupted.

B. be treated with a removable appliance.

All of the following display visceolastic properties EXCEPT A. irreversible hydrocolloids. B. dental porcelain. C. silver amalgam. D. dentin

B. dental porcelain.

A characteristic of the periodontium which allows safe temporary separation of the teeth is the A. nature of acellular cementum. B. elasticity of bone. C. modified continuous eruption of the teeth. D. passive eruption.

B. elasticity of bone.

The proximal surfaces of two adjacent teeth in contact form the borders of the A. interdental space. B. embrasures. C. interdental col. D. line angles.

B. embrasures.

A silane coupling agent is used to A. control polymerization shrinkage in composite resins. B. enhance the bond between a porcelain veneer and the resin cement. C. reduce the surface tension when investing a wax pattern. D. facilitate the soldering of gold castings.

B. enhance the bond between a porcelain veneer and the resin cement.

In the treatment of an acute anaphylactic reaction, the first drug that should be administered is A. hydroxyzine. B. epinephrine. C. hydrocortisone. D. diphenhydramine.

B. epinephrine.

Histologically, a pulp polyp consists of all of the following EXCEPT for A. a mass of collagenous fibres. B. epithelial rests of Malassez. C. proliferating capillaries. D. fibroblasts.

B. epithelial rests of Malassez.

By definition, a compound fracture of the mandible must have A. multiple bone fragments. B. exposure to the external environment. C. a tooth in the line of fracture. D. displacement of the fractured segments.

B. exposure to the external environment.

Cold working a metal increases its A. ductility. B. hardness. C. resistance to corrosion. D. strength

B. hardness.

In comparison to visible light, X rays A. have a longer wavelength. B. have higher energy. C. travel faster. D. can be focused.

B. have higher energy.

An 80 year old man develops multiple painful skin vesicles along the distribution of the right infraorbital nerve. The most likely diagnosis is A. psoriasis. B. herpes zoster. C. pemphigus vulgaris. D. candidiasis.

B. herpes zoster.

In dental radiography, the most effective method of reducing patient somatic exposure is to use A. a lead apron. B. high speed film. C. added filtration. D. collimation.

B. high speed film.

In an adult, progressive increase in mandibular length and interdental spacing is a feature of A. hyperparathyroidism. B. hyperpituitarism. C. hyperthyroidism. D. Addison's disease. E. Cushing's disease.

B. hyperpituitarism.

A 45 year old with insulin-dependent diabetes mellitus has a morning dental appointment. During the examination, the patient complains of being lightheaded and weak. Sweating is observed. The patient is most likely experiencing A. hyperglycemia. B. hypoglycemia. C. syncope. D. hyperventilation. E. cerebrovascular accident.

B. hypoglycemia.

Radiographic examination reveals early evidence of internal resorption. The most appropriate management is A. apical surgical intervention. B. immediate pulp extirpation. C. immediate pulpotomy. D. observation and re-evaluation in 3-6 months. E. calcium hydroxide pulpotomy.

B. immediate pulp extirpation.

The principal reason for a cavosurface bevel on an inlay preparation is to A. remove undermined enamel. B. improve marginal adaptation. C. decrease marginal percolation. D. increase resistance and retention forms.

B. improve marginal adaptation.

A smooth-surface proximal carious lesion A. initially begins as an enamel defect in the contact area. B. in enamel, can be depicted as a cone with the apex of the cone at the dentino-enamel junction. C. in enamel, can be depicted as a cone with the base of the cone at the dentino-enamel junction. D. initially begins as a subgingival enamel defect covered with plaque.

B. in enamel, can be depicted as a cone with the apex of the cone at the dentino-enamel junction.

The most likely cause of postoperative sensitivity following the placement of a posterior composite resin restoration with ideal occlusion is A. excessive etching of enamel margins. B. inadequate peripheral seal. C. excessive Primer resin. D. inadequate polymerization of primer resin.

B. inadequate peripheral seal.

Healthy attached gingiva A. has no basal cell layer. B. is closely bound to underlying periosteum. C. contains elastic fibers. D. has no rete pegs.

B. is closely bound to underlying periosteum.

Mandibular growth A. is sustained over a longer period of time in girls. B. is sustained over a longer period of time in boys. C. occurs at the same chronologic age in both sexes. D. occurs two years earlier in boys than in girls.

B. is sustained over a longer period of time in boys.

Periodontitis as a manifestation of systemic disease as described in the Armitage 1999 classification includes A. diabetes. B. leukemia. C. atherosclerosis. D. osteoporosis.

B. leukemia.

The tooth surfaces LEAST susceptible to caries are A. mesial of the maxillary arch. B. lingual of the mandibular arch. C. distal of the maxillary arch. D. occlusal of the mandibular arch. E. lingual of the maxillary arch.

B. lingual of the mandibular arch.

The penetration of a local anesthetic into nerve tissue is a function of the A. length of the central alkyl chain. B. lipid solubility of the un-ionized form. C. ester linkage between the aromatic nucleus and the alkyl chain. D. amide linkage between the aromatic nucleus and the alkyl chain.

B. lipid solubility of the un-ionized form.

A zirconia-based ceramic fixed partial denture can be used for a patient with A. periodontally involved abutment teeth. B. long clinical crowns. C. deep vertical anterior overlap. D. cantilever pontic. E. evidence of bruxism.

B. long clinical crowns.

Clinical diagnosis of periodontitis requires the presence of A. bleeding upon probing. B. loss of periodontal attachment. C. a periodontal pocket. D. tooth mobility.

B. loss of periodontal attachment.

Squamous cell carcinomas of the lip occur most frequently on the A. commissures. B. lower lip near the midline. C. inner surface of upper lip. D. inner surface of lower lip. E. upper lip near the midline

B. lower lip near the midline.

Mucoceles are most commonly found in the A. upper lip. B. lower lip. C. tongue. D. buccal mucosa. E. soft palate.

B. lower lip.

Mandibular overdentures are used to A. increase the strength of the denture. B. maintain the alveolar ridge morphology. C. improve periodontal health of abutment teeth. D. decrease costs.

B. maintain the alveolar ridge morphology.

The most common primary tooth to become ankylosed is a A. maxillary molar. B. mandibular molar. C. maxillary canine. D. mandibular canine.

B. mandibular molar.

Which of the following is the most acceptable extraction pattern in the management of an Angle Class II malocclusion in a non-growing patient? Extraction of A. mandibular first or second premolars. B. maxillary first or second premolars. C. mandibular incisors. D. maxillary first molars.

B. maxillary first or second premolars.

Composite resins bond to tooth structure through A. Van der Waals forces. B. micromechanical retention. C. chemical bonding.

B. micromechanical retention.

In radiography, minimum magnification and maximum definition are achieved by A. minimum OFD (object-film distance) and minimum FFD (focal-film distance). B. minimum OFD (object-film distance) and maximum FFD (focal-film distance). C. maximum OFD (object -film distance) and maximum FFD (focal-film distance). D. maximum OFD (object-film distance) and minimum FFD (focal-film distance).

B. minimum OFD (object-film distance) and maximum FFD (focal-film distance).

Assuming a maxillary cast is accurately mounted on an articulator, a centric relation record is used to A. determine the vertical dimension of occlusion. B. mount the mandibular cast. C. establish the occlusal plane. D. record the inclination of the condylar guidance.

B. mount the mandibular cast.

The most frequent cause of hyposalivation in elderly patients is A. Alzheimer's disease. B. multiple medications. C. salivary gland atrophy. D. Sjögren's syndrome.

B. multiple medications.

A 23 year old female complains of bilateral stiffness and soreness in the preauricular region. Her symptoms have been present for the past week and are most pronounced in the morning. The most likely cause is A. fibrous ankylosis of the temporomandibular joints. B. nocturnal bruxism. C. early osteoarthritis. D. mandibular subluxation.

B. nocturnal bruxism.

An adult patient has several multilocular lesions associated with vital mandibular teeth. A histopathologic analysis reveals the presence of multinucleated giant cells. The most appropriate management for this patient is to A. extract the involved teeth. B. obtain a serum parathormone (PTH) level. C. commence antibiotic therapy. D. initiate endodontic treatments. E. perform a mandibular resection

B. obtain a serum parathormone (PTH) level.

Excessively dark radiographs result from A. underdevelopment. B. overexposure. C. backward placement of the film. D. too little milliamperage.

B. overexposure.

a permanent first molar of a 7 year old child. The tooth is vital and has no periapical involvement. The most appropriate initial treatment is to perform a/an A. pulp capping. B. partial pulpotomy C. pulpotomy. D. pulpectomy. E. extraction.

B. partial pulpotomy

The retentive arm of a combination clasp (wrought wire retentive arm and cast lingual arm) is better than a cast arm because it A. has a lower yield strength. B. produces less stress on the abutment tooth during removal and insertion. C. can be used to engage deeper undercuts due to a high modulus of elasticity. D. is economical to fabricate.

B. produces less stress on the abutment tooth during removal and insertion.

Patients with thrombocytopenia have an increased risk of A. bacterial infections. B. prolonged bleeding. C. exercise intolerance.

B. prolonged bleeding.

The difference between gingivitis and periodontitis is A. radiographic changes are present only in gingivitis. B. radiographic changes are present only in periodontitis. C. changes in gingival colour are present only in gingivitis. D. changes in gingival colour are present only in periodontitis.

B. radiographic changes are present only in periodontitis.

After an inferior alveolar nerve block injection, a patient would develop seventh nerve paralysis if the injection was made into the A. internal maxillary artery. B. retroparotid space. C. internal pterygoid muscle. D. retromandibular vein. E. pterygoid plexus of veins.

B. retroparotid space.

One week after an amalgam restoration is placed in the mandibular first premolar, the patient returns complaining of a sharp pain of short duration when eating or drinking something cold. Teeth respond normally to electric pulp testing and heat and the radiographs are normal. The most likely diagnosis is A. hypercementosis. B. reversible pulpitis. C. pulpal microabscess. D. acute periradicular periodontitis.

B. reversible pulpitis.

Angle's "subdivision" refers to a molar relationship that is A. bilateral. B. unilateral. C. functional. D. transitional. E. traumatic.

B. unilateral.

When a radiograph shows the inferior alveolar canal superimposed on an impacted mandibular third molar, the canal is A. usually on the lingual side of the tooth. B. usually of the buccal side of the tooth. C. usually passing through the tooth. D. equally likely to be on the buccal or lingual side of the tooth.

B. usually of the buccal side of the tooth.

Tooth 4.5 was treated endodontically 2 years ago and is now sensitive to percussion. There is an isolated, narrow 8mm periodontal pocket on the buccal surface of the tooth. Radiographic findings are within normal limits. The most likely diagnosis is a/an A. endodontic lesion. B. vertical root fracture. C. chronic periodontal lesion

B. vertical root fracture.

A lower molar requiring a crown has an extensive MOD amalgam restoration. The crown margin is most appropriately placed A. on the existing amalgam. B. at the amalgam/tooth junction. C. 1mm apical to the amalgam margin.

C. 1mm apical to the amalgam margin.

The lowest level of fluoride in drinking water which will cause enamel mottling is A. 0.5ppm. B. 1.0ppm. C. 3.0ppm. D. 5.0ppm.

C. 3.0ppm.

Regardless of the focal spot-to-film distance employed for intraoral radiographic examinations, the diameter of the primary beam at the patient's skin surface should NOT be greater than A. the longest side of the film. B. the size of the lead diaphragm. C. 7cm. D. the size of the filter.

C. 7cm.

Which of the following would require a custom incisal guide table for a patient with mutually protected occlusion? A. A fixed partial denture from tooth 3.5-3.7. B. An all ceramic crown on tooth 1.2. C. A fixed partial denture from tooth 1.1-1.3. D. A single ceramometal crown on tooth 1.4.

C. A fixed partial denture from tooth 1.1-1.3.

Which of the following is NOT appropriate to be used to relieve ischemic discomfort of an acute coronary syndrome patient? A. Nitroglycerin. B. Morphine. C. Acetaminophen. D. Oxygen.

C. Acetaminophen.

Which of the following tumours has the best prognosis? A. Osteosarcoma. B. Melanoma. C. Ameloblastoma. D. Adenocarcinoma.

C. Ameloblastoma.

Which of the following should be corrected through orthodontic intervention? A. Maxillary mesiolingual cusps that interdigitate with the central fossa of the mandibular molars. B. Maxillary mesiobuccal cusp tips that contact the mandibular mesiobuccal cusp tips. C. Angle Class I occlusion with 30% overbite and 2mm overjet with mild maxillary spacing. D. Angle Class II molar relationship with 30% overbite, 2mm overjet and good arch alignment.

C. Angle Class I occlusion with 30% overbite and 2mm overjet with mild maxillary spacing.

Drug-induced gingival overgrowth will most likely occur on the interdental papillae of which area? A. Anterior palate. B. Posterior palate. C. Anterior labial. D. Posterior buccal.

C. Anterior labial.

Which of the following does NOT block cyclooxygenase-2? A. Acetaminophen. B. Acetylsalicylic acid. C. Celecoxib. D. Diflunisal. E. Ibuprofen.

C. Celecoxib.

Which of the following results from a necrotic pulp? A. Dentigerous cyst. B. Lateral periodontal cyst. C. Chronic periradicular periodontitis. D. Pulp polyp

C. Chronic periradicular periodontitis.

Which of the following findings is most likely to lead to tooth loss? A. Recurrent bleeding on probing. B. Recurrent periodontal abscess. C. Class II furcation on a mandibular molar. D. Cemental tears.

C. Class II furcation on a mandibular molar.

Which of the following is NOT a risk factor for periodontitis? A. Smoking tobacco. B. Poorly controlled diabetes. C. Coronary heart disease. D. Poor oral hygiene.

C. Coronary heart disease.

For which of the following is nystatin oral suspension an appropriate treatment? A. Herpetic gingivostomatitis. B. Nicotinic stomatitis. C. Denture stomatitis. D. Aphthous stomatitis.

C. Denture stomatitis.

An anxious 67 year old patient presents for consultation to extract tooth 4.6. The medical history includes a heart murmur, hypertension and a cerebrovascular accident 5 years ago. Medications include clopidrogel, hydrochlorothiazide and rosuvastatin. What is the most appropriate preoperative management? A. Prescribing antibiotic prophylaxis. B. Ordering an INR test within 24 hours prior to procedure. C. Discussing with the patient the use of sedation. D. Requesting a complete blood count.

C. Discussing with the patient the use of sedation.

Which of the following preoperative instructions are most appropriate for a well-controlled insulindependent diabetic patient who is scheduled at 9:00 a.m. for the extraction of two teeth under local anesthesia? A. Eat breakfast before the appointment and delay insulin injection until after the appointment. B. Take insulin before the appointment and delay breakfast until after the extractions. C. Eat breakfast before the appointment and take insulin according to the regime prescribed by the physician. D. Delay breakfast and insulin until after the dental appointment.

C. Eat breakfast before the appointment and take insulin according to the regime prescribed by the physician.

Which of the following methods of instrument sterilization uses the lowest temperature? A. Steam autoclave. B. Dry heat oven. C. Ethylene oxide method. D. Glass bead sterilizer. E. Alcohol autoclave.

C. Ethylene oxide method.

Which of the following processes is NOT active in causing tooth eruption? A. Growth of the dentin. B. Growth of the root. C. Growth of the enamel. D. Pressure from periapical tissue

C. Growth of the enamel.

Oral hairy leukoplakia primarily occurs with A. diabetes. B. hepatitis A. C. HIV. D. hypophosphatasia. E. Papillon-Lefèvre syndrome.

C. HIV.

What is the name of the area in which the resin of the adhesive system micromechanically interlocks with dentinal collagen? A. Active zone. B. Smear layer. C. Hybrid layer. D. Adhesive zone.

C. Hybrid layer.

What is the primary etiologic factor for generalized aggressive periodontitis? A. Altered lymphocyte activity. B. Generalized subgingival calculus. C. Impaired polymorphonuclear phagocytosis. D. Bacterial plaque.

C. Impaired polymorphonuclear phagocytosis.

Which of the following is a proliferative response of the soft tissue to an irritant? A. Cellulitis. B. Abscess. C. Pyogenic granuloma. D. Aphthous ulcer.

C. Pyogenic granuloma.

Epinephrine should be administered immediately in which of the following emergencies? A. A severe angina attack. B. A mild asthmatic attack. C. Severe anaphylaxis. D. Severe hypotension

C. Severe anaphylaxis.

Which of the following treatments is CONTRAINDICATED for a patient with necrotizing ulcerative gingivitis? A. Antibiotic therapy. B. Local debridement. C. Topical steroid therapy. D. Warm saline solution rinses

C. Topical steroid therapy.

Which of the following has the greatest effect on the mechanical properties of composite resin? A. Modulus of elasticity of the filler particles. B. Weight fraction of the filler particles. C. Volume fraction of filler particles. D. Hardness of filler particles. E. Size of the filler particles.

C. Volume fraction of filler particles.

From Type I to Type IV gold alloys there is A. an increase in gold content. B. no change in the gold content. C. a reduction in gold content. D. a reduction in platinum content.

C. a reduction in gold content.

The most reliable measurement of the effectiveness of root planing at re-evaluation 4-6 weeks later is A. root smoothness. B. absence of plaque. C. absence of bleeding upon probing. D. increased sulcular fluid flow.

C. absence of bleeding upon probing.

The periodontal ligament A. only has oblique fibres during the eruptive stage. B. increases in width with age. C. achieves its final structural form after complete eruption. D. has osteoblasts as its principle cells.

C. achieves its final structural form after complete eruption.

An advantage of glass ionomer cement is A. low solubility. B. wear resistance. C. adhesion to hard tooth tissues. D. low incidence of sensitivity.

C. adhesion to hard tooth tissues.

An anterior open bite is commonly associated with A. a horizontal growth pattern. B. a functional shift. C. an associated habit. D. a normal swallowing reflex.

C. an associated habit.

Reduced mobility of the temporomandibular joint is called A. Charcot's arthritis. B. osteoarthritis. C. ankylosis. D. arthrosis.

C. ankylosis.

The pulpal floor of an occlusal amalgam preparation on a mandibular first premolar should A. be 2mm into the dentin. B. slope apically from mesial to distal. C. be parallel to the buccolingual cusp plane. D. be perpendicular to the long axis of the tooth.

C. be parallel to the buccolingual cusp plane.

Ionization of local anesthetics is facilitated by a tissue pH A. above 7.4. B. at 7.4. C. below 7.4.

C. below 7.4.

In composite resin restorations, glass ionomer cements can be used as a base because they are A. sedative to a hyperemic pulp. B. neutral in colour. C. biocompatible. D. compatible with the expansion of composite resins.

C. biocompatible.

Antihistamines act by A. increasing the action of histaminase. B. altering the formation of histamine. C. blocking the actions of histamine by competitive inhibition. D. interfering with the degradation of histamine.

C. blocking the actions of histamine by competitive inhibition.

Maximum intercuspation on a complete denture should be established to correspond with centric relation because A. chewing occurs in centric relation. B. it makes it easier to set up the teeth. C. centric relation is a good reference position. D. it prevents cheek biting.

C. centric relation is a good reference position.

The most likely diagnosis for a 5 year old patient with multiple well-defined multilocular radiolucencies of the maxilla and mandible is A. ameloblastic fibromas. B. ameloblastomas. C. cherubism. D. hyperthyroidism. E. hypophosphatasia.

C. cherubism.

While performing a bony tuberosity reduction, the maxillary sinus lining is accidently perforated. The wound is to be closed with a suture that will resorb in 7-10 days. The suture material that should be used is A. polyethylene. B. catgut. C. chromic catgut. D. polylactic acid.

C. chromic catgut.

Condensing osteitis in the periapical region is indicative of a/an A. acute inflammation of the pulp. B. pulpal abscess. C. chronic inflammation of the pulp. D. early apical abscess formation.

C. chronic inflammation of the pulp.

A radiopaque area within the alveolar process containing several rudimentary teeth suggests a/an A. periapical cemento-osseous dysplasia. B. ameloblastoma. C. compound odontoma. D. complex odontoma. E. Pindborg tumor.

C. compound odontoma.

Increased radiographic density is caused by A. decreased mA. B. decreased kVp. C. decreased target-object distance. D. increased object-film distance.

C. decreased target-object distance.

The Fluorosis Index is used to measure the A. degree of protection offered against dental caries by fluoride supplements. B. concentration of fluoride in public water supplies. C. degree or severity of mottled enamel. D. opposition to fluoridation by citizens' groups. E. total amount of fluoride ingested.

C. degree or severity of mottled enamel.

Reduced thyroid hormone level in a child is associated with A. lack of tooth eruption. B. early tooth eruption. C. delayed tooth eruption. D. supernumerary teeth.

C. delayed tooth eruption.

On the basis of "Socransky's Modifications of Koch's Postulates", the potential pathogens in periodontal disease must possess all of the following EXCEPT A. be associated with disease, as evidenced by increases in the number of organisms at diseased sites. B. be eliminated or decreased in sites that demonstrate clinical resolution of disease with treatment. C. demonstrate a host response, in the form of alteration in the host cellular, not humoral, immune response. D. be capable of causing disease in experimental animal models. E. demonstrate virulence factors responsible for enabling the organisms to cause periodontal tissue destruction.

C. demonstrate a host response, in the form of alteration in the host cellular, not humoral, immune response.

The most frequent cause of malocclusion is A. thumbsucking. B. mouth breathing. C. heredity. D. ectopic eruption

C. heredity.

The tissue which is most sensitive to radiation is A. nerve. B. dental pulp. C. lymphoid. D. muscle.

C. lymphoid.

Which Gracey curette is most appropriate for the root planing of a 5mm deep pocket located on the distal surface of tooth 1.7? A. 7/8. B. 9/10. C. 11/12. D. 13/14.

D. 13/14.

Heat treatment alters a gold alloy's A. hardness. B. proportional limit. C. percentage elongation. D. All of the above.

D. All of the above.

Plaque accumulation on tooth surfaces is affected by A. the anatomy, position and surface characteristics of the teeth. B. the architecture of the gingival tissues and their relationship to the teeth. C. friction at the tooth surface from the diet, lips and tongue. D. All of the above.

D. All of the above.

Which of the following is true about the supraeruption of unopposed molars? A. Supraeruption occurs 60% of the time. B. Supraeruption is more prevalent in the mandibular arch. C. Unopposed molars have a mean supraeruption of 3.0mm. D. Attachment loss is one of the main predictors.

D. Attachment loss is one of the main predictors.

A radiograph reveals a radiolucency associated with the apex of tooth 1.5. There is a large restoration but the tooth is asymptomatic and the associated soft tissues appear normal. What is the most likely periapical diagnosis? A. Symptomatic apical periodontitis (acute periradicular periodontitis). B. Asymptomatic apical periodontitis (chronic periradicular periodontitis). C. Acute apical abscess (acute periradicular abscess). D. Chronic apical abscess (chronic periradicular abscess).

D. Chronic apical abscess (chronic periradicular abscess).

A furcation involvement in which the probe extends completely through the furcation is classified as A. incipient. B. Class I. C. Class II. D. Class III.

D. Class III.

The metabolic clearance of which of the following drugs is NOT reduced in an elderly patient? A. Carbamazepine. B. Warfarin. C. Morphine. D. Clindamycin.

D. Clindamycin.

Which of the following is most likely to have its effect reduced in a patient with genetic polymorphism of CYP2D6? A. Acetaminophen. B. Amoxicillin. C. Clindamycin. D. Codeine. E. Ibuprofen.

D. Codeine.

Which of the following is NOT associated with the administration of acetylsalicylic acid? A. Tinnitus. B. Analgesia. C. Antipyresis. D. Constipation. E. Inhibition of prostaglandin synthesis.

D. Constipation.

Which of the following is a sign of local anesthetic overdose? A. Rash. B. Wheezing. C. Fainting. D. Convulsions. E. Swelling.

D. Convulsions.

Which of the following is a sign of an allergic reaction to penicillin? A. Hyperventilation. B. Nausea. C. Oliguria. D. Dermatitis. E. Constipation.

D. Dermatitis.

Which of the following is essential for successful periodontal treatment? A. Scaling. B. Final evaluation and maintenance on a oneyear recall. C. Periodontal flap surgery. D. Elimination of local etiologic factors.

D. Elimination of local etiologic factors.

Which of the following is the most probable postoperative complication of intracoronal bleaching for a tooth that has an inadequate internal seal? A. Fracture. B. Discolouration. C. Internal resorption. D. External root resorption.

D. External root resorption.

A healthy 38 year old has a well-defined radiolucency 2mm in diameter at the apex of tooth 4.1. The tooth responds within normal limits to all tests performed. What is the most appropriate management? A. Extraction. B. Apicoectomy. C. Incision and drainage. D. Follow-up in 6 months. E. Open the tooth for drainage.

D. Follow-up in 6 months.

Which condition is associated with elevated serum alkaline phosphatase and elevated urinary hydroxyproline levels? A. Fibrous dysplasia. B. Paget disease. C. Sjögren syndrome. D. Gardner syndrome.

D. Gardner syndrome.

Which of the following is NOT associated with infectious mononucleosis? A. Pharyngitis. B. Lymphadenopathy. C. Petechiae of the palate. D. Gingival enlargement. E. Fatigue.

D. Gingival enlargement.

The predominant micro-organisms associated with periodontitis are A. Gram-positive aerobes. B. Gram-negative aerobes. C. Gram-positive anaerobes. D. Gram-negative anaerobes.

D. Gram-negative anaerobes.

Which radiograph best depicts the buccal cortex of the mandible? A. Bite-wing. B. Periapical. C. Panoramic. D. Occlusal.

D. Occlusal.

A radiograph displaying a "cotton-wool"' appearance and generalized hypercementosis is suggestive of A. fibrous dysplasia. B. osteopetrosis. C. osteogenesis imperfecta. D. Paget's disease. E. cleidocranial dysplasia.

D. Paget's disease.

Widening of the periodontal ligament space is NOT seen radiographically in A. trauma from occlusion. B. orthodontic tooth movement. C. Scleroderma (systemic sclerosis). D. Paget's disease.

D. Paget's disease.

Upon stimulation of salivary flow, which gland is the main source of salivary volume? A. Submandibular. B. Submaxillary. C. Sublingual. D. Parotid.

D. Parotid.

In a xerostomic patient, which salivary glands are most likely responsible for the lack of lubrication? A. Accessory. B. Labial. C. Parotid. D. Sublingual and submandibular.

D. Sublingual and submandibular.

Which of the following drugs can be administered to manage pain following the acute onset of a migraine? A. Amitriptyline. B. Nifedipine. C. Propranolol. D. Sumatriptan.

D. Sumatriptan.

What is the correct position of the needle tip for the administration of local anesthetic for an inferior alveolar nerve block? A. Anterior to the buccinator muscle. B. Medial to the medial pterygoid muscle. C. Lateral to the ramus of the mandible. D. Superior to the mandibular foramen. E. Inferior to the pterygomandibular raphe.

D. Superior to the mandibular foramen.

An article reports that subjects receiving chlorhexidine varnish had significantly lower (p<0.05) Streptococcus mutans levels than did those receiving fluoride varnish. What type of reporting error is of the most concern when interpreting these results? A. Type II (ß) error. B. Type I (ß) error. C. Type II (α) error. D. Type I (α) error.

D. Type I (α) error.

A gluteal intramuscular injection may be safely administered in which quadrant? A. Lower medial. B. Upper medial. C. Lower lateral. D. Upper lateral.

D. Upper lateral.

The mechanical objectives when preparing the root canal system for obturation with gutta-percha should include A. development of a continuously tapering cone in the root canal. B. removal of irregularities. C. maintenance of an intact foramen. D. all of the above.

D. all of the above.

The most significant factor in the predictable correction of an anterior crossbite is the A. age of the patient. B. degree of incisor overbite. C. shape of the incisors. D. amount of mesio-distal spacing.

D. amount of mesio-distal spacing.

Premature loss of a primary maxillary second molar usually produces a malocclusion in the permanent dentition that is characterized by A. anterior crowding. B. labially displaced maxillary canines. C. delayed eruption of the permanent first molar. D. an Angle Class II molar relationship on the affected side. E. an Angle Class III molar relationship on the affected side.

D. an Angle Class II molar relationship on the affected side.

A bite-wing radiograph of tooth 1.4 reveals caries penetrating one third into the mesial enamel. The most appropriate management of tooth 1.4 is to A. place an amalgam restoration. B. place a porcelain inlay. C. place a direct composite restoration. D. apply fluoride and improve oral hygiene.

D. apply fluoride and improve oral hygiene.

The greatest dimensional change in denture bases will occur A. after the dentures have been in the mouth 24 hours. B. when the dentures have been stored in tap water at room temperature. C. when a complete denture opposes natural teeth. D. at the time the dentures are removed from the flask.

D. at the time the dentures are removed from the flask.

A line angle NOT present on a Class I cavity preparation on tooth 1.5 is A. mesiopulpal. B. buccopulpal. C. linguopulpal. D. axiopulpal.

D. axiopulpal.

The inferior border of the lingual bar of a removable partial denture should A. displace the lingual frenum. B. be in contact with the cingula of the incisor teeth. C. be superior to the gingival border. D. be as inferior as the movement of the frenum of the tongue will permit.

D. be as inferior as the movement of the frenum of the tongue will permit.

Pressure and tension have little effect on growth of A. the frontomaxillary suture. B. the alveolus. C. the mandible. D. cartilage.

D. cartilage.

Decreased alveolar bone density is associated with decreased levels of A. thyroxin. B. hydrocortisone. C. parathyroid hormone. D. estrogen.

D. estrogen.

A patient wearing complete dentures complains of tingling and numbness in the lower lip bilaterally. This is often an indication of A. allergy to denture base material. B. impingement of denture on mandibular nerves. C. defective occlusal contacts. D. impingement of denture upon mental nerves. E. neoplastic invasion of the inferior mandibular nerves

D. impingement of denture upon mental nerves.

The most effective drug for relief of angina pectoris is A. codeine. B. digitalis. C. quinidine. D. nitroglycerin. E. pentobarbital sodium.

D. nitroglycerin.

The facial growth spurt A. occurs in males before females. B. starts on average at 7 years for females. C. starts on average at 13 years for males. D. parallels body growth.

D. parallels body growth.

Methemoglobinemia is a potential complication of an excessive dose of A. bupivacaine. B. lidocaine. C. mepivacaine. D. prilocaine.

D. prilocaine.

An anterior crossbite of a permanent maxillary incisor in the mixed dentition is most often associated with A. a functional shift. B. unexplainable genetic factors. C. lingually situated supernumerary teeth. D. prolonged retention of a primary incisor. E. premature eruption of a maxillary incisor.

D. prolonged retention of a primary incisor.

Vitamin C is essential for A. formation of collagen. B. osteoid. C. dentin. D. cementum. E. All of the above.

E. All of the above.

Which of the following viruses has the greatest chance of transmission in an occupational exposure to a vaccinated dental healthcare worker? A. Hepatitis B. B. Hepatitis C. C. HIV. D. HPV.

B. Hepatitis C.

A decrease of which of the following is indicative of hypoparathyroidism? A. Serum phosphorus. B. Serum calcium. C. Thyroid activity. D. Serum alkaline phosphatase.

B. Serum calcium.

Which of the following would maximize vitamin E intake following osseous surgery? A. Lettuce. B. Wheat germ. C. Eggs. D. Fish.

B. Wheat germ.

A Kennedy Class II denture with no provision for indirect retention causes A. resorption of the supporting edentulous area. B. dislodgement of the saddle area during mastication. C. temporomandibular joint dysfunction. D. gingivitis.

B. dislodgement of the saddle area during mastication.

If mucous glands are seen in the epithelial lining of a dentigerous cyst, this is most appropriately called A. anaplasia. B. metaplasia. C. dysplasia. D. neoplasia. E. hyperplasia.

B. metaplasia.

Which periodontal pathogen can use the hormone estrogen as a growth factor? A. Porphyromonas gingivalis. B. Aggregatibacter (Actinobacillus) actinomycetemcomitans. C. Prevotella intermedia. D. Tannerella forsythia.

C. Prevotella intermedia.

A subluxated tooth will have A. mobility and displacement. B. no mobility and no displacement. C. mobility and no displacement.

C. mobility and no displacement.

Initial therapy for periodontal pockets that are deeper than 5mm must include supragingival scaling and A. polishing. B. irrigation. C. root planning. D. antibiotics.

C. root planning.

Which condition has NOT been associated with Candida albicans infection? A. Median rhomboid glossitis. B. Angular cheilitis. C. Denture stomatitis. D. Aphthous ulcerations. E. Oral hairy leukoplakia.

D. Aphthous ulcerations.

Which of the following conditions may develop as a result of juvenile diabetes mellitus? A. Ataxia. B. Aphasia. C. Deafness. D. Blindness. E. Motor paralysis.

D. Blindness.

When designing a removable partial denture, undercuts are found in which area of teeth? A. Height of contour. B. Suprabulge area. C. Proximal guide plane. D. Infrabulge area.

D. Infrabulge area.

Following a pulpotomy in a second primary molar with extensive occlusal caries, what is the most appropriate restoration? A. Resin-modified glass ionomer. B. Bonded amalgam. C. Composite resin. D. Stainless steel crown.

D. Stainless steel crown.

Following administration of a posterior superior alveolar nerve block, a hematoma occurs. Which of the following statements is correct? A. The formation of this hematoma indicates poor injection technique. B. This nerve block is not commonly associated with hematoma formation. C. Management of this hematoma includes immediate application of heat for at least the first 6 hours. D. The patient may experience trismus the next day.

D. The patient may experience trismus the next day.

A 35 year old female patient has multiple petechiae hemorrhages of the palatal mucosa and ecchymoses of the right and left buccal mucosae. Which of the following is the most likely laboratory finding? A. Anemia. B. Prolonged partial thromboplastin time (PTT). C. Elevated INR. D. Thrombocytopenia.

D. Thrombocytopenia.

A recommended method for disinfecting alginate impressions is to immerse the impression for 10 minutes in A. a complex phenolic. B. 2% glutaraldehyde. C. 10% ethyl alcohol. D. a 1:10 dilution of sodium hypochlorite.

D. a 1:10 dilution of sodium hypochlorite.

The most important objective of occlusal adjustment of a natural dentition is to A. prevent temporomandibular joint syndrome. B. increase the shearing action in mastication. C. improve oral hygiene by preventing food impaction. D. achieve a more favorable direction and distribution of forces of occlusion.

D. achieve a more favorable direction and distribution of forces of occlusion.

A dentist infected with Hepatitis C virus but without disease symptoms should A. not be allowed to practice. B. be allowed to practice but should be excluded from performing exposure prone procedures. C. be allowed to practice but should be excluded from performing exposure prone procedures after assessment and agreement by an expert review panel and if there is compliance with standard precautions (routine practices). D. be allowed to practice after assessment and agreement by an expert review panel and if there is compliance with standard precautions (routine practices).

D. be allowed to practice after assessment and agreement by an expert review panel and if there is compliance with standard precautions (routine practices).

Epinephrine is one drug used in the management of an anaphylactic reaction (type I allergic reaction) because it A. contracts bronchial muscles. B. decreases heart rate. C. decreases systolic blood pressure. D. causes vasoconstriction in many vascular beds.

D. causes vasoconstriction in many vascular beds.

The current recommended regimen of antibiotic prophylaxis for a patient with a prosthetic heart valve and an allergy to penicillin is A. amoxicillin 3g orally one hour before procedure; then 1.5g six hours after initial dose. B. amoxicillin 2g orally one hour before procedure only. C. clindamycin 300mg orally one hour before procedure; then 150mg six hours after initial dose. D. clindamycin 600mg orally one hour before procedure only. E. erythromycin stearate, 2g orally two hours before procedure only.

D. clindamycin 600mg orally one hour before procedure only.

During endodontic treatment a file separates. The fragment is 3mm long and is lodged tightly in the apical third of the canal. No radiographic changes at the apex are evident. In addition to informing the patient, the most appropriate management is to A. extract the tooth. B. perform an apicoectomy and place a retrograde filling. C. resect the apical section of the root containing the separated instrument. D. complete the root canal filling and monitor at recall examination.

D. complete the root canal filling and monitor at recall examination.

In a bilateral sagittal split osteotomy, the segments of the mandible with the condyles are referred to as the A. anterior segments. B. posterior segments. C. proximal segments. D. distal segments.

D. distal segments.

For application of porcelain to a ceramo-metal alloy, the correct viscosity is achieved by mixing the porcelain powder with A. a porcelain modifier. B. pure methyl alcohol. C. a mild detergent. D. distilled water.

D. distilled water.

The yield strength of an orthodontic wire is A. the same as the proportional limit. B. decreased by work hardening. C. the same as the stress at fracture. D. higher than the proportional limit

D. higher than the proportional limit

Patients with resistance to activated protein C (factor V Leiden) are at risk for A. fibrinolysis. B. bleeding diatheses. C. thrombocytopenia. D. hypercoagulation. E. erythrocytosis

D. hypercoagulation.

In a patient with complete dentures, cheek biting may result from A. an excessive vertical dimension of occlusion. B. the use of steep-cusped posterior teeth. C. insufficient coverage of the retromolar pad areas. D. insufficient horizontal overlap of the posterior teeth.

D. insufficient horizontal overlap of the posterior teeth.

Areas of isolated gingival recession are most frequently seen on teeth that are A. nonvital. B. moderately mobile. C. ankylosed. D. labially prominent in the arch.

D. labially prominent in the arch.

The pulpal floor of a Class II cavity is cut perpendicular to the long axis of the tooth EXCEPT in the A. maxillary first premolar. B. maxillary second premolar. C. mandibular second premolar. D. mandibular first premolar.

D. mandibular first premolar.

Tachycardia is a term which describes a pulse rate of A. less than 50. B. less than 60. C. less than 70. D. more than 70.

D. more than 70.

Osteogenesis imperfecta is manifested by A. punched-out radiolucencies in the jaws. B. numerous unerupted supernumerary teeth. C. osteoporosis and anemia. D. multiple fractures and blue sclera.

D. multiple fractures and blue sclera.

The most appropriate radiographic examination for a 4 year old without visible or clinically detectable caries or anomalies, and with open proximal contacts is A. maxillary and mandibular anterior occlusals. B. a pair of posterior bite-wings. C. maxillary and mandibular posterior periapicals. D. no radiographic examination.

D. no radiographic examination.

In the mandibular first premolar, the occlusal dovetail of an ideal disto-occlusal amalgam preparation is usually not extended into the mesial fossa because of the A. small lingual lobe. B. large buccal cusp. C. large buccal pulp horn. D. prominent transverse ridge.

D. prominent transverse ridge.

Percussion of a tooth is used to evaluate A. ankylosis. B. occlusal trauma. C. periapical status. D. pulpal status. E. mobility.

D. pulpal status.

Prior to cementing an onlay in a vital tooth using a resin cement, the application of cavity varnish will A. protect the pulp. B. improve seal. C. reduce prevent postoperative sensitivity. D. reduce bond strength. E. reduce microleakage.

D. reduce bond strength.

The principal ingredient of a zinc phosphate cement powder is A. zinc phosphate. B. silica. C. magnesium oxide. D. zinc-oxide. E. calcium hydroxide.

D. zinc-oxide.

Which of the following is common to all forms of hemorrhagic shock? A. Sepsis. B. Hypovolemia. C. Hypertension. D. Vasoconstriction. E. Impaired tissue perfusion.

E. Impaired tissue perfusion.

Which of the following is a sign of a severe toxic reaction to ketoconazole? A. Jaundice. B. Hypertension. C. Xerostomia. D. Salivary gland swelling

A. Jaundice.

Following root canal therapy, the most desirable form of tissue response at the apical foramen is A. cementum deposition. B. connective tissue capsule formation. C. epithelium proliferation from the periodontal ligament. D. dentin deposition.

A. cementum deposition.

To evaluate an existing occlusion, diagnostic casts should be mounted on an articulator in A. centric relation. B. balancing occlusion. C. either centric relation or balancing occlusion. D. horizontal protrusive relation.

A. centric relation.

For sterilization to occur in an autoclave, all of the following must be monitored EXCEPT A. chemical levels. B. time. C. pressure. D. temperature.

A. chemical levels.

During the setting phase, a dental stone mixture will exhibit A. expansion. B. contraction. C. loss in compressive strength. D. gain in moisture content.

A. expansion.

In a 4 year old the most appropriate treatment for a chronically infected, non-restorable primary first molar is to A. extract it and place a space maintainer. B. observe it until it exfoliates. C. extract it only. D. observe it until it becomes symptomatic.

A. extract it and place a space maintainer.

The prognosis for a replanted developed permanent tooth is most influenced by the A. length of time the tooth has been out of the mouth. B. use of an antibiotic. C. effectiveness of the irrigation of the socket. D. rigidness of the splint applied. E. thoroughness of the curettage of the root surface.

A. length of time the tooth has been out of the mouth.

Short-acting barbiturates are metabolized mainly in the A. liver. B. kidneys. C. small intestine. D. pancreas. E. spleen.

A. liver.

The angle SNA can be used to evaluate the A. maxillary protrusion. B. overbite. C. upper incisor inclination. D. facial height. E. mandibular angle.

A. maxillary protrusion.

Radiographically, a keratocystic odontogenic tumour (odontogenic keratocyst) can appear as a A. mixed radiolucency and radiopacity. B. radiolucency around the crown of an impacted tooth. C. radiolucency containing multiple rudimentary teeth.

A. mixed radiolucency and radiopacity.

To improve denture stability, mandibular molar teeth should normally be placed A. over the crest of the mandibular ridge. B. buccal to the crest of the mandibular ridge. C. over the buccal shelf area. D. lingual to the crest of the mandibular ridge.

A. over the crest of the mandibular ridge.

The antibiotic of choice for a periradicular dental abscess is A. penicillin V. B. cephalosporin. C. erythromycin. D. metronidazole. E. ampicillin.

A. penicillin V.

In the presence of an acute bacterial infection, laboratory tests will show an increase in A. polymorphonuclear leukocytes. B. plasma cells. C. lymphocytes. D. monocytes. E. eosinophils.

A. polymorphonuclear leukocytes.

Adequate position of a needle in the lumen of a blood vessel for a venipuncture is confirmed by the A. presence of blood upon aspiration. B. rapidity of filling of the syringe upon aspiration. C. pain associated with vessel puncture.

A. presence of blood upon aspiration.

The most likely diagnosis for a child with a painful, fiery-red, diffuse gingivitis is A. primary herpetic gingivostomatitis. B. aggressive periodontitis. C. idiopathic fibromatosis. D. aphthous stomatitis.

A. primary herpetic gingivostomatitis.

If removal of teeth is indicated in a patient who is to receive radiation therapy for a carcinoma of the tongue, the teeth should be extracted A. prior to the radiation therapy. B. during the radiation therapy. C. immediately post-radiation therapy. D. six months post-radiation therapy.

A. prior to the radiation therapy.

The short action of thiopental is due to A. rapid redistribution. B. rapid renal excretion. C. rapid metabolism. D. build up of tolerance. E. conjugation with serum proteins

A. rapid redistribution.

The placement of a post in an endodontically treated tooth with minimal coronal tooth structure provides A. retention for the core. B. a ferrule effect. C. reinforcement of the remaining root structure. D. resistance to root fracture.

A. retention for the core.

In partial denture design, the major connector should A. rigidly connect the bilateral components. B. act as a stress-breaker. C. not interfere with lateral forces. D. dissipate vertical forces.

A. rigidly connect the bilateral components.

For an otherwise healthy patient, with an acute localized periodontal abscess, initial treatment must include A. scaling and root planing. B. occlusal adjustment. C. prescription of an antibiotic. D. prescription of an analgesic.

A. scaling and root planing.

The inorganic ion which is the chief offender in hypertension is A. sodium. B. ammonium. C. magnesium. D. potassium.

A. sodium.

The most effective method for protecting dental personnel who expose radiographs is to have them A. stand behind a barrier during exposure. B. stand 2 metres away from the x-ray generator during the exposure. C. wear a lead apron during exposure. D. wear a dosimeter during exposure.

A. stand behind a barrier during exposure.

Saliva production is increased when blood flow to the salivary glands is A. stimulated by parasympathetic activity. B. stimulated by sympathetic activity. C. inhibited by acetylcholine. D. inhibited by cyclic AMP.

A. stimulated by parasympathetic activity.

An acute periapical abscess originating from a mandibular third molar generally points and drains in the A. submandibular space. B. pterygomandibular space. C. buccal vestibule. D. buccal space.

A. submandibular space.

When closing a diastema with direct composite, care must be taken to ensure that the A. width to height relationships are appropriate. B. entire embrasure is filled. C. restoration does not extend subgingivally. D. enamel is reduced 0.3 to 0.5mm prior to etching.

A. width to height relationships are appropriate.

Abnormal development of the first pharyngeal arch may produce defects in the A. zygomatic bones and the external ears. B. mandible and the external nose. C. maxilla and the muscles of facial expression. D. palate and the hyoid bone.

A. zygomatic bones and the external ears.

Generalized aggressive periodontitis is characterized by attachment loss affecting the first molars and incisors and at least A. 1 other permanent tooth. B. 3 other permanent teeth. C. 5 other permanent teeth. D. 7 other permanent teeth

B. 3 other permanent teeth.

The mean annual radiation dose equivalent for human populations on earth is approximately A. 3 to 4 micro- (μ) Sv. B. 3 to 4 milli- (m) Sv. C. 3 to 4 Sv. D. 3 to 4 kSv.

B. 3 to 4 milli- (m) Sv.

The occlusal cavosurface margin for a Class I amalgam restoration should be A. beveled. B. 90°. C. chamfered. D. acute.

B. 90°.

Which of the following statements about sodium is correct? A. Salt-sensitive people should avoid foods like oranges and bananas. B. A high salt intake aggravates but does not cause hypertension. C. Renal sodium excretion varies indirectly with total sodium intake. D. The sodium recommendation is increased during pregnancy.

B. A high salt intake aggravates but does not cause hypertension.

Which of the following most appropriately describes a traumatic neuroma? A. A slow-growing, painless neoplasm. B. A slow-growing, hypersensitive nodule. C. A tumour at a recent extraction site. D. A tumour of the tongue.

B. A slow-growing, hypersensitive nodule.

Which of the following conditions may result from horizontally brushing the teeth? A. Erosion. B. Abrasion. C. Attrition. D. Hypoplasia.

B. Abrasion.

Which of the following can cause xerostomia? A. Candidiasis. B. Acute anxiety. C. Crohn disease. D. Pilocarpine administration. E. McCune Albright syndrome.

B. Acute anxiety.

Which of the following is associated with aggressive periodontitis in adolescents? A. Treponema denticola. B. Aggregatibacter (Actinobacillus) actinomycetemcomitans. C. Porphyromonas gingivalis. D. Prevotella intermedia.

B. Aggregatibacter (Actinobacillus) actinomycetemcomitans.

The most common senile dementia in the elderly is A. vascular dementia. B. Alzheimer's disease. C. a result of cerebral arteriosclerosis. D. multi-infarct dementia.

B. Alzheimer's disease.

Which porcelain stain colour is added to give the appearance of translucency? A. Grey. B. Blue. C. Yellow. D. White. E. Orange.

B. Blue.

Which of the following statements is true? A. A major percentage of radiographically detected proximal radiolucensies in enamel are cavitated. B. Caries progression through enamel can take as long as 6 to 8 years. C. Caries progression can only be arrested in enamel. D. Arrested carious lesions in enamel are softer than intact enamel

B. Caries progression through enamel can take as long as 6 to 8 years.

Which antibiotic is CONTRAINDICATED for a patient with a history of a Type 1 anaphylactic reaction to penicillin? A. Azithromycin. B. Cephalexin. C. Clindamycin. D. Erythromycin.

B. Cephalexin.

Which of the following may be used to disinfect gutta-percha points? A. Autoclave. B. Chemical solutions. C. Flame sterilization. D. Dry heat sterilization.

B. Chemical solutions.

Which of the following is LEAST likely to cause pain? A. Symptomatic apical periodontitis (acute periradicular periodontitis). B. Chronic apical abscess (chronic periradicular abscess). C. Symptomatic irreversible pulpitis. D. Reversible pulpitis.

B. Chronic apical abscess (chronic periradicular abscess).

Which of the following is the most frequent major congenital malformation of the head and neck? A. Cystic hygroma colli. B. Cleft palate. C. Encephalotrigeminal angiomatosis. D. Double lip. E. Commissural pits.

B. Cleft palate.

Which pontic type is best for a knife edge residual ridge where esthetics is not a major concern? A. Sanitary. B. Conical. C. Ridge lap. D. Modified ridge lap.

B. Conical.

Which disorder presents with all permanent teeth exhibiting bulbous crowns, cervical constriction and obliterated pulp canals and chambers? A. Amelogenesis imperfecta. B. Dentinogenesis imperfecta. C. Dentin dysplasia type I. D. Dentin dysplasia type II.

B. Dentinogenesis imperfecta.

A survey of the master cast shows that the 3.5 and 3.7 abutments for a fixed partial denture have different paths of insertion with respect to 3.7. A semi-precision attachment is chosen rather than preparing the teeth again. Where should the male part of the attachment ideally be located? A. Distal of the 3.5 retainer. B. Distal of the 3.6 pontic. C. Mesial of the 3.7 retainer. D. Mesial of the 3.6 pontic.

B. Distal of the 3.6 pontic.

Which of the following is characteristic of an antral pseudocyst? A. Pain and soreness of the face. B. Dome-shaped appearance on a radiograph. C. Buccal expansion of the maxillary sinus.

B. Dome-shaped appearance on a radiograph.

After initiating preventive management for a 16 year old patient with multiple extensive carious lesions, which of the following restorative treatments is most appropriate? A. Place amalgam restorations over the next few months. B. Excavate caries and place temporary restorations within the next few weeks. C. Delay any treatment until the hygiene improves. D. Restore all teeth with composite resin over the next few months.

B. Excavate caries and place temporary restorations within the next few weeks.

The local anesthetic technique requiring the needle to contact the neck of the condyle is the A. posterior superior alveolar nerve block. B. Gow-Gates block. C. Vazirani-Akinosi block. D. inferior alveolar nerve block.

B. Gow-Gates block.

Which of the following improves the prognosis of pulp vitality after indirect pulp capping? A. Use of calcium hydroxide. B. Having a well sealed restoration. C. Reducing other traumas to the pulp. D. Removing all demineralized dentin.

B. Having a well sealed restoration.

Which of the following increases the probability of success of a direct pulp cap? A. Having a large exposure allowing more calcium absorption. B. The ability of the capping and restorative material to seal dentin completely. C. Having copius bleeding that rids the pulp of bacterial toxins. D. Using material that allows swelling of pulpal tissue during healing.

B. The ability of the capping and restorative material to seal dentin completely.

Which of the following is NOT a true cyst? A. Keratocystic odontogenic tumour (odontogenic keratocyst). B. Traumatic bone cyst. C. Radicular cyst. D. Lateral periodontal cyst.

B. Traumatic bone cyst.

Initial caries of enamel has which of the following shape in pits and fissures? A. Inverted V-shape (base towards dentino-enamel junction). B. V-shape (apex towards dentino-enamel junction). C. Inverted U-shape (base towards dentino-enamel junction). D. U-shape (apex towards dentino-enamel junction).

B. V-shape (apex towards dentino-enamel junction).

In achieving hemostasis, external cold application produces A. positive chemotaxis. B. a transient vasoconstriction. C. increased vascular permeability. D. accelerated healing.

B. a transient vasoconstriction.

When compared to zinc phosphate cement, glass ionomer cement has a/an A. lower solubility in oral fluids. B. ability to release fluoride. C. higher compressive strength. D. lower film thickness.

B. ability to release fluoride.

The gingival tissues remain healthier when margins of crowns are placed A. about lmm below the gingival crest. B. above the gingival crest. C. at the gingival crest.

B. above the gingival crest.

For a patient with cardiovascular disease, local anesthesia A. affects blood pressure more than general anesthesia. B. affects blood pressure less than general anesthesia. C. is responsible for bacteremia.

B. affects blood pressure less than general anesthesia.

Trauma from occlusion A. initiates gingivitis. B. affects the blood supply to gingivae. C. initiates periodontitis.

B. affects the blood supply to gingivae.

If an impression were taken with a polysulfide impression material of teeth exhibiting severe external undercuts a stone model should be poured A. immediately. B. after 30 minutes. C. after 24 hours.

B. after 30 minutes.

The appliance of choice to correct an anterior crossbite is determined by the A. amount of overbite. B. age of the patient. C. cooperation of the patient. D. practitioner preference.

B. age of the patient.

Compared to dental amalgam, posterior composite resins A. require less time to place. B. allow for conservation of tooth structure. C. have higher thermal conductivity. D. have greater clinical longevity.

B. allow for conservation of tooth structure.

The dimensional stability of polyether impression material is considered to be good EXCEPT if the material is A. dehydrated. B. allowed to absorb water after setting. C. used in uneven thickness. D. distorted by rapid removal of the impression from the mouth. E. contaminated with latex.

B. allowed to absorb water after setting

The primary stress bearing area of the maxillary complete denture is the A. hard palate. B. alveolar ridge. C. soft palate. D. zygoma.

B. alveolar ridge.

A preparation for a Class II amalgam restoration in primary molars should include A. occlusally divergent buccal and lingual walls. B. an axial wall that parallels the dentino-enamel junction. C. an isthmus that occupies two thirds of the intercuspal distance. D. undercut enamel and dentin.

B. an axial wall that parallels the dentino-enamel junction.

Papillary hyperplasia under a denture is usually due to A. a candida infection. B. an ill fitting denture. C. failure to remove the denture at night. D. an allergy to the denture material.

B. an ill fitting denture.

An 8 year old patient has a permanent central incisor with a necrotic pulp and a wide open apex. The most appropriate management is to perform a/an A. pulpotomy. B. apexification. C. regenerative endodontics. D. root canal therapy.

B. apexification.

The palate grows in length by A. endochondral growth. B. apposition on the free edge of the palatine bone. C. the downward and forward growth of the nasal septum.

B. apposition on the free edge of the palatine bone.

To ensure maximum marginal strength for an amalgam restoration the cavosurface angle should A. approach 45 degrees. B. approach 90 degrees. C. be beveled. D. be chamfered.

B. approach 90 degrees.

Cephalometric standards A. can be applied to patients universally. B. are the basis of orthodontic diagnostic. C. very with patient development.

B. are the basis of orthodontic diagnostic.

Ludwig's angina may cause death by A. heart failure. B. asphyxia. C. convulsions. D. paralysis of muscles of respiration. E. pyemia.

B. asphyxia.

The Periodontal Screening and Recording System (PSR®) is designed to A. document the progression of periodontitis. B. assess the periodontal status of a patient. C. measure loss of attachment for large patient populations. D. evaluate oral hygiene.

B. assess the periodontal status of a patient.

Epinephrine in a local anesthetic solution A. increases absorption of the anesthetic. B. assists hemostasis at the site of injection. C. decreases the action of the anesthetic agent. D. assists in post-operative healing.

B. assists hemostasis at the site of injection.

An end result of ionizing radiation used to treat oral malignancies is A. deformity of the jaws. B. reduced vascularity of the jaws. C. increased vascularity of the jaws. D. increased brittleness of the jaws.

B. reduced vascularity of the jaws.

Proper collimation of the useful beam for film size and focal spot-film distance reduces A. image definition. B. secondary radiation. C. radiographic contrast. D. intensity of the central beam.

B. secondary radiation.

When compared to dental amalgams made from lathe cut particles, dental amalgams made from spherical particles A. require more mercury. B. set more quickly. C. are more difficult to adapt to the cavity preparation. D. require higher condensation forces.

B. set more quickly.

All of the following are possible effects of acetylsalicylic acid except A. reduction of fever. B. shortening of bleeding time. C. suppression of inflammatory response. D. bleeding from the gastrointestinal tract.

B. shortening of bleeding time.

Intraoral soft tissue examination will NOT assist in the diagnosis of A. lichen planus. B. sinusitis. C. erythema multiforme. D. anemia. E. vitamin deficiencies

B. sinusitis.

The smear layer created by root canal instrumentation can be removed by A. hydrogen peroxide and ethyl chloride. B. sodium hypochlorite and EDTA. C. chlorhexidine and chloroform. D. calcium hydroxide and pheno

B. sodium hypochlorite and EDTA.

Increased overjet, moderate facial convexity and a deep labio-mental sulcus is most characteristic of which facial type? A. Prognathic. B. Orthognathic. C. Retrognathic.

C. Retrognathic.

A daily dose of 81 mg of acetylsalicylic acid is used for its A. analgesic properties. B. antipyretic effect. C. antiplatelet action. D. anti-inflammatory function.

C. antiplatelet action.

A daily chlorhexidine rinse following periodontal flap surgery is primarily used to A. enhance regeneration. B. encourage wound healing. C. facilitate plaque control. D. create new attachment.

C. facilitate plaque control.

Hypothyroidism in adults is associated with A. exophthalmos. B. weight loss. C. generalized edema. D. tachycardia. E. mental defects.

C. generalized edema.

One week following extraction of teeth 1.8 and 4.8, an 18 year old male returns to the dental office complaining of persistent bleeding from the extraction sites. The medical history is unremarkable, except for episodes of bruising and joint swelling as a child. Subsequent blood tests show normal bleeding time and a factor VIII level of 14%. The most likely cause of the bleeding is A. a dry socket. B. ibuprofen intake. C. hemophilia A. D. cirrhosis of the liver. E. vitamin K deficiency.

C. hemophilia A.

Increased tooth mobility and the absence of lamina dura are signs of A. hyperthyroidism. B. hyperpituitarism. C. hyperparathyroidism. D. scleroderma.

C. hyperparathyroidism.

Multiple giant cell lesions of the bone are associated with A. hyperthyroidism. B. hypothyroidism. C. hyperparathyroidism. D. hypoparathyroidism.

C. hyperparathyroidism.

To prevent mesial drift of a permanent first molar, the ideal time to place a distal extension space maintainer is A. as soon as the tooth erupts through the gingival tissue. B. after the permanent second molar has erupted. C. immediately after extraction of the primary second molar. D. as soon as the extraction site of the primary second molar has completely healed.

C. immediately after extraction of the primary second molar.

Yellow or brown stains appearing on radiographs months after processing result from A. processing at an excessive temperature. B. storing radiographs at 30°C. C. incomplete fixing and/or washing of radiographs. D. using expired film.

C. incomplete fixing and/or washing of radiographs.

Incomplete tooth fracture A. can readily be diagnosed using transillumination. B. most commonly involves the supporting cusps. C. is associated with medium to large-sized restorations. D. elicits dull, prolonged pain on chewing.

C. is associated with medium to large-sized restorations.

Polycarboxylate cement may be used as a base material beneath a metallic restoration because A. its thermal conductivity is similar to dentin and enamel. B. its thermal diffusivity is similar to dentin and enamel. C. its compressive strength when set will resist forces of condensation. D. All of the above.

C. its compressive strength when set will resist forces of condensation.

Compared to heat-cured acrylic resins, cold-cure acrylic resins are A. harder and more colour stable. B. less hard and more colour stable. C. less hard and less colour stable. D. harder and less colour stable.

C. less hard and less colour stable.

Following premature deciduous tooth loss, space loss occurs most frequently in the area of the A. maxillary lateral incisor. B. mandibular central incisor. C. mandibular second premolar. D. maxillary first premolar.

C. mandibular second premolar.

Benzodiazepines A. may produce hypertension. B. are contraindicated in a patient with acute open angle glaucoma. C. may produce muscular weakness. D. may cause insomnia.

C. may produce muscular weakness.

In order to prevent gingival recession, a full gold crown should have A. a slightly narrow food table. B. a slightly overcontoured tooth form. C. normal contour reproduced. D. the margins extended 1mm subgingivally.

C. normal contour reproduced.

A labial frenum causes a diastema between the permanent maxillary central incisors. The lateral incisors and canines have not erupted. The most appropriate immediate management is to A. perform a frenectomy. B. close the space with a fixed orthodontic appliance. C. observe the case until the eruption of permanent maxillary lateral incisors and canines. D. close the space with a removable orthodontic appliance. E. close the space after the eruption of the permanent lateral incisors.

C. observe the case until the eruption of permanent maxillary lateral incisors and canines.

Proximal retention grooves are necessary to provide resistance for proximal-occlusal amalgam restorations when the A. occlusal extension is wide faciolingually relative to the proximal extension. B. restoration is a pin-retained cusp replacement. C. occlusal extension is narrow faciolingually relative to the proximal extension. D. bonded amalgam technique is not being used.

C. occlusal extension is narrow faciolingually relative to the proximal extension.

Rests on terminal abutment teeth for a removable partial denture provide A. primary retention. B. indirect retention. C. occlusal force transmission. D. lateral force transmission.

C. occlusal force transmission.

A mixed lesion with a radiolucent rim and corticated border causing impaction of a permanent tooth is most likely a/an A. adenomatoid odontogenic tumour. B. calcifying epithelial odontogenic tumour. C. odontoma. D. ameloblastic fibro-odontoma.

C. odontoma.

A very apprehensive patient experiencing pain may be prescribed a barbiturate, chloral hydrate or an antihistamine to control the anxiety. In which of the following would you expect an exaggerated response to the use of these drugs? A/An A. diabetic. B. arthritic patient. C. patient with chronic renal disease.

C. patient with chronic renal disease.

In an 8 year old patient, the most appropriate management of mild anterior crowding is to A. interproximally reduce all proximal contacts. B. extract the deciduous canines. C. perform a space analysis. D. regain space in the arch.

C. perform a space analysis.

A 2cm, white lesion of the buccal mucosa has not resolved after elimination of all local irritants. The most appropriate management is to A. cauterize it. B. apply toluidine blue staining. C. perform an incisional biopsy. D. re-examine at 6 month intervals. E. refer patient to family physician.

C. perform an incisional biopsy.

A laboratory remount of processed dentures is done in order to correct occlusal disharmony produced by errors primarily in the A. mounting of the casts on the articulator. B. registration of jaw relation records. C. processing of acrylic dentures. D. registration of condylar guidance.

C. processing of acrylic dentures.

An epinephrine-containing retraction cord has the potential of A. interfering with the setting of the impression material. B. causing tissue necrosis. C. producing a systemic reaction. D. discolouring gingival tissue.

C. producing a systemic reaction.

Antiobiotic prophylaxis prior to tooth extraction is required for (a) A. diabetes mellitus. B. mitral valve prolapse with regurgitation. C. prosthetic heart valve. D. organic heart murmur. E. functional heart murmur.

C. prosthetic heart valve.

The most common abuser of an elderly person is a/an A. friend or acquaintance. B. adult child. C. spouse. D. sibling.

C. spouse.

A patient in the late mixed dentition stage has an end-to-end first molar relationship with normal skeletal and incisor relationships. What is the most probable occlusal relationship at age 20? A. Angle Class II without anterior overlap. B. Angle Class II with anterior overlap. C. Angle Class I without anterior overlap. D. Angle Class I with anterior overlap.

D. Angle Class I with anterior overlap.

What is the most stable area for support of a mandibular complete denture? A. Retromylohyoid area. B. Residual ridge. C. Mylohyoid ridge. D. Buccal shelf. E. Genial tubercle.

D. Buccal shelf.

What is the most likely cause of a maxillary denture dislodging when the patient opens wide or makes extreme lateral excursions? A. Insufficient posterior palatal seal. B. Poor denture base adaptation. C. Labial frenum impingement. D. Coronoid process interference. E. Pronounced midpalatal raphe.

D. Coronoid process interference.

Which type of malocclusion should be corrected as early as possible? A. Class II Division 1 associated with an anterior open bite. B. Class II Division 2 associated with an increased anterior overbite. C. Class III associated with an anterior open bite. D. Cross-bite associated with a functional shift of the mandible from initial contact to maximum intercuspation. E. Anterior open bite associated with a lip or digit sucking habit.

D. Cross-bite associated with a functional shift of the mandible from initial contact to maximum intercuspation.

Which laboratory test is the most appropriate in a patient with acute bacterial infection who does not respond to routine antibiotic therapy? A. Platelet count. B. Hemoglobin level. C. Cytological smear. D. Culture and sensitivity. E. Immunoglobulin concentration.

D. Culture and sensitivity.

Which ethical principle is violated when a dentist practices while impaired? A. Autonomy. B. Beneficence. C. Justice. D. Nonmaleficence. E. Veracity.

D. Nonmaleficence.

Which of the following is the most active cell in synthesizing and secreting antibodies? A. Mast cell. B. Macrophage. C. Eosinophilic granulocyte. D. Plasma cell. E. T-cell lymphocyte.

D. Plasma cell.

Which of the following drugs does NOT cause gingival enlargement? A. Nifedipine. B. Cyclosporine. C. Phenytoin. D. Prednisolone.

D. Prednisolone.

Which of the following conditions requires further information prior to making a decision regarding antibiotic prophylaxis before a surgical procedure? A. Previous coronary bypass. B. Presence of a coronary artery stent. C. Existing mitral valve prolapse. D. Repaired congenital heart disease.

D. Repaired congenital heart disease.

Which of the following conditions is NOT associated with the fully edentulous state? A. Residual ridge reduction. B. Decrease in masticatory function. C. Altered taste perception. D. Rheumatoid arthritis

D. Rheumatoid arthritis

What is the most frequent malignant tumour of the tongue? A. Adenoma. B. Adenocarcinoma. C. Rhabdomyosarcoma. D. Squamous cell carcinoma. E. Granular cell myoblastoma.

D. Squamous cell carcinoma.

In a cavity preparation which closely approximates the pulp, you would protect the pulp with A. a zinc phosphate cement base. B. a calcium hydroxide cement base. C. a calcium hydroxide wash and cavity varnish. D. a calcium hydroxide cement liner and a glass ionomer cement base.

D. a calcium hydroxide cement liner and a glass ionomer cement base.

Recurrent herpes labialis is A. caused by a different organism than is primary herpetic gingivostomatitis. B. a form of disease which heals by scarring. C. occurs more frequently in children. D. a contagious lesion.

D. a contagious lesion.

The higher modulus of elasticity of a chromiumcobalt- nickel alloy, compared to a Type IV gold alloy, means that chromium-cobalt-nickel partial denture clasp will require A. a thicker cross section. B. a shorter retentive arm. C. more taper. D. a shallower undercut.

D. a shallower undercut.

Fractured incisal angles in the permanent teeth of adolescent patients are best restored using A. glass-ionomer. B. gold castings. C. full coverage restorations. D. acid etch composite resin techniques.

D. acid etch composite resin techniques.

Trauma from occlusion may A. initiate marginal gingivitis. B. affect the blood supply to gingiva. C. initiate periodontitis. D. affect the progression of periodontitis

D. affect the progression of periodontitis

Adjustment of the occlusal plane of natural teeth opposed by a complete or partial denture should be completed A. after the teeth have been set on the trial denture. B. immediately after making the final casts. C. upon delivery of the denture. D. after the diagnosis and treatment plan has been established.

D. after the diagnosis and treatment plan has been established.

A root fragment of an impacted third molar could be displaced into the submandibular space during its surgical removal when the A. attachment of the mylohyoid muscle is apical of the roots. B. root of the mandibular third molar lies close to the buccal cortex. C. buccal cortical bone in the mandibular third molar area is extremely thin. D. bone on the lingual surface is fenestrated below the mylohyoid muscle.

D. bone on the lingual surface is fenestrated below the mylohyoid muscle.

The start of a mandibular bone formation coincides with A. tooth germ development. B. completion of fusion of the facial processes. C. bone formation in the maxilla. D. completion of a mandible cartilage model.

D. completion of a mandible cartilage model.

Most cases of erosive oral lichen planus are effectively treated with A. antifungals. B. antibacterials. C. antimalarials. D. corticosteroids.

D. corticosteroids.

The "smear layer" is an important consideration in A. plaque accumulation. B. caries removal. C. pulp regeneration. D. dentin bonding.

D. dentin bonding.

The canal of a maxillary canine has been instrumented to within 1mm of the apex and is ready to be obturated. A radiograph indicates that the master cone is 2.5mm short of the apex. The most appropriate management is to A. proceed with the filling as the cone is within acceptable limits. B. fit a larger cone within 2mm of the apex. C. cut the cone 1mm and insert. D. discard the cone and fit a smaller one.

D. discard the cone and fit a smaller one.

Cervical caries on the maxillary primary incisors in a 12-month old child is most likely caused by A. lack of systemic fluoride. B. poorly formed enamel. C. lack of calcium during pregnancy. D. excessive bottle use.

D. excessive bottle use.

A patient complains of the discolouration of an upper central incisor. Radiographically, the pulp chamber and the root canal space are obliterated and the periodontal ligament space appears normal. The most appropriate management would be to A. perform endodontic treatment and nonvital bleaching. B. perform endodontic treatment and fabricate a porcelain veneer. C. fabricate a metal-ceramic crown. D. fabricate a porcelain veneer.

D. fabricate a porcelain veneer.

Generally, glass ionomer cements contain A. zinc oxide and distilled water. B. zinc oxide and polyacrylic acid. C. fluoroaluminosilicate powder and orthophosphoric acid. D. fluoroaluminosilicate powder and polyacrylic acid.

D. fluoroaluminosilicate powder and polyacrylic acid.

The most likely complication associated with the extraction of an isolated maxillary second molar is A. a dry socket. B. nerve damage. C. fracture of the malar ridge. D. fracture of the tuberosity. Decreased

D. fracture of the tuberosity.

All of the following conditions are associated with AIDS EXCEPT A. acute marginal periodontitis. B. hairy leukoplakia. C. candidiasis. D. geographic tongue.

D. geographic tongue.

An incipient lesion on an interproximal surface is usually located A. at the contact area. B. facial to the contact area. C. lingual to the contact area. D. gingival to the contact area. E. occlusal to the contact area.

D. gingival to the contact area.

Clinical management of gingival enlargement caused by Phenytoin (Dilantin®) therapy includes A. the use of analgesics to control pain. B. withdrawal of the medication. C. extraction of the teeth. D. gingivectomy and maintenance of good oral hygiene. E. a mouth guard to control mouth breathing.

D. gingivectomy and maintenance of good oral hygiene.

Calculus attaches to tooth surfaces by all of the following EXCEPT A. organic pellicle. B. mechanical locking to tooth/root irregularities. C. close adaption to cementum. D. hemidesmosomes.

D. hemidesmosomes.

Orthodontic uprighting of a mandibular second molar that has tipped mesially into the space of a missing mandibular first molar in an otherwise intact dentition may result in A. decreased overbite. B. intrusion of the mandibular second molar. C. extrusion of the mandibular second premolar. D. initial mobility of the mandibular second molar. E. development of posterior crossbite.

D. initial mobility of the mandibular second molar

A vital canine is to be used as the anterior abutment of a four unit fixed partial denture and it has 2.0mm remaining coronal tooth structure. The most acceptable foundation restoration would be A. bonded amalgam core build-up. B. a pin retained amalgam core build-up. C. a pin retained composite resin core build-up. D. intentional devitalization followed by a post and core restoration.

D. intentional devitalization followed by a post and core restoration.

A 10 year old patient is missing a permanent maxillary left lateral incisor. There are no other orthodontic problems. A dental implant is planned when the patient is an adult. The most appropriate management is to A. extract the primary maxillary left canine as soon as possible. B. encourage the permanent left canine to erupt into an Angle's Class I relationship. C. let the permanent canine erupt into a mesial position and reposition it into an Angle's Class I as soon as possible. D. let the permanent canine erupt into a mesial position and reposition it into an Angle's Class I before placing the implant. E. let the permanent canine erupt into a mesial position and place the implant distal to the canine.

D. let the permanent canine erupt into a mesial position and reposition it into an Angle's Class I before placing the implant.

The labial/buccal attached gingiva on permanent teeth is normally widest at the A. mandibular canines. B. maxillary premolars. C. mandibular premolars. D. maxillary lateral incisors.

D. maxillary lateral incisors.

If a complete mandibular denture causes a burning sensation in the premolar region, this is due to the denture exerting pressure in/on the A. fibres of the buccinator muscle. B. lingual branch of the mandibular division of the trigeminal nerve. C. underlying bone. D. mental branch of the inferior alveolar nerve. E. buccal frenum area.

D. mental branch of the inferior alveolar nerve.

In restoring occlusal anatomy, the protrusive condylar path inclination has its primary influence on the morphology of A. cusp height. B. anterior teeth only. C. mesial inclines of maxillary cusps and distal inclines of mandibular cusps. D. mesial inclines of mandibular cusps and distal inclines of maxillary cusps.

D. mesial inclines of mandibular cusps and distal inclines of maxillary cusps.

The most frequent cause of death occurring under general anesthesia is A. overdosage of anesthetic agent. B. cardiac arrest. C. traction on the viscera. D. mismanagement of the airway. E. overdosage of premedication.

D. mismanagement of the airway.

Extreme resorption of an edentulous mandible can bring the alveolar ridge to the level of the attachment of the A. buccinator, styloglossus and geniohyoid muscles. B. mylohyoid, buccinator and styloglossus muscles. C. superior constrictor, mylohyoid and buccinator muscles. D. mylohyoid, buccinator and genioglossus muscles.

D. mylohyoid, buccinator and genioglossus muscles.

The most prevalent inflammatory cells found in gingival tissue 24 hours following flap surgery are A. monocytes. B. macrophages. C. lymphocytes. D. polymorphonuclear leukocytes.

D. polymorphonuclear leukocytes.

healthy gingiva, the level of the interproximal alveolar crest is related to the A. thickness of the alveolar process. B. location of the gingival margin. C. amount of underlying trabecular bone. D. position of the cemento-enamel junction.

D. position of the cemento-enamel junction.

To ensure a clinically acceptable setting time, polyalkenoic cements contain A. salicylic acid. B. phosphoric acid. C. maleic acid. D. tartaric acid. E. itaconic acid.

D. tartaric acid.

The most logical explanation for causing swelling beneath the eye caused by an abscessed maxillary canine is that the A. lymphatics drain superiorly in this region. B. bone is less porous superior to the root apex. C. infection has passed into the angular vein which has no valves. D. the root apex lies superior to the attachment of the caninus and levator labii superioris muscles.

D. the root apex lies superior to the attachment of the caninus and levator labii superioris muscles.

The primary stimulus for growth of the mandible is A. genetic. B. epigenetic. C. environmental. D. functional. E. A. and D.

E. A. and D.

Disuse atrophy of the periodontium causes A. changes in the arrangement of fibre bundles. B. narrowing of the periodontal ligament. C. osteoporosis of the alveolar process. D. decrease in tooth mobility. E. All of the above.

E. All of the above.

In an 11 year old with an otherwise acceptable occlusion, an impacted maxillary canine A. could be extracted. B. could be retained and the first premolar removed to allow the canine to erupt. C. could be surgically exposed to speed its eruption. D. could constitute a problem requiring consultation with an orthodontist. E. All of the above.

E. All of the above.

The benefits of open flap debridement alone include A. direct access for thorough debridement. B. pocket reduction. C. increased opportunity for new attachment. D. A. and B. E. All of the above.

E. All of the above.

When uprighting a molar to be used as a bridge abutment, consideration must be given to the A. local periodontium. B. vertical changes due to tipping of the molar. C. residual space for a pontic. D. use of fixed appliances for optimum control. E. All of the above.

E. All of the above.

Which treatment procedure is indicated for a patient with asymptomatic age related gingival recession? A. Connective tissue graft. B. Gingivoplasty. C. Lateral sliding flap. D. Gingival graft. E. No treatment.

E. No treatment.

A line angle NOT present on a Class I cavity preparation on tooth 1.6 is A. mesiopulpal. B. buccopulpal. C. linguopulpal. D. axiopulpal. E. None of the above.

E. None of the above.

Sterilization of carious dentin without pulp injury is assured by the application of A. phenol. B. 70% ethyl alcohol. C. chlorhexidine. D. absolute alcohol. E. None of the above.

E. None of the above.

Which of the following "in office" preventive procedures is most practical and effective for an uncooperative 4-year old patient from a noncompliant family? A. Oral hygiene instruction. B. Dietary counseling. C. Fluoride varnish every six months. D. Fluoride supplements. E. Pit and fissure sealants on all primary molars.

E. Pit and fissure sealants on all primary molars.

Heated impression modeling compound is "tempered" in warm water before placement in the mouth in order to A. avoid burning the soft tissues. B. reduce contraction error. C. initiate a chemical reaction. D. minimize distortion.

A. avoid burning the soft tissues.

During orthodontic therapy, the width of the periodontal ligament radiographically appears A. increased. B. decreased. C. unchanged.

A. increased.

Overhangs on restorations initiate chronic inflammatory periodontal disease by A. increasing plaque retention. B. increasing food retention. C. causing traumatic occlusion. D. causing pressure atrophy.

A. increasing plaque retention.

A 7 year old child who complains of pain when eating, has a large carious lesion on a permanent molar. A radiograph reveals no periapical change. The most appropriate treatment is a/an A. indirect pulp capping. B. direct pulp capping. C. partial pulpotomy. D. pulpotomy.

A. indirect pulp capping.

Caries disclosing solution used during the excavation of a deep carious lesion will determine the extent of A. infected dentin. B. affected dentin. C. inorganic dentinal matrix deminieralization. D. reversibly denatured dentinal matrix.

A. infected dentin.

The most frequent location of a dentigerous cyst is the A. inferior third molar area. B. symphysis of the mandible. C. midline of the hard palate. D. apical area of a devitalized tooth. E. premolar area.

A. inferior third molar area.

The most common method of entry of infection by the tubercle bacillus is A. inhalation. B. ingestion. C. needle-stick.

A. inhalation.

If hydroquinone is added to the monomer, it will A. inhibit polymerization. B. initiate polymerization. C. activate polymerization. D. None of the above.

A. inhibit polymerization.

Hypochromic anemia is associated with A. iron deficiency. B. aminopyrine therapy. C. vitamin B12 deficiency. D. folic acid deficiency.

A. iron deficiency.

Maxillary skeletal arch expansion A. is easiest in the preadolescent patient. B. requires surgery in skeletally immature patients. C. decreases arch space. D. increases overbite.

A. is easiest in the preadolescent patient.

Presence of bleeding on probing A. is indicative of current inflammation. B. is indicative of past inflammation. C. has a high positive predictive value (PPV) for further attachment loss.

A. is indicative of current inflammation.

The presence of hepatitis B surface antigen (HBsAg) and hepatitis B e antigen (HBeAg) in blood indicates the individual A. is infectious for hepatitis B. B. has acquired immunity to hepatitis B. C. is not infectious for hepatitis B and has not acquired immunity to hepatitis B. D. has never been infected with hepatitis B.

A. is infectious for hepatitis B.

Glossodynia can be associated with A. vitamin B12 deficiency. B. occult malignancy. C. lichenoid reaction to amalgam. D. sialorrhea. E. vitamin D overdose.

A. vitamin B12 deficiency.

The best method to control the setting time of an irreversible hydrocolloid without affecting its physical properties is to alter the A. water temperature. B. water:powder ratio. C. mixing time. D. composition.

A. water temperature.

The pterygomaxillary fissure is formed by the maxilla and which other bone? A. Temporal. B. Sphenoid. C. Frontal. D. Occipital.

B. Sphenoid.

During normal growth, the gnathion, as viewed on successive cephalograms, will move A. downward and backward. B. downward and forward. C. backward and upward. D. forward only.

B. downward and forward.

The primary objective of periodontal debridement is removal of A. calculus. B. plaque. C. cementum. D. dentin.

B. plaque.

A double blind controlled clinical trial that assessed the analgesic effects of etoricoxib and comparator agents on the second and third days after third molar extraction included 600 officers from dental clinics at 10 military bases. What form of bias is present in this study? A. Publication. B. Recall. C. Sampling. D. Selection.

C. Sampling.

The cells responsible for root resorption are A. fibroblasts. B. cementoblasts. C. osteoblasts. D. osteoclasts.

D. osteoclasts.

What is the most common intraoral location of squamous cell carcinoma? A. Lateral tongue. B. Buccal mucosa. C. Tonsilar bed. D. Attached gingiva.

A. Lateral tongue.

Which of the following is NOT used to inhibit calcification of plaque? A. Fluoride. B. Magnesium. C. Pyrophosphate. D. Zinc.

B. Magnesium.

Periapical osseous dysplasia (periapical cementoosseous dysplasia) is A. painful. B. expansile. C. associated with vital teeth. D. premalignant.

C. associated with vital teeth.

Patients who have gingival enlargements surgically removed should be forewarned that there is a high incidence of A. altered taste sensation. B. dentinal hypersensitivity. C. reoccurrence of gingival enlargement. D. post-operative swelling.

C. reoccurrence of gingival enlargement.

If there is insufficient arch space for a permanent tooth to erupt, the tooth may A. cause resorption of the root of another tooth. B. erupt out of position. C. not erupt. D. All of the above.

D. All of the above.

Tooth 3.3 has a 9mm probing depth with a 6mm three-wall infrabony mesial defect. It tests vital and is not mobile. Which of the following is the most appropriate treatment? A. Gingival curettage. B. Modified Widman flap. C. Osseous resective surgery. D. Guided tissue regeneration

D. Guided tissue regeneration

Which of the following is consistent with a diagnosis of reversible pulpitis? A. Lingering pain to cold. B. Pain on percussion. C. Lingering pain to heat. D. No spontaneous pain.

D. No spontaneous pain.

Abrasion is most commonly seen on the A. lingual surface of posterior teeth. B. occlusal surface of posterior teeth. C. incisal edges. D. facial surfaces of teeth.

D. facial surfaces of teeth.

Most bone loss following the placement of dental implants occurs at A. 0 - 1 year. B. 2 - 4 years. C. 5 - 7 years. D. 10 - 12 years.

A. 0 - 1 year.

Which of the following drugs are classic antagonists for curare over-dosage? A. Anticholinesterases. B. Ganglionic stimulants. C. Ganglionic blocking agents. D. Alpha adrenergic blocking agents. E. Beta adrenergic blocking agents.

A. Anticholinesterases.

Which of the following fibre groups are attached to bone? A. Apical. B. Interradicular. C. Transseptal. D. Circular

A. Apical.

Which of the following structures may be associated with the role of the central nervous system in sleep (nocturnal) bruxism? A. Basal ganglia (nigrostriatal). B. A delta and C nerves. C. Sphenopalatine ganglion. D. Petrous nerves.

A. Basal ganglia (nigrostriatal).

On a bite-wing radiograph of posterior teeth, which of the following is most likely to be misdiagnosed as proximal caries? A. Cemento-enamel junction. B. Marginal ridge. C. Carabelli cusp. D. Calculus. E. Cemental tear.

A. Cemento-enamel junction.

On a bite-wing radiograph of posterior teeth, which of the following is most likely to be misdiagnosed as proximal caries? A. Cemento-enamel junction. B. Marginal ridge. C. Carabelli cusp. D. Calculus. E. Cemental tear. The

A. Cemento-enamel junction.

Which of the following is most often associated with a nonvital tooth? A. Chronic periradicular periodontitis. B. Internal resorption. C. Periapical cemento-osseous dysplasia. D. Hyperplastic pulpitis.

A. Chronic periradicular periodontitis.

In which of the following will the effects of polymerization shrinkage be greatest? A. Class I occlusal restoration. B. Preventive resin restoration. C. Direct veneer restoration. D. Class IV restoration.

A. Class I occlusal restoration.

Which one of the following would be of greatest value in determining the etiology of an oral ulceration? A. History of the oral lesion. B. Cytological smear. C. Systemic evaluation. D. Laboratory tests.

A. History of the oral lesion.

Microdontia occurs during what stages of dental development? A. Initiation. B. Histodifferentiation. C. Morphodifferentiation. D. Apposition. E. Mineralization.

A. Initiation.

What is the most likely diagnosis of a white diffuse lesion with a wrinkled appearance on the buccal mucosa which disappears upon stretching? A. Leukoedema. B. Lichen planus. C. Candidiasis. D. Linea alba. E. White sponge nevus

A. Leukoedema.

Which of the following root surfaces have concavities that make root planing difficult? A. Mesial of maxillary first premolars. B. Lingual of mandibular first premolars. C. Mesial of maxillary incisors. D. Distal of the palatal roots of maxillary molars.

A. Mesial of maxillary first premolars.

In an Angle's Class I occlusion, the cusp of which permanent tooth is in contact with the central fossa of the mandibular first molar? A. Mesiolingual cusp of the maxillary first molar. B. Distolingual cusp of the maxillary first molar. C. Mesiolingual cusp of the maxillary second molar. D. Distolingual cusp of the maxillary second molar.

A. Mesiolingual cusp of the maxillary first molar.

Which of the following variables has the greatest effect on bond strength? A. Moisture control. B. Size of the bracket base. C. Type of acid etch. D. Composition of the resin cement. E. Patient oral hygiene.

A. Moisture control.

If an infection from an abscessed permanent mandibular first molar perforates the lingual cortex, which muscle determines the space to which the infection will spread in the soft tissues? A. Mylohyoid. B. Temporalis. C. Digastric. D. Medial pterygoid.

A. Mylohyoid.

What is the most probable syndrome affecting a 9 year old patient with a history of 3 keratocystic odontogenic tumours (odontogenic keratocyst)? A. Nevoid basal cell carcinoma syndrome (Gorlin Syndrome). B. Familial colorectal polyposis (Gardner's Syndrome). C. Crouzon Syndrome. D. Apert Syndrome.

A. Nevoid basal cell carcinoma syndrome (Gorlin Syndrome).

Which maxillary central incisor characteristic is the most limiting in the construction of a Class II functional appliance? A. Retroclination. B. Proclination. C. Overeruption. D. Undereruption.

A. Retroclination.

Which of the following medications is most appropriate in the management of a patient experiencing an acute asthmatic attack? A. Salbutamol. B. Sodium cromoglycate. C. Fluticasone. D. Zileuton.

A. Salbutamol.

Immediately following an inferior alveolar nerve block, the patient exhibits facial paralysis. The needle has penetrated through which ligament? A. Sphenomandibular. B. Stylomandibular. C. Stylohyoid. D. Pterygomandibular.

A. Sphenomandibular.

Bacteria that are strongly associated with the onset of caries are A. Streptococcus mutans. B. Lactobacillus caseii. C. Veillonella alcalescens. D. Enterococcus faecalis. E. Actinomyces viscosus.

A. Streptococcus mutans.

Which of the following microorganisms are most frequently found in infected root canals? A. Streptococcus viridans. B. Staphylococcus aureus. C. Lactobacilli. D. Enterococci. E. Staphylococcus albus.

A. Streptococcus viridans.

Which of the following is an indication for the use of occlusal sealants? A. Teeth showing signs of opacity in pits or fissures. B. Teeth that have remained caries-free for 4 years and more. C. Teeth that have proximal cavitated lesions. D. Patient's water supply is non-fluoridated.

A. Teeth showing signs of opacity in pits or fissures.

Which of the following is classified as a muscle of mastication? A. Temporalis. B. Buccinator. C. Digastric. D. Mylohyoid

A. Temporalis.

Which of the following factors could cause a partial denture framework to fit tighter in the mouth than on the cast? A. Too much water in the mix of the stone for the cast. B. Not enough water in the mix of the stone for the cast. C. Duplication impression slightly oversized. D. Improper wax-up of the partial denture

A. Too much water in the mix of the stone for the cast.

What is the most common manifestation of occlusal trauma? A. Tooth mobility. B. Periodontal pocket formation. C. Gingival recession. D. Pulp calcifications.

A. Tooth mobility.

Which of the following procedures must be done to ensure acceptable mercury hygiene in a dental office? A. Use of high volume evacuation when working with amalgam. B. Use of air spray when condensing, polishing or removing amalgam. C. Storage of amalgam scrap in a dry container with a lid. D. A quarterly mercury assessment for office personnel.

A. Use of high volume evacuation when working with amalgam.

Which of the following posts are the most radiopaque? A. Zirconia. B. Titanium. C. Carbon fibre. D. Plastic.

A. Zirconia.

Sickle cell anemia is A. a genetic disease. B. caused by exposure to radiation. C. a viral infection. D. a drug reaction. E. an auto-immune disease.

A. a genetic disease.

Irreversible hydrocolloid materials are best removed from the mouth by A. a quick snap. B. a slow teasing motion. C. twisting and rocking. D. having the patient create a positive pressure.

A. a quick snap.

The initial appearance of a successful apicoectomy would show on a radiograph as A. a radiolucent area. B. woven bone. C. cortical bone around surgical site. D. sclerotic dentin.

A. a radiolucent area.

After setting, alginate impressions A. absorb water. B. remain dimensionally stable for 12 hours. C. have higher tear strength than polyvinylsiloxane impressions. D. can be poured twice with little effect on accuracy of the resulting cast.

A. absorb water.

Salbutamol is the most appropriate drug to manage A. acute asthma. B. angina pectoris. C. myocardial infarction. D. epilepsy. E. vasodepressor syncope.

A. acute asthma.

A patient that has been prescribed metronidazole should avoid A. alcohol. B. antacids. C. caffeine. D. cheese. E. grapefruit juice.

A. alcohol.

Lidocaine (Xylocaine®) is an example of a local anesthetic which is chemically classified as an A. amide. B. ester. C. aldehyde. D. ethamine. E. aminide.

A. amide.

In domestic violence cases, the highest risk for victim fatality is associated with A. an actual or pending separation. B. severe depression of the abuser. C. prior threats to kill the victim. D. a new partner in the victim's life. E. a history of abuse of the abuser as a child.

A. an actual or pending separation.

A 10 year old patient complains of discomfort in a maxillary primary second molar when eating. The tooth is mobile with a large mesio-occlusal amalgam restoration. The most likely diagnosis is A. an exfoliating tooth. B. a hyperemic pulp. C. a hyperplastic pulp. D. an acute pulpitis. E. traumatic occlusion

A. an exfoliating tooth.

An 8 year old patient presents 4 hours post-trauma with an oblique crown fracture of 2.1 exposing 2mm of vital pulp. The most appropriate pulpal treatment is A. apexogenesis. B. apexification. C. extraction. D. root canal treatment.

A. apexogenesis.

In an 8-year old, the optimum time for treatment of an anterior crossbite of dental origin is A. as soon as possible. B. after the maxillary canines are in position. C. after the maxillary premolars erupt. D. following completion of jaw growth.

A. as soon as possible.

The most appropriate time to remove a supernumerary tooth that is disturbing the eruption of a permanent tooth is A. as soon as possible. B. after ⅔ to ¾ of the permanent root has formed. C. after the apex of the permanent root has completely formed. D. after the crown appears calcified radiographically.

A. as soon as possible.

A patient had a coronary arterial stent placed 1 year ago following a myocardial infarction and has been asymptomatic since. The most appropriate management is to A. avoid using epinephrine impregnated retraction cord. B. use local anesthetics without epinephrine. C. perform only emergency dental treatment for 1 year. D. not recline the patient more than 45 degrees. E. prescribe antibiotics to prevent infective endocarditis.

A. avoid using epinephrine impregnated retraction cord.

Following orthodontic alignment, relapse of the mandibular incisors CANNOT A. be predicted from characteristics of the original malocclusion. B. occur if the second or third molars are removed. C. occur if retainers are worn until the mandibular growth is complete. D. occur if a supracrestal fiberotomy is performed.

A. be predicted from characteristics of the original malocclusion.

A patient on broad spectrum antibiotics for 4 weeks has widespread, sore, red and white oral mucosal lesions. The most likely diagnosis is A. candidiasis. B. leukoplakia. C. erythema multiforme. D. erosive lichen planus. E. pemphigoid.

A. candidiasis.

The bond between porcelain and metal in a ceramometal (porcelain bonded to metal) crown is A. chemical. B. mechanical. C. equally chemical and mechanical. D. neither chemical nor mechanical.

A. chemical.

When exposing radiographic film, the amount of radiation received by the patient is best reduced by A. collimation. B. decreased object-film distance. C. low kVp correlated with high milliamperage. D. decreased target-object distance.

A. collimation.

The major connector of a removable partial denture should be designed to A. connect rigidly the component parts of the partial denture. B. act as a stress-breaker. C. dissipate vertical forces.

A. connect rigidly the component parts of the partial denture.

The purpose of calibrating examiners in a randomized controlled clinical trial is to ensure A. consistent measurement of clinical outcomes. B. similarity of experimental and control groups. C. high ethical standards in conducting the trial. D. measurement of all relevant clinical outcomes.

A. consistent measurement of clinical outcomes.

A patient fails to demonstrate effective plaque control during initial periodontal therapy for moderate periodontitis. The most appropriate management is A. continued initial therapy. B. gingival curettage. C. gingivectomy. D. an apically positioned flap.

A. continued initial therapy.

maxillary central incisor that is erupting in a lingually directed path should be A. corrected before it reaches the occlusal plane. B. allowed to erupt until all incisors can be banded. C. allowed to erupt into cross-bite and then corrected. D. ignored because pressures of the tongue will correct it as it erupts. E. ignored because pressures of the lip will cause the problem to recur.

A. corrected before it reaches the occlusal plane

In the keyhole model of the enamel prism A. crystallites in the tail are angled compared to the head. B. the water is located at the prism edges. C. proteins are only present in the tail. D. crystallites have an identical molecular structure.

A. crystallites in the tail are angled compared to the head.

For a patient with chronic renal failure, A. dental procedures should be performed on nondialysis days. B. normal bleeding can be expected if the platelet count is normal. C. the arm used for vascular access during dialysis can be used to measure blood pressure. D. the gravity of the disease is estimated by albumine clearance.

A. dental procedures should be performed on nondialysis days.

During orthodontic treatment, a healthy adolescent will most frequently present with A. gingivitis. B. horizontal bone loss. C. necrotizing ulcerative gingivitis. D. vertical bone loss

A. gingivitis.

The most common form of periodontal disease is A. gingivitis. B. chronic periodontitis. C. gingival hyperplasia. D. aggressive periodontitis.

A. gingivitis.

Pernicious anemia may cause A. glossitis. B. lingual ulceration(s). C. parotid swelling. D. gastric hyperacidity.

A. glossitis.

In chewing, maximum contact between teeth occurs in the position of A. habitual occlusion. B. lateral excursion on the non-working side. C. protrusive excursion. D. All of the above

A. habitual occlusion.

Fixed partial denture ovate pontics should A. have a convexe surface touching the mucosa. B. have a concave surface touching the mucosa. C. have a flat surface touching the mucosa. D. hide the porcelain-metal junction on their gingival aspect.

A. have a convexe surface touching the mucosa.

The most common sensory change in the healthy elderly is a decrease in A. hearing. B. taste. C. touch perception. D. olfaction.

A. hearing.

Absence of occlusal contacts on a provisional restoration may result in the definitive restoration exhibiting A. heavy occlusal contact. B. no occlusal contact. C. tight proximal contacts. D. open proximal contacts.

A. heavy occlusal contact.

The microscopic appearance of the central giant cell granuloma of the jaws is similar to that of lesions which occur in A. hyperparathyroidism. B. Paget's disease. C. cleidocranial dysplasia. D. hyperpituitarism.

A. hyperparathyroidism.

Epinephrine should NOT be used as a vasoconstrictor for patients with uncontrolled A. hyperthyroidism. B. hyperparathyroidism. C. myxedema. D. asthma.

A. hyperthyroidism.

Chronic alcoholism causes impairment of liver function and A. increased bleeding time. B. increased coagulation time. C. decreased effectiveness of local anesthetics. D. increased risk of secondary infections. E. increased risk of hemolysis after antibiotic therapy.

A. increased bleeding time.

The occlusal parameter most useful to differentiate between an overbite of dental or skeletal origin is the A. mandibular curve of Spee. B. mandibular curve of Wilson. C. maxillary curve of Wilson. D. maxillary incisor morphology. E. maxillary lip length.

A. mandibular curve of Spee.

Following the removal of a vital pulp, the root canal is medicated and sealed. The patient returns with apical periodontitis. The most common cause is A. over-instrumentation. B. lateral perforation. C. pulp tissue left in the root canal. D. infection.

A. over-instrumentation.

A patient's 4 mandibular incisors were traumatized 3 years ago in an accident. Radiographs now show apical radiolucencies associated with all 4 teeth. The most appropriate management is to A. test the pulp vitality and perform root canal therapy on teeth with no response. B. perform root canal therapy on all 4 teeth and curette the periapical area. C. extract the teeth and place a bonded bridge. D. postpone treatment and recheck status periodically.

A. test the pulp vitality and perform root canal therapy on teeth with no response.

The base of a distal extension partial denture should cover the maximum support area because A. the force transmitted per unit area will be kept to a minimum. B. maximum number of artificial teeth can be placed. C. phonetics is improved. D. strength of the base is increased.

A. the force transmitted per unit area will be kept to a minimum.

In a child, correction of a bilateral posterior constriction of the maxillary arch has the WORST long term prognosis for stability if A. the maxillary posterior teeth are centred on the alveolar process. B. a quadhelix is used for treatment. C. there is a functional shift from initial contact to maximum intercuspation. D. there is a history of prolonged thumb sucking.

A. the maxillary posterior teeth are centred on the alveolar process.

In an edentulous maxilla, the direction of resorption of the alveolar ridge is A. upward and palatally. B. upward and facially. C. uniform in all directions. D. upward only.

A. upward and palatally.

block injection, the patient's face becomes quickly and visibly swollen. The immediate treatment should be to A. use pressure followed by cold packs over the swelling. B. use hot packs over the swelling. C. refer the patient to a hospital. D. administer 100mg hydrocortisone intravenously. E. administer diphenhydramine hydrochloride (Benadryl®) 50mg intravenously.

A. use pressure followed by cold packs over the swelling.

The characteristic oral lesion(s) of pemphigus is/are A. vesicles and bullae. B. Fordyce granules. C. white plaques. D. hairy tongue. E. candidiasis.

A. vesicles and bullae.

Which of the following has/have analgesic, antipyretic and anti-inflammatory effects? A. Acetominophen. B. Acetylsalicylic acid. C. Bradykinin. D. A. and B. E. None of the above.

B. Acetylsalicylic acid.

Which of the following medications is CONTRAINDICATED in the management of a patient who is taking warfarin? A. Acetaminophen. B. Acetylsalicylic acid. C. Codeine. D. Clindamycin

B. Acetylsalicylic acid.

The level of streptococcus mutans has been shown to be significantly higher in the bacterial plaque adjacent to which type of posterior restoration? A. Glass-ionomer. B. Composite resin. C. Amalgam. D. Gold castings.

B. Composite resin.

What is the most appropriate appliance to correct an Angle Class I malocclusion with a labially tipped maxillary central incisor and spacing in a 15 year old patient? A. Growth modification appliance. B. Hawley with an active labial bow. C. Molar distalizing appliance. D. Bilateral expansion appliance.

B. Hawley with an active labial bow.

Which of the following has a direct effect on periodontal tissues in diabetes mellitus-associated gingivitis? A. Lipopolysaccharides. B. Matrix metalloproteinases. C. Proinflammatory cytokines. D. Prostaglandins. E. Leukotoxin.

B. Matrix metalloproteinases.

Which of the following teeth are most often lost as a result of periodontitis? A. Mandibular molars. B. Maxillary molars. C. Mandibular incisors. D. Maxillary incisors.

B. Maxillary molars.

The mesial furcation of the permanent maxillary first molar is best assessed from which aspect of the tooth? A. Mesiobuccal. B. Mesiopalatal. C. Midmesial.

B. Mesiopalatal.

Which of the following drugs is indicated for the management of a Gram-negative anaerobic infection? A. Clotrimazole. B. Metronidazole. C. Omeprazole. D. Sulfamethoxazole.

B. Metronidazole.

In the absence of "Hanks balanced salt solution", what is the most appropriate media to transport an avulsed tooth? A. Saliva. B. Milk. C. Saline. D. Tap water.

B. Milk.

Which of the following does NOT describe the energy of x-ray photons exiting the x-ray unit? A. Normally distributed. B. Mono-energetic. C. Proportional to frequency. D. Inversely proportional to wavelength.

B. Mono-energetic.

A patient had a myocardial infarction 6 months ago. Which of the following NSAIDs is most appropriate for the patient? A. Ibuprofen. B. Naproxen. C. Celecoxib. D. Ketolorac.

B. Naproxen.

Which of the following conditions has the WORST prognosis for a furcation involved tooth? A. Wide root separation. B. Narrow root separation. C. A bifurcation ridge. D. A cemento-enamel projection

B. Narrow root separation.

Which syndrome has multiple cysts of the jaws? A. Gardner syndrome. B. Nevoid basal cell carcinoma syndrome. C. Peutz-Jeghers syndrome. D. Sjögren syndrome.

B. Nevoid basal cell carcinoma syndrome.

Which of the following drugs can interfere with the effectiveness of oral contraceptives? A. Codeine. B. Penicillin V. C. Acetaminophen. D. Magnesium trisilicate. E. None of the above.

B. Penicillin V.

Which of the following drugs potentiates the action of sedative drugs? A. Digitalis. B. Phenothiazine. C. Propranolol. D. Methyldopa. E. Spironolactone.

B. Phenothiazine.

Which of the following does NOT improve the retention of a Class II inlay? A. Adding an occlusal dovetail. B. Placing a gingival bevel. C. Increasing the parallelism of walls. D. Lengthening the axial walls.

B. Placing a gingival bevel.

Which cells migrate into the gingival sulcus in the largest numbers in response to the accumulation of plaque? A. Plasma cells and monocytes. B. Polymorphonuclear leukocytes. C. Macrophages. D. Lymphocytes. E. Mast cells

B. Polymorphonuclear leukocytes.

In the mixed dentition, which of the following conditions should be treated immediately? A. A maxillary midline diastema. B. Posterior cross-bite with a midline discrepancy. C. Crowding of maxillary and mandibular incisors. D. An end-to-end molar relationship.

B. Posterior cross-bite with a midline discrepancy.

After a tooth has had nonsurgical endodontic treatment, which of the following is NOT an indication for additional endodontic intervention? A. Presence of clinical signs and symptoms. B. Presence of a periradicular radiolucency. C. Enlargement of the original periradicular radiolucency. D. Development of a periradicular radiolucency.

B. Presence of a periradicular radiolucency.

A 3 year old patient complains of pain of the mouth and headaches for 24 hours. The clinical exam reveals bilateral painful regional lymphadenopathy and bad breath. Vesicles are seen on the hard palate, the soft palate, the gingiva, the tongue and the lips. The patient's temperature is 38.5ºC. What is the most probable diagnosis? A. Erythema multiforme. B. Primary herpetic gingivostomatitis. C. Hand-foot-mouth disease. D. Herpetiform aphtaus ulcers.

B. Primary herpetic gingivostomatitis.

Which of the following is contagious? A. Pemphigus. B. Primary herpetic gingivostomatitis. C. Recurrent aphthous stomatitis. D. Necrotizing ulcerative gingivitis.

B. Primary herpetic gingivostomatitis.

A group of researchers would like to study the relationship between oral health status in childhood and adult systemic health conditions. They have found a 40 year old database that contains information on the oral health of children along with their names. They plan on linking this information with current administrative health care data to conduct this research. What ethical concern(s) would the researchers have to consider? A. Selection bias. B. Privacy and confidentiality. C. Beneficence. D. Justice.

B. Privacy and confidentiality.

Which of the following Class II Division 1 malocclusion(s) is/are most likely to be corrected with a cervical headgear? A. Retrognathic mandible, retrognathic maxilla, open bite. B. Prognathic maxilla, decreased lower face height, increased over bite. C. Increased lower anterior face height, prognathic mandible, retrognathic maxilla. D. Open bite, prognathic maxilla, prognathic mandible.

B. Prognathic maxilla, decreased lower face height, increased over bite.

Which of the following require prophylactic antibiotics prior to dental procedures causing a bacteremia? A. Implanted cardiac pacemakers. B. Prosthetic cardiac valves. C. Coronary artery bypass grafts. D. Cardiac stents one year after placement.

B. Prosthetic cardiac valves.

In Canada, the practice of dentistry is regulated through the A. Canadian Dental Association. B. Provincial Regulatory Authorities. C. National Dental Examining Board of Canada. D. Commision on Dental Accreditation.

B. Provincial Regulatory Authorities.

A 7 year old patient presents with a crown fracture of a permanent maxillary central incisor that occurred 3 days ago. The incisal half of the crown is missing, resulting in a 2mm exposure of vital pulp. What is the most appropriate initial management for this tooth? A. Apexification. B. Pulpotomy. C. Pulp capping. D. Pulpectomy.

B. Pulpotomy.

A 7 year old patient has a normal occlusion except that the 2 primary maxillary central incisors have recently exfoliated, creating a tongue thrust. What is the most appropriate management for this patient? A. Immediately refer the patient to a speech therapist. B. Re-evaluate following the eruption of the permanent maxillary central incisors. C. Radiographically monitor the eruption of permanent maxillary central incisors. D. Prescribe a tongue crib appliance. E. Prescribe an appliance to temporarily replace the missing incisors.

B. Re-evaluate following the eruption of the permanent maxillary central incisors.

Which of the following modifications is recommended when making radiographs on an 8 year old child compared to an adult? A. Increase the kVp. B. Reduce the exposure time. C. Increase the mA. D. Use a lower speed receptor.

B. Reduce the exposure time.

An adult female patient presents to the dental office with fractured teeth, a lacerated lip and a black eye. She is with her 6 year old son, who is crying and upset. The dentist's receptionist discretely reports that the child was asking his mother "Why was Daddy hurting you?" Upon questioning, the patient confides that her husband was the source of her injuries and indicates that she would not be pursuing any action. She is referred to a specialist due to the complexity of her dental injuries, but she requests that the source of her injuries not be disclosed. What is the dentist's obligation? A. Respect the patient's request regarding confidentiality. B. Report her injuries to an adult protection agency. C. Forward all information, including the source of the injuries, to the specialist. D. Report the situation to a relevant child protection agency.

B. Report her injuries to an adult protection agency.

A 5 year old has an enamel fracture of tooth 6.1. What is the most appropriate immediate management? A. Pulpotomy. B. Smooth sharp edges. C. Pulpectomy. D. Extraction.

B. Smooth sharp edges.

Which of the following is a characteristic of aggressive periodontitis? A. Amount of local factors is consistent with disease severity. B. Specific periodontal microbial etiology. C. A manifestation of a systemic disease. D. Ulcerations of the gingiva.

B. Specific periodontal microbial etiology.

What should be the immediate management of an acute anginal episode? A. Oral ibuprofen. B. Sublingual nitroglycerin. C. Subcutaneous epinephrine. D. Inhaled salbutamol.

B. Sublingual nitroglycerin.

Gold casting alloys are classified as Type I-IV according to which of the following physical properties? A. Percentage of gold present in the alloy. B. Surface hardness. C. Melting point. D. Elastic strength. E. Ductility.

B. Surface hardness.

Which of the following is the most likely cause of osteoporosis, glaucoma, hypertension and peptic ulcers in a 65 year old with Crohn's disease? A. Uncontrolled diabetes. B. Systemic corticosteroid therapy. C. Chronic renal failure. D. Prolonged NSAID therapy. E. Malabsorption syndrome.

B. Systemic corticosteroid therapy.

It is advisable to polish any restorative material as smoothly as possible in order to prevent A. microleakage. B. accumulation of plaque. C. overhanging margins. D. electro-chemical action.

B. accumulation of plaque.

Diabetes mellitus is the result of A. hypersecretion of the posterior pituitary. B. atrophy of the islands of Langerhans. C. destruction of the adrenal cortex. D. destruction of the posterior pituitary or associated hypothalamic centres.

B. atrophy of the islands of Langerhans.

The most appropriate management for a longstanding, asymptomatic, well demarcated, adherent, white plaque on the ventral tongue of a 66 year old non-smoker is A. topical antifungal treatment. B. biopsy and long-term clinical follow-up. C. re-evaluate in 2 weeks. D. refer to their physician.

B. biopsy and long-term clinical follow-up.

Duraflor® A. is only effective on dry, plaque-free teeth. B. can remineralize early root carious lesions. C. causes unsightly stain on exposed roots. D. should only be used on individuals in unfluoridated areas.

B. can remineralize early root carious lesions.

Hyperkeratosis, acanthosis, dysplasia, increased mitosis, intact basal cell layer and chronic inflammatory cells are histologic features that may be found in A. squamous cell carcinoma. B. carcinoma in situ. C. papillofibroma. D. endothelioma.

B. carcinoma in situ.

Which of the following statements is true about diazepam? It A. improves performance of fine motor skills. B. causes amnesia. C. is available without prescription in Canada. D. does not produce a dependence syndrome

B. causes amnesia.

Enamel pearls form when A. ameloblasts migrate apically down the root. B. cells of the epithelial root sheath do not migrate away from the dentin. C. cells of the dental follicle fail to develop. D. epithelial rests transform into ameloblast vesicles.

B. cells of the epithelial root sheath do not migrate away from the dentin.

Hypothyroidism affects the dental developmental pattern by A. interfering with jaw growth. B. delaying the eruption timetable. C. causing sclerotic bone to form over the occlusal surface of erupting teeth. D. accelerating the eruption timetable.

B. delaying the eruption timetable.

In the mandibular third molar region, a circumscribed radiolucent area 3cm in diameter contains the crown of the developing third molar. The radiolucent area suggests a/an A. simple bone cyst/traumatic bone cyst. B. dentigerous cyst. C. Stafne bone defect/static bone cavity. D. eruption cyst.

B. dentigerous cyst.

The most significant factor in determining the prognosis of complete anterior crossbite correction is the A. age of patient. B. depth of the overbite. C. amount of crowding of the mandibular arch. D. amount of crowding of the maxillary arch.

B. depth of the overbite.

Water irrigation devices have been shown to A. eliminate plaque. B. dislodge food particles from between teeth. C. disinfect pockets for up to 18 hours. D. prevent calculus formation.

B. dislodge food particles from between teeth.

Strain hardening a metal will reduce its A. modulus of elasticity. B. ductility. C. proportional limit. D. yield strength.

B. ductility.

Rigor mortis is A. due to intracellular Ca2+ ion depletion. B. due to cellular ATP depletion. C. mechanistically identical to a tetanic contraction. D. characterized by spastic paralysis. E. an isotonic contraction.

B. due to cellular ATP depletion.

Multiple congenitally missing teeth may be characteristic of A. cleidocranial dysplasia. B. ectodermal dysplasia. C. Gardner's syndrome. D. Gorlin-Goltz's syndrome

B. ectodermal dysplasia.

Swelling related to increased tissue fluid is called A. thrombosis. B. edema. C. hematoma. D. embolism. E. surgical emphysema.

B. edema.

Maximum shrinkage after gingival curettage can be expected from tissue that is A. fibroedematous. B. edematous. C. fibrotic. D. formed within an infrabony pocket. E. associated with exudate formation.

B. edematous.

Procaine (Novocaine®) is an example of a local anesthetic which is chemically classified as an A. amide. B. ester. C. aldehyde. D. ethamine. E. aminide.

B. ester.

A patient with complete dentures complains of clicking. The most common causes are A. reduced vertical dimension and improperly balanced occlusion. B. excessive vertical dimension and poor retention. C. use of too large a posterior tooth and too little horizontal overlap. D. improper relation of teeth to the ridge and excessive anterior vertical overlap.

B. excessive vertical dimension and poor retention.

Cleft lip and palate usually result from A. failure of proper union of the median and lateral nasal processes. B. failure of the union of the median nasal process with the lateral nasal and maxillary processes. C. anhidrotic ectodermal dysplasia. D. failure of development of both the lateral nasal and maxillary processes.

B. failure of the union of the median nasal process with the lateral nasal and maxillary processes.

On T1-weighted magnetic resonance images, the tissue that has the highest intensity is A. blood. B. fat. C. skeletal muscle. D. cortical bone.

B. fat.

The periodontal ligament is constantly remodeled due to the activity of A. osteoclasts. B. fibroblasts. C. macrophages. D. mesenchymal cells.

B. fibroblasts.

The most common site of intraoral squamous cell carcinoma is the A. palate. B. floor of the mouth. C. gingiva. D. buccal mucosa.

B. floor of the mouth.

Pyogenic granuloma is most frequently found on the A. tongue. B. gingiva. C. buccal mucosa. D. tonsillar pillars. E. lips.

B. gingiva.

Compared to a set of opposing complete dentures, a maxillary denture opposing a full complement of natural teeth is more often associated with A. less denture tooth wear. B. greater incidence of denture fracture. C. improved retention of the denture. D. improved stability of the denture. E. improved appearance of the denture.

B. greater incidence of denture fracture.

A Class III malocclusion is normally associated with A. sleeping habits. B. growth discrepancy. C. tooth size - jaw size discrepancy. D. trauma.

B. growth discrepancy.

A 6 year old has circumscribed suppurative lesions that look like cigarette burns on the left ear lobe and the right knuckles. The parents are cooperative and respond to the dentist in a coherent manner. The most likely diagnosis is A. battered child syndrome. B. impetigo. C. osteogenesis imperfecta. D. Münchhausen syndrome. E. von Willebrand desease.

B. impetigo.

Occlusal trauma can A. initiate periodontitis. B. increase tooth mobility. C. exacerbate gingival hyperplasia. D. cause gingival recession.

B. increase tooth mobility.

significant mechanism by which acetylsalicylic acid produces its analgesic and anti-inflammatory effect is A. antagonism of histamine. B. inhibition of prostaglandin synthesis. C. local anesthetic effect on pain fibres. D. release of adrenal steroids from the adrenal cortex. E. synaptic inhibition in the dorsal column.

B. inhibition of prostaglandin synthesis.

Local anesthetics A. do not readily pass the blood-brain barrier. B. interfere with the propagation of action potentials in nerve fibres. C. selectively interfere with the propagation of action potentials in nociceptive fibres. D. do not have an effect on any other tissue than the nervous tissue.

B. interfere with the propagation of action potentials in nerve fibres.

Clinical examination of a 42 year old heavy smoker reveals a white patch in the tonsillar pillar region. The patch cannot be wiped off. The most likely diagnosis is A. lichen planus. B. leukoplakia. C. white sponge nevus. D. frictional hyperkeratosis. E. pseudomembranous candidiasis.

B. leukoplakia.

The permanent anterior tooth that exhibits the greatest variation in size and shape is the A. maxillary central incisor. B. maxillary lateral incisor. C. mandibular central incisor. D. mandibular lateral incisor.

B. maxillary lateral incisor.

The efficacy of pit and fissure sealants is affected by A. occlusal relationship. B. opacity of the sealant. C. stage of tooth eruption. D. type of polymerization reaction. E. systemic fluoride treatment.

B. opacity of the sealant.

Blue sclera is characteristic of A. osteopetrosis. B. osteogenesis imperfecta. C. osteitis deformans. D. fibrous dysplasia.

B. osteogenesis imperfecta.

The most common clinical finding in the diagnosis of an acute apical abscess is A. mobility of the tooth. B. pain on percussion. C. discoloration of the crown. D. presence of a cellulitis. E. lymph node enlargement. A

B. pain on percussion.

A large carious exposure occurs on a permanent first molar of a 7 year old. There is no periapical involvement and the tooth is vital. The treatment should be to A. cap the exposure with calcium hydroxide and place zinc-oxide and eugenol. B. perform a pulpotomy and place calcium hydroxide. C. perform a pulpectomy. D. extract the tooth and place a space maintainer.

B. perform a pulpotomy and place calcium hydroxide.

During guided tissue regeneration therapy, the regenerative cells originate primarily from the A. lamina propria. B. periodontal ligament. C. cellular cementum. D. collagen membrane.

B. periodontal ligament.

Acute or subacute suppurative osteomyelitis occurs most frequently in the A. anterior maxilla. B. posterior mandible. C. posterior maxilla. D. anterior mandible.

B. posterior mandible.

Biocompatibility tests conducted in vitro A. follow screening with an animal model. B. predict human cell growth or death. C. reproduce the in vivo environment. D. are more easily standardized than clinical studies.

B. predict human cell growth or death.

The purpose of phenidone in radiographic developing solution is to chemically A. oxidize silver halide to metallic silver in the emulsion. B. reduce silver halide to metallic silver in the emulsion. C. remove silver halide from the emulsion that has been exposed to radiation. D. remove silver halide from the emulsion that has not been exposed to radiation.

B. reduce silver halide to metallic silver in the emulsion.

Upon examination of an edentulous patient, it is observed that the tuberosities contact the retromolar pads at the correct occlusal vertical dimension. The treatment of choice is to A. reduce the retromolar pads surgically to provide the necessary clearance. B. reduce the tuberosities surgically to provide the necessary clearance. C. construct new dentures at an increased occlusal vertical dimension to gain the necessary clearance. D. proceed with construction of the denture and reduce the posterior extension of the mandibular denture to eliminate interferences

B. reduce the tuberosities surgically to provide the necessary clearance.

If a patient in her first trimester of pregnancy requires the replacement of a large MOD amalgam restoration with extensive recurrent caries and thermal sensitivity, the most appropriate treatment is to A. delay treatment until after the baby is born. B. restore with reinforced zinc oxide eugenol. C. restore with amalgam. D. restore with a composite resin.

B. restore with reinforced zinc oxide eugenol.

Two weeks following the placement of a restoration, a patient complains of pain to hot and cold in the restored tooth. The most likely diagnosis is A. galvanic shock. B. reversible pulpitis. C. gingival irritation. D. cracked tooth syndrome.

B. reversible pulpitis.

Bonding composite to enamel is most appropriately performed by isolation with A. cotton rolls. B. rubber dam. C. cheek retractors. D. a matrix system.

B. rubber dam.

The most appropriate method to diagnose a cystic tumour is to A. examine the fluid under a microscope. B. submit the tissue for histological analysis. C. perform a cytologic smear. D. culture the fluid. E. order blood tests.

B. submit the tissue for histological analysis.

A patient reports pain on mastication since the placement of a metal-ceramic crown 2 weeks earlier. The most likely cause is A. hyperemia. B. supraocclusion. C. dentin hypersensitivity. D. acute pulpitis.

B. supraocclusion.

Moistened dentin is prefrerred over dry dentin prior to the application of dentin bonding agents because A. curing time is reduced. B. the collagen matrix is maintained. C. the smear layer is removed by the application of water. D. enamel bonding is improved

B. the collagen matrix is maintained.

An ameloblastoma is most frequently found in A. the anterior region of the maxilla. B. the mandible, near the junction of the body and the ramus. C. the posterior region of the maxilla. D. in the anterior region of the mandible near the midline.

B. the mandible, near the junction of the body and the ramus.

With excessive forward mandibular growth in a patient with minimal overjet, the mandibular incisors will most likely A. tip labially. B. tip lingually. C. tip laterally. D. remain unchanged.

B. tip lingually.

Gingivectomy is recommended A. when the bottom of the pocket is apical to the mucogingival junction. B. to eliminate the suprabony pockets when the pocket wall is fibrous and firm. C. to treat moderately deep pockets with mild intrabony defects.

B. to eliminate the suprabony pockets when the pocket wall is fibrous and firm.

"Tic douloureux" is synonymous with A. psychogenic facial pain. B. trigeminal neuralgia. C. facial paralysis. D. temporomandibular joint dysfunction.

B. trigeminal neuralgia.

Tooth 4.5 was treated endodontically 2 years ago and is now sensitive to percussion. There is an isolated, narrow 8mm periodontal pocket on the buccal surface of the tooth. Radiographic findings are within normal limits. The most likely diagnosis is a/an A. endodontic lesion. B. vertical root fracture. C. chronic periodontal lesion.

B. vertical root fracture.

What si the most appropriate orthodontic tooth movement to ensure stability of open bite correction? A. Extrusion of the upper incisors. B. Intrusion of the upper incisors. C. Instrusion of the upper molars. D. Extrusion of the upper molars.

C. Instrusion of the upper molars.

Which of the following is NOT a function of the wedge in the restoration of a Class II cavity with amalgam? A. It separates the teeth to allow restoration of the contact. B. It assists in the adaptation of the matrix band to the proximal portion of the preparation. C. It absorbs moisture from the cavity preparation, allowing the restoration to be placed in a dry field. D. It provides stability to the matrix band and retainer assembly.

C. It absorbs moisture from the cavity preparation, allowing the restoration to be placed in a dry field.

What is the most appropriate time to extract a mesiodens? A. As soon as diagnosed. B. Just prior to the eruption of the first permanent molars. C. Just prior to the eruption of the maxillary central incisors. D. Just prior to the eruption of the maxillary canines.

C. Just prior to the eruption of the maxillary central incisors.

Which permanent tooth usually erupts first? A. Maxillary central incisor. B. Maxillary canine. C. Mandibular first molar. D. Mandibular central incisor.

C. Mandibular first molar.

Which foramen presents as an apical radiolucency in the mandibular premolar region? A. Lingual. B. Mandibular. C. Mental. D. Incisive.

C. Mental.

A 13 year old complains of red, bleeding and swollen gums. Clinical examination reveals this is present only on the labial gingiva of the maxillary anterior teeth. What is the most likely etiologic factor? A. Blood dyscrasia. B. Insulin-dependent diabetes mellitus. C. Mouth breathing habit. D. Pubertal hormones.

C. Mouth breathing habit.

Which of the following is NOT a property of lidocaine? A. Local anesthesia. B. Topical anesthesia. C. Muscle relaxation. D. Antiarrhythmic action.

C. Muscle relaxation.

Which of the following is consistent with a diagnosis of pulpal necrosis? A. Poorly localized spontaneous pain. B. Positive response to thermal tests. C. Negative response to electric pulp test. D. Extreme pain elicited by palpation and percussion tests.

C. Negative response to electric pulp test.

If a tooth has an inadequate ferrule, which of the following is an effective strategy to increase tooth structure available for crown preparation? A. Cementation of the restoration with a glass ionomer cement. B. Sub-gingival preparation and prolonged temporization. C. Orthodontic eruption. D. Elective endodontic treatment and a post core.

C. Orthodontic eruption.

Before performing surgery on a patient who is taking warfarin, which of the following should be evaluated? A. Bleeding time. B. Clotting time. C. Prothrombin time. D. Coagulation time.

C. Prothrombin time.

Which of the following changes in colour, contour and texture are indicative of plaque induced gingivitis? A. Red, swollen, increased stippling. B. Cyanotic, cleft formation, lack of stippling. C. Red, swollen, lack of stippling. D. Pink, swollen, lack of stippling.

C. Red, swollen, lack of stippling.

An extreme overjet, a recessive chin and a deep labial mento-labial sulcus are common findings in which facial type? A. Prognathic. B. Orthognathic. C. Retrognathic.

C. Retrognathic.

Caries in older persons is most frequently found on which of the following locations? A. Pits and fissures. B. Proximal enamel. C. Root surfaces. D. Incisal dentin.

C. Root surfaces.

Which substance found in radiograph processing solutions is of most concern environmentally? A. Ammonium thiosulfate. B. Acetic acid. C. Silver. D. Sodium sulfite.

C. Silver.

Hyperplastic lingual tonsils may resemble which of the following? A. Epulis fissuratum. B. Lingual varicosities. C. Squamous cell carcinoma. D. Median rhomboid glossitis. E. Prominent fungiform papillae

C. Squamous cell carcinoma.

Hyperplastic lingual tonsils may resemble which of the following? A. Epulis fissuratum. B. Lingual varicosities. C. Squamous cell carcinoma. D. Median rhomboid glossitis. E. Prominent fungiform papillae.

C. Squamous cell carcinoma.

A 70 year old patient was diagnosed with squamous cell carcinoma of the lateral border of the tongue. The tumour measures 3.5 x 3.0 cm. A CT of the neck confirms the presence of a 2.5cm ipsilateral submandibular lymph node suggestive of locoregional disease. Chest X-rays and liver function tests were within normal limits. The TNM classification of this patient's disease is A. T1N1M0. B. T1N2M0. C. T2N1M0. D. T2N2M1.

C. T2N1M0.

Which of the following is the most frequent cause of ankylosis of the temporomandibular joint? A. Intra-articular injection of steroids. B. Chronic subluxation. C. Trauma. D. Anterior disc dislocation.

C. Trauma.

A patient with a tumor in the right infratemporal fossa shows a significant shift of the mandible to the right when opening. Which nerve is involved? A. Facial nerve VII. B. Glossopharyngeal nerve IX. C. Trigeminal nerve V. D. Hypoglossal nerve XII.

C. Trigeminal nerve V.

Most of the somatosensory information from the oral cavity reaches the brain through which nerve? A. Trochlear. B. Hypoglossal. C. Trigeminal. D. Glossopharyngeal.

C. Trigeminal.

For which of the following teeth is the risk of root fracture increased if a rotational force is used during extraction? A. Upper canine. B. Lower canine. C. Upper first bicuspid. D. Lower first bicuspid. E. Upper lateral incisor.

C. Upper first bicuspid.

When taking radiographs on a 10 year old patient, which of the following should be used to decrease radiation exposure? A. Prescribe radiographs every 3 years. B. Decrease the kilovoltage to 50kVp. C. Use of high speed film. D. Use of a lead apron.

C. Use of high speed film.

Multiple neurofibromatosis and "café au lait" spots on the skin are typical of A. Gardner's syndrome. B. Plummer-Vinson syndrome. C. Von Recklinghausen's disease. D. Down syndrome.

C. Von Recklinghausen's disease.

The most common barrier preventing elderly individuals from seeking dental treatment is A. the cost associated with dental treatments. B. difficulty getting to and from the dental office. C. a lack of perceived need for treatment. D. fear of being "hurt".

C. a lack of perceived need for treatment.

On a semi-adjustable articulator, the incisal guide table represents A. a reference point for the establishment of occlusal vertical dimension. B. the anterior equivalent of the condylar guidance. C. a mechanical equivalent of the horizontal and vertical overlap of the anterior teeth. D. the mechanical equivalent of the Curve of Wilson.

C. a mechanical equivalent of the horizontal and vertical overlap of the anterior teeth.

The histopathologic changes in chronic gingivitis are characterized by A. loss of rete pegs and destruction of the basement membrane. B. hyalinization of the principal fibres of the periodontal ligament. C. an inflammatory infiltrate of plasma cells and lymphocytes. D. an inflammatory infiltrate in which polymorphonuclear cells predominate.

C. an inflammatory infiltrate of plasma cells and lymphocytes.

Angle's classification of occlusion is based on A. a full complement of teeth. B. antero-posterior skeletal relationship of maxilla to mandible. C. antero-posterior relationship of maxillary and mandibular first permanent molars. D. vertical relationships in the lower face.

C. antero-posterior relationship of maxillary and mandibular first permanent molars.

Histologically, periodontitis is distinguished from gingivitis by A. increased collagen destruction. B. increased gingival inflammation. C. apical migration of dentogingival epithelium

C. apical migration of dentogingival epithelium

Accessory canals in permanent teeth are most commonly found in the A. cervical third of the root. B. middle third of the root. C. apical third of the root.

C. apical third of the root.

In an infrabony periodontal pocket, the tip of the periodontal probe is located A. at the level of the alveolar crest. B. coronal to the level of the alveolar crest. C. apical to the level of the alveolar crest.

C. apical to the level of the alveolar crest.

The most appropriate management of a noncavitated, smooth surface carious lesion is A. placing an amalgam restoration. B. placing a composite restoration. C. applying topical fluoride. D. prescribing a chlorhexidine rinse. E. observation.

C. applying topical fluoride.

The line drawn through the occlusal rests of two principal abutments is A. survey line. B. terminal line. C. axis of rotation/fulcrum line. D. line of greatest torque.

C. axis of rotation/fulcrum line.

When using the periodontal probe to measure pocket depth, the measurement is taken from the A. base of the pocket to the cementoenamel junction. B. free gingival margin to the cementoenamel junction. C. base of the pocket to the crest of the free gingiva. D. base of the pocket to the mucogingival junction.

C. base of the pocket to the crest of the free gingiva.

Aging pulps show a relative increase in A. sensitivity. B. cell numbers. C. calcification. D. vascularity.

C. calcification.

Hemangiomas of the jaws A. never occur in bone. B. are malignant. C. can appear cystic radiographically. D. are metastatic lesions.

C. can appear cystic radiographically.

Patients with a history of ankle swelling, shortness of breath and orthopnea are most likely suffering from A. asthma. B. emphysema. C. congestive heart failure. D. constrictive pericarditis.

C. congestive heart failure.

A laboratory-fabricated composite resin inlay compared to a direct composite resin restoration has increased A. colour stability. B. surface smoothness. C. control of polymerization shrinkage. D. bondability to tooth structure.

C. control of polymerization shrinkage.

The greater palatine foramen is most likely to be radiographically misdiagnosed as a A. rarefying osteitis. B. nasolabial cyst. C. cyst of the incisive papilla.

C. cyst of the incisive papilla. Heated

A mucocele results from A. aplasia of the duct. B. hyperplasia of the duct. C. damage to the duct. D. hypersecretion.

C. damage to the duct.

Using more water when mixing dental stone will result in a cast that exhibits A. increased expansion and decreased strength. B. decreased expansion and increased strength. C. decreased expansion and decreased strength. D. increased expansion and increased strength.

C. decreased expansion and decreased strength.

After many caries free years, a 78 year old patient develops multiple root surface caries. This is most likely the result of A. changes in cementum composition. B. exposure of the cementoenamel junctions. C. decreased salivary flow. D. changes in dietary pattern.

C. decreased salivary flow.

The most important advantage of using reinforced zinc oxide eugenol cement as a temporary restoration is that it A. stimulates dentin repair. B. occludes dentinal tubules. C. desensitizes the pulp. D. chelates to tooth structure.

C. desensitizes the pulp.

Conversion of a flush terminal plane to a mesial step/Class I terminal plane in the absence of orthodontics is primarily the result of A. loss of the mandibular primate space. B. greater maxillary than mandibular forward growth. C. differences in leeway between the maxillary and mandibular arches. D. distal movement of the maxillary first permanent molars.

C. differences in leeway between the maxillary and mandibular arches.

The term used to describe epithelial changes including nuclear hyperchromatism, alteration of nuclear/cytoplasmic ratio and abnormal mitoses is A. acanthosis. B. hyperparakeratosis. C. dysplasia. D. acantholysis.

C. dysplasia.

The term used to describe epithelial changes including nuclear hyperchromatism, decreased nuclear-cytoplasmic ratio and abnormal mitoses is A. acanthosis. B. hyperkeratosis. C. dysplasia. D. parakeratosis. E. hyperparakeratosis.

C. dysplasia.

A patient presents with hypodontia, conical teeth, fine, scanty, fair hair, and an intolerance to hot weather. The most likely diagnosis is A. achondroplasia. B. malignant hyperthermia. C. ectodermal dysplasia. D. cystic fibrosis.

C. ectodermal dysplasia.

In patients wearing complete dentures, the most frequent cause of tooth contact (clicking) during speaking is A. nervous tension. B. incorrect centric relation position. C. excessive occlusal vertical dimension. D. lack of vertical overlap. E. unbalanced occlusion

C. excessive occlusal vertical dimension.

The radiographs of a 9 year old with tooth 1.1 completely erupted and tooth 2.1 unerupted reveal a palatally located mesiodens. The most appropriate management is to A. monitor the eruption of 2.1 for another year. B. uncover the mesiodens, wait for eruption and then extract it. C. extract the mesiodens and allow passive eruption of 2.1. D. extract the mesiodens and orthodontically extrude 2.1.

C. extract the mesiodens and allow passive eruption of 2.1.

A patient on anticoagulant drugs who requires an extraction has a prothrombin time of 20 seconds. The normal value is 15 seconds. The most appropriate management is to A. administer vitamin K after the extraction. B. administer vitamin K before the extraction. C. extract the tooth and use local measures to control bleeding. D. discontinue anticoagulation drugs one week before extraction.

C. extract the tooth and use local measures to control bleeding.

During radiographic film processing, silver halide is removed from the emulsion during the A. developing stage. B. post-developing rinse stage. C. fixing stage. D. post-fixing wash stage.

C. fixing stage.

A simple bone cyst/traumatic bone cyst is a radiolucency most frequently seen A. in the mandibular ramus. B. posteriorly to the maxillary molars. C. from the symphysis to the ramus of the mandible. D. in the maxillary premolar area.

C. from the symphysis to the ramus of the mandible

A therapeutic advantage of penicillin V over penicillin G is A. greater resistance to penicillinase. B. broader antibacterial spectrum. C. greater absorption when given orally. D. slower renal excretion. E. None of the above.

C. greater absorption when given orally.

A cold stimulus applied to a tooth will produce a hypersensitive response if the tooth A. is nonvital. B. has a periodontal pocket. C. has a hyperemic pulp. D. has chronic proliferative pulpitis.

C. has a hyperemic pulp.

Periapical osseous dysplasia (periapical cementoosseous dysplasia) A. is associated with a vital tooth. B. is found mainly in children. C. has an orange peel radiographic pattern.

C. has an orange peel radiographic pattern.

Warfarin (Coumadin®) acts by A. preventing formation of thromboplastin. B. preventing fibrinogen conversion to fibrin. C. inhibiting the synthesis of prothrombin in the liver. D. incorporating ionic calcium.

C. inhibiting the synthesis of prothrombin in the liver

The most appropriate treatment following the extraction of a first primary molar in a 4 year old child is A. regular assessment of arch development. B. to perform space analysis. C. insertion of a space maintainer. D. extraction of the contra-lateral molar. E. extraction of the opposing molar.

C. insertion of a space maintainer.

A positive and prolonged reaction to a heat stimulus indicates that the pulp is A. normal. B. reversibly inflamed. C. irreversibly inflamed. D. necrotic.

C. irreversibly inflamed.

The most appropriate method to prevent root canal obstruction during the instrumentation phase of endodontic treatment is to A. obtain adequate access. B. use a chelating agent. C. irrigate copiously. D. use reamers instead of files

C. irrigate copiously.

An oroantral communication occurs A. more often with the removal of maxillary second premolars. B. less often when the maxillary sinus has pneumatized into the alveolus. C. more often with the removal of maxillary first molars. D. less often in elderly patients.

C. more often with the removal of maxillary first molars.

The most appropriate radiograph(s) to determine the location of an impacted maxillary cuspid is/are A. occlusal. B. periapical. C. periapical and occlusal. D. panoramic.

C. periapical and occlusal.

Aspiration prior to a local anesthetic injection reduces the A. toxicity of local anesthetic. B. toxicity of vasoconstrictor. C. possibility of intravascular administration. D. possibility of paresthesia.

C. possibility of intravascular administration.

The primary factor for selecting periodontal flap surgery rather than gingivectomy is A. presence of gingival edema. B. pocket depth. C. presence of subgingival calculus. D. need for access to the bony defect

C. presence of subgingival calculus.

The mesial and distal walls of a Class I amalgam preparation diverge toward the occlusal surface in order to A. resist the forces of mastication. B. provide resistance and retention form. C. prevent undermining of the marginal ridges. D. extend the preparation into areas more readily cleansed.

C. prevent undermining of the marginal ridges.

The location and extent of subgingival calculus is most accurately determined clinically by A. radiopaque solution used in conjunction with radiographs. B. disclosing solution. C. probing with a fine instrument. D. visual inspection

C. probing with a fine instrument.

Clasps should be designed so that upon insertion or removal of a partial denture, the reciprocal arms contact the abutment teeth when the retentive arms pass over the height of contour in order to A. prevent distortion of the clasps. B. assure complete seating of the framework. C. provide needed support to abutment teeth during a period of added stress.

C. provide needed support to abutment teeth during a period of added stress

Bite-wing radiographs are most valuable for detecting A. hyperemia of the pulp. B. occlusal carious lesions. C. proximal surface caries. D. cervical caries.

C. proximal surface caries.

A 75 year old male patient whose wife died 10 months ago presents for his recall appointment. Looking wasted and fatigued, he confirms he has lost about 6kg in the last 8 months but is otherwise in good health. The most appropriate management for this patient is to A. recommend that he drink 3 cans of a nutritional supplement each day. B. refer him to a qualified dietician/nutritionist and follow up after his appointment. C. refer him back to his physician requesting a more thorough assessment. D. provide him with a copy of Canada's Food Guide to Healthy Eating.

C. refer him back to his physician requesting a more thorough assessment.

The function of the fixer solution is to A. bring out the contrast. B. convert the latent image to black metallic silver. C. remove unexposed silver halide. D. continue the action of the developer.

C. remove unexposed silver halide.

An 86 year old patient with poor oral hygiene has a cavitated lesion with active caries. The lesion is on the vestibular root surface of a maxillary posterior tooth, and the patient does not want any mercury in their mouth. The most appropriate restorative material for this lesion is a/an A. microfill resin composite. B. hybrid resin composite. C. resin-modified glass ionomer. D. amalgam.

C. resin-modified glass ionomer.

The Frankel functional regulator appliance performs all of the following EXCEPT A. increasing vertical dimension. B. repositioning the mandible forward. C. retraction of the maxillary molars. D. expansion of the dental arches.

C. retraction of the maxillary molars.

A patient presents with pain from tooth 4.7 which is an abutment for a 4 unit bridge from 4.4 to 4.7. Clinical and radiographic examinations reveal tooth 4.7 has extensive distal caries and apical radiolucency. The most appropriate initial management is to A. prescribe an antibiotic and an analgesic and reappoint the patient. B. perform endodontic therapy through the 4.7 crown. C. section the bridge at 4.4, remove 4.7 crown and assess 4.7. D. remove entire bridge and assess restorability of abutments.

C. section the bridge at 4.4, remove 4.7 crown and assess 4.7.

The most appropriate time to begin orthodontic correction of an Angle Class II malocclusion is A. following eruption of the maxillary first permanent molars. B. following eruption of the maxillary permanent central and lateral incisors. C. several months prior to the pubertal growth spurt. D. during the pubertal growth spurt.

C. several months prior to the pubertal growth spurt.

The most appropriate restoration for a primary first molar with extensive carious destruction of the crown is a A. posterior composite resin. B. pin retained amalgam. C. stainless steel crown. D. resin-modified glass ionomer.

C. stainless steel crown.

A clinical sign of unilateral fracture of the body of the zygoma is A. cerebrospinal rhinorrhea. B. impaired hearing. C. subconjunctival haemorrhage. D. otorrhea.

C. subconjunctival haemorrhage.

A dentist must be prudent in deciding how far to follow a patient's informed choice for suboptimal treatment because A. the law protects a patient's right to make poor decisions. B. a patient's informed choice must always be honoured. C. the principle of do-no-harm overrides the patient's personal choice

C. the principle of do-no-harm overrides the patient's personal choice

During the cementation of a gold inlay, the best way to assure that it stays well seated in the cavity until the final setting of the cement is A. to ask the patient to keep the mouth open and to apply no pressure on the inlay. B. to ask the patient to gently tap the teeth together continuously. C. to apply firm pressure to the inlay. D. to tap on the inlay with an orange wood stick and mallet.

C. to apply firm pressure to the inlay.

Erratic and inconsistent electric pulp test results can be explained by all of the following EXCEPT A. the presence of multiple canals in various stages of pulp pathosis. B. failure to isolate and dry the tooth. C. tooth mobility. D. poor contact between the electrode and the tooth.

C. tooth mobility.

The most common risk associated with vital bleaching using 10% carbamide peroxide in a custom tray is A. superficial enamel demineralization. B. soft tissue reaction. C. tooth sensitivity. D. cytotoxicity

C. tooth sensitivity.

The most common cause of malocclusion with a Class I molar relationship is A. a thumbsucking habit. B. crossbite in the posterior segments. C. tooth size and jaw size discrepancy. D. improper eruption of permanent first molars

C. tooth size and jaw size discrepancy.

When prescribing antibiotics for an orofacial infection in a healthy elderly patient, the usual adult dose and duration of the prescription should be written using the following guidelines. The dose is A. decreased by one half, duration unchanged. B. decreased by one third, duration unchanged. C. unchanged, duration unchanged. D. increased by one third, duration unchanged. E. unchanged, duration extended by one half.

C. unchanged, duration unchanged.

Periapical surgery is CONTRAINDICATED for a tooth that has a A. large periapical lesion. B. sinus tract related to a periapical lesion. C. vertical root fracture. D. a post and core retained crown.

C. vertical root fracture.

The full palatal major connector is indicated where A. there is a high, narrow palatal vault. B. a well-defined, undercut palatal torus is present. C. very few teeth remain in a flat or U-shaped arch. D. palatal tissue is soft and compressible.

C. very few teeth remain in a flat or U-shaped arch.

In primary teeth, a pulpotomy using calcium hydroxide A. will cause an acute inflammatory reaction. B. is successful treatment in 90 percent of cases. C. will cause internal resorption. D. is the treatment of choice for small mechanical exposures. E. will stimulate apical closure.

C. will cause internal resorption.

When a simple tipping force is applied to the crown of a single-rooted tooth, the centre of rotation is located A. at the apex. B. at the cervical line. C. within the apical half of the root. D. within the cervical one third of the root.

C. within the apical half of the root.

To decrease abutment tooth sensitivity, a fixed bridge may be temporarily seated using A. polycarboxylate cement. B. acrylic resin cement. C. zinc oxide eugenol cement. D. glass ionomer cement.

C. zinc oxide eugenol cement.

The most appropriate material of choice for obturating the root canal system of a primary tooth is A. MTA. B. gutta percha. C. zinc-oxide eugenol. D. formocresol.

C. zinc-oxide eugenol.

A surgical flap approach to periodontal pocket elimination permits A. healing by primary intention. B. retention of gingiva. C. access to perform osseous recontouring. D. All of the above.

D. All of the above.

In clinical dentistry, stiffness of wire is a function of A. length of the wire segment. B. diameter of the wire segment. C. alloy composition. D. All of the above. E. None of the above.

D. All of the above.

Needle deflection increases as A. depth of injection increases. B. needle length increases. C. needle gauge decreases. D. All of the above.

D. All of the above.

The success of indirect pulp capping is dependent upon A. removal of all caries at the enamel-dentin junction. B. use of calcium hydroxide. C. a well sealed restoration. D. All of the above.

D. All of the above.

Which of the following are mechanisms of growth of the naso-maxillary complex? A. Sutural. B. Cartilaginous. C. Appositional. D. All of the above.

D. All of the above.

Which of the following can be associated with an impacted tooth? A. Periapical osseous dysplasia (periapical cemento-osseous dysplasia). B. Peripheral odontogenic fibroma. C. Cementoblastoma. D. Ameloblastoma.

D. Ameloblastoma.

Which of the following statements concerning the airway is correct? A. Sympathetic fibres constrict the bronchioles. B. The trachea is membranous posteriorly to accommodate the pulsations of the aorta. C. The right primary bronchus forms two secondary bronchi. D. An aspirated foreign body would likely fall into the right primary bronchus.

D. An aspirated foreign body would likely fall into the right primary bronchus.

A 4 year old has generalized bone loss, mobile teeth and generalized calculus. Which condition should NOT be included in the differential diagnosis? A. Cyclic neutropenia. B. Papillon-Lefèvre syndrome. C. Chédiak-Higashi syndrome. D. Crouzon syndrome. E. Leukocyte adhesion deficiency syndrome.

D. Crouzon syndrome.

Tooth 3.6 has a disto-occlusal amalgam restoration with a gingival overhang. There is radiographic evidence of bone loss and deep probing depths with bleeding upon probing. Which of the following types of microorganisms are most likely associated with the subgingival environment in this site? A. Gram-positive and aerobic. B. Gram-positive and anaerobic. C. Gram-negative and aerobic. D. Gram-negative and anaerobic.

D. Gram-negative and anaerobic.

A patient suddenly becomes pale and sweaty after an injection of 4ml of lidocaine 2% with epinephrine l:l00,000. The radial pulse is slow and steady. The respiration is slow. The blood pressure is 80/60. What is the most probable diagnosis? A. A toxic reaction to lidocaine. B. A toxic reaction to epinephrine. C. An allergic reaction to the local anesthetic. D. Incipient syncope. E. An impending adrenal insufficiency.

D. Incipient syncope.

Solitary eosinophilic granuloma is associated with A. multiple myeloma. B. hyperparathyroidism. C. hypoparathyroidism. D. Langerhans cell histiocytosis.

D. Langerhans cell histiocytosis.

Which root surfaces are most likely to have concavities which make root planing difficult? A. Distal surfaces of mandibular canines. B. Distal surfaces of maxillary canines. C. Mesial surfaces of maxillary central incisors. D. Mesial surfaces of maxillary first premolars.

D. Mesial surfaces of maxillary first premolars.

What is the most likely diagnosis of an ulcerated gingival lesion whose biopsy report confirms epithelial basal layer separation from the lamina propria? A. An aphthous ulcer. B. Erosive lichen planus. C. Pemphigus vulgaris. D. Mucous membrane pemphigoid (cicatricial pemphigoid).

D. Mucous membrane pemphigoid (cicatricial pemphigoid).

In the preparation of gypsum products, an increase in the water/powder ratio will A. increase the surface hardness. B. increase the compressive strength. C. accelerate the setting reaction. D. None of the above.

D. None of the above.

Which of the following explanations of the caries process is most appropriate for a 10 year old patient who snacks frequently on soft drinks and doughnuts? A. When you eat doughnuts and soft drinks, because of all the sugar in them you'll get cavities. B. Bacteria in your mouth are the main cause of caries. C. Not brushing your teeth means the sugar from your snack attacks your teeth for about twenty minutes. D. The 'bugs' in your mouth eat the sugar in the food you eat, and change it into acid which can make holes in your teeth.

D. The 'bugs' in your mouth eat the sugar in the food you eat, and change it into acid which can make holes in your teeth.

Which of the following teeth is the LEAST desirable to use as an abutment tooth for a fixed partial denture? A. Tooth with pulpal involvement. B. Tooth with minimal coronal structure. C. Tooth rotated and tipped out of line. D. Tooth with short, tapered root and a long clinical crown.

D. Tooth with short, tapered root and a long clinical crown.

Which of the following symptoms are consistent with an anaphylatic reaction to penicillin? A. Deafness, dizziness, acute anemia and bronchial constriction. B. Crystalluria, nausea, vomiting, diarrhea, and bronchial constriction. C. Oliguria, hematuria, bronchial constriction, and cardiovascular collapse. D. Urticaria, diarrhea, bronchial constriction and cardiovascular collapse.

D. Urticaria, diarrhea, bronchial constriction and cardiovascular collapse.

Premature loss of a primary maxillary second molar usually produces a malocclusion in the permanent dentition that is characterized by A. anterior crowding. B. labially displaced maxillary canines. C. delayed eruption of the permanent first molar. D. a Class II molar relationship on the affected side. E. a Class III molar relationship on the affected side.

D. a Class II molar relationship on the affected side.

Management of a "dry socket" should include A. saline irrigation of socket. B. vigorous curettage of the socket. C. placement of topical antibiotics in the socket. D. a prescription for systemic antibiotics.

D. a prescription for systemic antibiotics.

Clinical examination of a 15 year old girl shows permanent central incisors, permanent canines and primary canines all in contact and anterior to the premolars. The most likely cause is A. ankylosed permanent canines. B. ankylosed primary canines. C. impacted permanent lateral incisors. D. congenitally missing permanent lateral incisors.

D. congenitally missing permanent lateral incisors.

High humidity in a room where zinc oxide and eugenol impression paste is being mixed will A. increase the setting time. B. not affect the setting. C. prevent any setting. D. decrease the setting time.

D. decrease the setting time.

For an adult patient, the recommended time interval between bite-wing radiographic examination for the detection of dental caries is A. 6 months. B. 12 months. C. 24 months. D. dependent upon caries risk.

D. dependent upon caries risk.

The effects of polymerization shrinkage of composite resin must be taken into account for all the following EXCEPT the placement of a/an A. occlusal restoration. B. mesiocclusal restoration. C. vestibular restoration. D. direct veneer restoration.

D. direct veneer restoration.

A smooth-surfaced, solid, exophytic oral mucosal lesion with no colour change is most likely a/an A. mucocele. B. papilloma. C. hemangioma. D. fibroma. E. intramucosal nevus.

D. fibroma.

The junctional epithelium, once it has migrated apically, attaches to the cementum by means of A. collagen fibres. B. oxytalan fibres. C. desmosomes. D. hemidesmosomes.

D. hemidesmosomes.

Acetylsalicylic acid should be avoided in each of the following EXCEPT A. gastric ulcer. B. gout. C. severe asthma. D. hyperlipidemia. E. type 2 diabetic.

D. hyperlipidemia.

The primary purpose of surgical therapy for the treatment of periodontitis is to A. apically position the flap. B. eliminate periodontal pockets. C. remove the ulcerated epithelium of the periodontal pocket. D. improve access for removal of local etiologic factors.

D. improve access for removal of local etiologic factors.

An Angle Class II, division 1 malocclusion can be differentiated from an Angle Class II, division 2 malocclusion based upon the A. molar relationship. B. severity of the Angle Class II malocclusion. C. amount of overbite. D. inclination of maxillary incisors. E. amount of crowding present.

D. inclination of maxillary incisors.

A surgical template (stent/guide) for an immediate maxillary denture is used to A. control hemorrhage while the new denture is being fabricated. B. protect the extraction sites while fitting the denture. C. assist in remounting the denture prior to refining the occlusion. D. indicate areas that require additional hard or soft tissue reduction.

D. indicate areas that require additional hard or soft tissue reduction.

A 30 year old patient has bilateral asymptomatic, bony hard nodules on the lingual surface of the anterior mandible. The most appropriate management is A. excisional biopsy of the nodules. B. referral for potential premalignant colonic polyps. C. vitality test of teeth adjacent to the nodules. D. observation at routine recalls. E. a bone scan for metastatic tumours.

D. observation at routine recalls.

When preparing a posterior tooth for an extensive amalgam restoration, a retentive pin hole preparation should be placed A. perpendicular to the pulpal floor. B. parallel to the contour of the final restoration. C. angled 30° away from the pulp chamber. D. parallel to the external root contour.

D. parallel to the external root contour.

A lowering of serum calcium is the stimulus for the endogenous release of A. thyroid hormone. B. adrenocortical hormone. C. insulin. D. parathyroid hormone. E. adrenalin.

D. parathyroid hormone.

A possible complication of acute osteomyelitis of the mandible is A. trismus. B. cavernous sinus thrombosis. C. facial nerve paralysis. D. paresthesia of the inferior alveolar nerve.

D. paresthesia of the inferior alveolar nerve.

Odontoblast gap junctions A. adhere the cells to one another. B. attach the cells to the basement membrane. C. seal off the dentin from the pulp. D. permit lateral cell-cell communication.

D. permit lateral cell-cell communication.

An 8 year old patient with all primary molars still present exhibits a cusp-to-cusp relationship of permanent maxillary and mandibular first molars and good alignment of the lower incisors. The management of this patient should be to A. refer for orthodontic consultation. B. use a cervical headgear to reposition maxillary molars. C. disk the distal surfaces of primary mandibular second molars. D. place patient on appropriate recall schedule.

D. place patient on appropriate recall schedule.

The most common infrabony site for breast cancer to metastasize to the maxillofacial region is the A. anterior maxilla. B. anterior mandible. C. posterior maxilla. D. posterior mandible. E. hard palate.

D. posterior mandible.

"Cuspid guided occlusion" occurs when the A. teeth on the nonworking side make contact in lateral excursions. B. teeth on the working side make contact in lateral excursions. C. canine and lateral incisors make contact in lateral excurison. D. posterior teeth make no contact in lateral excursions on the working side.

D. posterior teeth make no contact in lateral excursions on the working side.

Low serum levels of parathyroid hormone and vitamin D combined with low bone mass in the skeleton are consistent with the diagnosis of A. hypoparathyroidism. B. hypothyroidism. C. dietary calcium deficiency. D. postmenopausal osteoporosis.

D. postmenopausal osteoporosis.

In dental radiology, patient protection from radiation is most important for A. patients receiving antibiotics. B. patients receiving corticosteroids. C. individuals over fifty-years of age. D. pregnant women. E. young adults.

D. pregnant women.

In the mandibular dental arch of a 12-year old boy, the permanent first molars are in contact with the first premolars and the crowns of the second premolars have erupted lingually. The likely cause is A. ankylosis of the mandibular second premolars. B. lack of space. C. teeth too large for the dental arch. D. premature loss of deciduous second molars. E. faulty lingual eruption of the second premolars.

D. premature loss of deciduous second molars.

A sedative drug should A. cause excitement. B. eliminate all sensation. C. produce unconsciousness. D. produce a mild state of central nervous system depression.

D. produce a mild state of central nervous system depression.

When prescribing nonsteroidal anti-inflammatory drugs (NSAIDs), it is important to consider that prostaglandins A. impair blood coagulation. B. induce vasoconstriction. C. prevent edema. D. protect the gastric mucosa.

D. protect the gastric mucosa.

The greatest cariogenic potential is exhibited by A. cheese. B. apples. C. chewing gum. D. raisins.

D. raisins.

The main purpose of collimation of an x-ray beam is to A. permit the use of lower kilovoltage during exposure. B. filter out useless short wavelength rays. C. permit use of the long cone technique. D. reduce the diameter of the primary beam. E. reduce exposure time.

D. reduce the diameter of the primary beam.

On a semi-adjustable articulator, the incisal guidance is the mechanical analogue of A. horizontal guidance. B. the curve of Monson. C. the curve of Spee. D. relationship between the anterior teeth.

D. relationship between the anterior teeth.

A facebow is used to record the A. vertical dimension of occlusion. B. horizontal condylar inclination. C. incisal guidance. D. relationship of the maxilla to the horizontal hinge axis.

D. relationship of the maxilla to the horizontal hinge axis.

A patient presents with a chief complaint of "severe pain in my right ear" which began when eating, three hours ago. An examination reveals tenderness over the right preauricular region, maximum interincisal opening of 21 mm with deflection to the right, right lateral excursion of 9 mm and left lateral excursion of 2 mm. The most likely diagnosis is A. left anterior disc displacement with reduction. B. right anterior disc displacement with reduction. C. left anterior disc displacement without reduction. D. right anterior disc displacement without reduction.

D. right anterior disc displacement without reduction.

Planing the enamel at the gingival cavosurface of a Class II amalgam preparation on a permanent tooth A. should result in a long bevel. B. is contraindicated because of the low edge strength of amalgam. C. is unnecessary since the tooth structure in this area is strong. D. should remove unsupported enamel which may fracture. E. should result in a sharp gingivoproximal line angle.

D. should remove unsupported enamel which may fracture.

Firm contact between approximating teeth is important because it A. locates the marginal ridges of each tooth. B. keeps the teeth from having any movement during function. C. insures proper cusp form and increases masticatory efficiency. D. stabilizes the dental arches and gives protection to the gingival papillae.

D. stabilizes the dental arches and gives protection to the gingival papillae.

A 57 year old man received 10mg of diazepam intravenously. He becomes unresponsive to verbal stimuli, and his respirations are depressed to 10 per minute. Appropriate treatment is to A. administer ephedrine. B. observe the patient. C. force the patient to drink coffee. D. support respiration with oxygen.

D. support respiration with oxygen.

For a patient with complete dentures, insufficient space between the maxillary tuberosity and the retromolar pad will require A. avoiding covering the pad with the mandibular base. B. not covering the tuberosity with the maxillary base. C. surgically reducing the retromolar pad. D. surgically reducing the maxillary tuberosity.

D. surgically reducing the maxillary tuberosity.

A "broken stress" or "non-rigid" connector is indicated for a fixed partial denture when A. the retainers can be so prepared as to have equal retentive qualities. B. 2 or 3 teeth are to be replaced. C. constructing a mandibular fixed prosthesis. D. the abutments cannot be prepared in parallel without excessive removal of tooth structure.

D. the abutments cannot be prepared in parallel without excessive removal of tooth structure.

In taking an interocclusal wax record in a protrusive position, the dentist should examine the wax record to insure that A. the incisal edges of the anterior teeth have made contact. B. the patient has not closed in a lateral position. C. all cusps have penetrated the wax record and are in contact with the opposing teeth. D. there is no perforation of the wax record.

D. there is no perforation of the wax record.

Selection of the appropriate kilovoltage for dental films is influenced by A. line voltage fluctuation. B. diameter of the primary beam of radiation. C. type of timer. D. tissue density. E. filter thickness.

D. tissue density.

A single hypoplastic defect located on the labial surface of a maxillary central incisor is most likely due to a/an A. dietary deficiency. B. endocrine deficiency. C. tetracycline therapy. D. trauma to the maxillary primary central incisor. E. high fluoride intake.

D. trauma to the maxillary primary central incisor.

A direct or indirect pulp cap has the greatest chance of clinical success when there is a A. history of spontaneous pain. B. prolonged response to cold stimulus. C. apical lesion. D. vital pulp.

D. vital pulp.

The earliest radiographic sign of traumatic occlusion is A. hypercementosis. B. root resorption. C. alteration of the lamina dura. D. widening of the periodontal ligament space. E. ankylosis.

D. widening of the periodontal ligament space.

The use of an intra-coronal attachment is CONTRAINDICATED for a tooth A. that is nonvital. B. requiring a core procedure build up. C. supporting a partial denture. D. with short crown length.

D. with short crown length.

In the bisecting angle principle of intraoral radiography, the radiopacity that can obliterate the apices of maxillary molars is the A. maxillary sinus. B. palatine bone and the zygoma. C. orbital process of the zygomatic bone. D. zygoma and the zygomatic process of the maxilla.

D. zygoma and the zygomatic process of the maxilla.

Which of the following periodontal procedures is indicated on a maxillary canine that will receive a full crown with subgingival margins when the abutment has 1mm of attached gingiva, no sign of inflammation or loss of attachment? A. Root planing. B. Coronally positioned flap. C. Localized gingivectomy. D. Autogenous connective tissue graft. E. There is no indication that this tooth requires periodontal treatment.

E. There is no indication that this tooth requires periodontal treatment.

Side effects of chemotherapeutic treatment for malignancy include A. thinning of the oral mucosa. B. ulceration. C. necrosis. D. spontaneous bleeding. E. all of the above.

E. all of the above.

Initial scaling and oral hygiene instruction in the treatment of periodontitis results in all of the following EXCEPT A. pocket shrinkage. B. decreased hemorrhage during surgery. C. evaluation of the patient's motivation. D. improved healing after surgery. E. correction of pathological migration of teeth.

E. correction of pathological migration of teeth.

A 22 year old patient has been experiencing general malaise, fever, sore throat and coughing for one week. There are multiple ulcerations of the oral mucosa, crusting of the lips and red circular lesions on the palms of the hands. The most likely diagnosis is A. gonorrhea. B. infectious mononucleosis. C. acute herpetic gingivostomatitis. D. AIDS. E. erythema multiforme.

E. erythema multiforme.

The most appropriate time for surgical treatment of a patient with mandibular prognathism is A. just before the beginning of the prepubertal growth spurt. B. just after the end of the prepubertal growth spurt. C. just before eruption of the second permanent molars. D. just after eruption of the second permanent molars. E. following the completion of growth.

E. following the completion of growth.

Which of the following phases in the setting reaction of dental amalgam is weak and corrosion-prone? A. α1. B. α2. C. gamma D. gamma 1. E. gamma 2.

E. gamma 2

Ankylosed primary second molars may clinically exhibit A. percussion sensitivity. B. discolouration. C. temperature sensitivity. D. buccolingual displacement. E. infra-occlusal position.

E. infra-occlusal position.

Heavy cigarette smoking significantly increases the incidence of A. aphthous stomatitis. B. geographic tongue. C. lichen planus. D. atrophic glossitis. E. mucosal pigmentation.

E. mucosal pigmentation.

Ocular lesions may be associated with A. lichen planus. B. herpangina. C. necrotizing ulcerative gingivitis. D. leukoplakia. E. mucous membrane pemphigoid (cicatricial pemphigoid).

E. mucous membrane pemphigoid (cicatricial pemphigoid).

A 45 year old patient has 32 unrestored teeth. The only defects are deeply stained grooves in the posterior teeth. Clinical examination reveals no evidence of caries in the grooves. The most appropriate management is A. application of a resin based pit and fissure sealants. B. application of a glass ionomer pit and fissure sealants. C. conservative Class I amalgams. D. prophylactic odontotomy. E. no treatment.

E. no treatment.

A patient complains of acute pain 24 hours after the insertion of a restoration in a tooth with no preexisting periapical pathology. The tooth is vital and tender to percussion. The radiograph will show A. an apical radiolucency. B. acute osteitis. C. root resorption. D. condensing osteitis. E. normal lamina dura.

E. normal lamina dura.

The most appropriate management for a gingival cyst of the newborn is A. curettage. B. incisional biopsy. C. cytologic smear. D. excisional biopsy. E. observation.

E. observation.

An 8 year old patient with all primary molars still present exhibits a cusp-to-cusp relationship of permanent maxillary and mandibular first molars. The management of this patient should be to A. plan serial extractions for more normal adjustment of the occlusion. B. refer the patient to an orthodontist for consultation. C. place a cervical headgear to reposition maxillary molars. D. disk the distal surfaces of primary mandibular second molars to allow normal adjustment of permanent molars. E. observe.

E. observe.

knocked out his front tooth but that it is still intact. Your instructions should be to A. put the tooth in water and come to your office at the end of the day. B. wrap the tooth in tissue and come to your office in a week's time. C. put the tooth in alcohol and come to your office immediately. D. place tooth under the tongue and come to your office immediately. E. place the tooth in milk and come to your office immediately.

E. place the tooth in milk and come to your office immediately.

To obtain block anesthesia of the second division of the trigeminal nerve, the solution must be deposited in proximity to the A. foramen ovale. B. pterygoid plexus. C. foramen spinosum. D. infraorbital foramen. E. pterygopalatine fossa.

E. pterygopalatine fossa.

The best way to protect the abutments of a Class I removable partial denture from the negative effects of the additional load applied to them is by A. splinting abutments with adjacent teeth. B. keeping a light occlusion on the distal extensions. C. placing distal rests on distal abutments. D. using cast clasps on distal abutments. E. regular relining of the distal extensions.

E. regular relining of the distal extensions.

The characteristic pain of trigeminal neuralgia is A. dull and prolonged. B. infrequent, sharp and prolonged. C. regularly recurrent, dull and persistent. D. annoying but controlled with salicylates. E. sharp, stabbing and excruciating.

E. sharp, stabbing and excruciating.

Angiotensin II converting enzyme is primarily produced in A. osteoblasts. B. epithelial cells of the small intestine. C. hepatocytes. D. Kupffer cells. E. vascular endothelial cells of the lung.

E. vascular endothelial cells of the lung.

Following trauma, bluish-grey discolouration of the crown of an anterior tooth is due to E. external resorption. F. pulpal hemorrhage. G. discoloured composite restoration. H. chromogenic bacteria.

F. pulpal hemorrhage.


Related study sets

Chapter 7 Self Assessment (Conceptual)

View Set

MED SURG I Chapter 16: Postoperative Nursing Management

View Set

Nutrition Chapter 9: Water and Minerals

View Set

Quickbooks 2014 Flashcards Chapter 6

View Set

Anatomy Ch 4: Tissues (Mastering A&P)

View Set

Microsoft PowerPoint Assessment / LinkedIn Skills Assessments

View Set

Computing Devices IoT, Operating Systems, Peripherals and Connectors, Core Hardware Components

View Set